SlideShare a Scribd company logo
1 of 533
COMPENDIO
Dr. Alejandro Sandoval G. Travesi
CMN La Raza
ENARM
Consejos
1. Leer diariamente y en bloques
2. Adiós Partys un tiempo
3. Has ejercicio y come bien durante el estudio
4. Toma algún curso bueno si tienes la posibilidad
5. Ten Fe.
HEMATOLOGIA
Hemoglobina Hematocrito
Recién nacido
19.5 ± 5.0 54 ± 10
Mujeres
14.0 ± 2.0 42 ± 5
Hombres 16.0 ± 2.0 47 ± 7
Cifras normales de hemoglobina y hematocrito
al nivel del mar (Wintrobe)
• BANDAS
• 1-3%
QUIMICA Y ELECTROLITOS
• GLUCOSA
• 70- 105
• UREA
• 10- 50
• ACIDO URICO
• 2.5- 7.7
• NITROGENO UREICO
• 6- 20
• CREATININA SERICA
• 0.5- 1.2
• COLESTEROL
• 120- 200
• TRIGLICERIDOS
• 10- 160
• HDL
• 40- 90
• LDL
• 0 – 120
• VLDL
• 0- 50
• ALBUMINA
• 3.5- 5.5
• SODIO
• 135- 145
• POTASIO
• 4- 5.3
• CLORO
• 98- 106
PFH Y ENZIMAS CARDIACAS
• BILIRRUBINA TOTAL
• 0 – 1.5
• BILIRRUBINA INDIRECTA
• 0 – 1.50
• BILIRRUBINA DIRECTA
• 0 – 0.5
• AST/TGO
• 6 – 38
• ALT/TGP
• 0 – 35
• DHL
• 100 – 190
• GGT
• 7 – 50
• CPK
• 0 – 226
GASES ARTERIALES
• PH
• 7.35--7.45
• PCO2
• 33-45mmHg
• PO2
• 75-105mmHg
OSMOLARIDAD SERICA Y LCR
• OSMOLALITY SERUM
• 275-295mOsmol/Kg
• LCR
• CELLCOUNT
• 0.5cells/mm3
• CHLORIDE
• 118--132mEq/L
• GAMMA GLOBULIN
• 3--12% total Proteins
• GLUCOSE
• 40--70mg/dL
• PRESSURE
• 70--180mm h2O
• PROTEINS,TOTAL
• < 40mg/dL
ANEMIA POR DEFICIT DE HIERRO
ANEMIA POR DEFICIT DE HIERRO
ANEMIA MEGALOBLÁSTICA
# de Eritrocitos Otros
estudios
Aspirado de
médula ósea:
HIPERCELULAR
bilirrubina
indirecta = lig.
DHL =
Hemoglobina
Hematocrito
V.G.M.
H.G. M.
NC.M.H.G.
Reticulocitos N o lig.
Leucocitos
Diferencial L=
Frotis N=
Plaquetas
VITAMINA B12
A que se refiere el termino POLICITEMIA ESPURIA?
R = AUMENTO DEL HTO por aumento de la concentración del VOLUMEN PLASMÁTICO NO POR EL > DE LA
MASA ERITROCITARIA
Cual es el cc de la TROMBOSIS ESENCIAL?
R = El primer signo puede ser trombosis mesentérica, hepática o portal. Eritromelalgia.
Cuales son los datos de laboratorio de la trombosis esencial?
1) PLAQUETAS >2, 000,000
2) FSP CON > DE PLAQUETAS GRANDES
3) MO CON > DE MEGACARIOCITOS
4) GEN FILADELFIA AUSENTE que descarta leucemia mieloide crónica
Que otras enfermedades pueden causar > plaquetario sin ser TE?
R = AR, CUCI e infección crónica
Cual es el manejo de la TE?
R = HIDROXIUREA y ASA hasta reducir a < 500, 000
Que es la MIELOFIBROSIS?
1) Trastorno mieloproliferativo caracterizado por FIBROSIS DE LA MO, esplenomegalia y cuadro leuco
eritroblastico en FSP con POIQUILOCITOS EN LAGRIMA.
2) HEMATOPOYESIS EXTRAMEDULAR
Cual es la etiología de la mielofibrosis?
R = Se desarrolla en respuesta a un > EN LA SECRECIÓN DE FACTOR DE CRECIMIENTO DERIVADO DE LAS PLAQUETAS
Cual es el cc de la mielofibrosis?
1) Fatiga, distensión abdominal por esplenomegalia, DOLOR OSEO PRINCIPALMENTE RETROESTERNAL,
hepatomegalia.
2) Hemorragias por secuestro de plaquetas
3) HIPERTENSIÓN PORTAL Y VARICES ESOFÁGICAS POR HEMATOPOYESIS HEPÁTICA
Cuales son los datos de laboratorio y punción obtenidos en la mielofibrosis?
1) Anemia
2) FSP con POIQUILOCITOS EN LAGRIMA, formas inmaduras mieloides y PLAQUETAS GIGANTES
desgranuladas
3) PUNCIÓN SECA por aumento de fibras reticulares
Cual es la triada clásica de la mielofibrosis?
1) POIQUILOCITOS EN LAGRIMA
2) SANGRE LEUCO-ERITROBLASTICA
3) PLAQUETAS GIGANTES
4) Supervivencia de 3-5ª
Cual es el manejo de la mielofibrosis?
R = Transfusiones continuas, TETOSTERONA. TMO + talidomida
Cuando se considera LEUCEMIA según la OMS?
R = Cuando hay al menos 20% DE BLASTOS EN SANGRE O MO.
LEUCEMIA AGUDA LINFOBLASTICA
LEUCEMIA AGUDA LINFOBLASTICA
Cuales son las bases para el dx de LLC?
R = Es una malignidad ORIGINADA EN LOS LINFOCITOS B, en la cual cursan asintomáticos, CON
INFILTRACIÓN DE LB EN LOS ÓRGANOS
Cual es el cc de LLC?
R = Fatiga, LINFADENOPATIA Y HEPATO-ESPLENOMEGALIA
Cual es el hallazgo de laboratorio de la LLC?
1) LINFOCITOSIS AISLADA > 5000
2) MARCADOR CD 19 EN LB Y CD 5 EN LT
Cual es el manejo de la LLC?
R = Clorambucil
Cual es la estirpe histológica de la que proviene la LEUCEMIA DE CÉLULAS VELLOSAS?
R = LINFOCITOS B
Cual es el cc de LCV?
R = Fatiga, ESPLENOMEGALIA o asintomático
PUNCION SECA
I. MIELOFIBROSIS
• Cual es la etiología de la mielofibrosis?
R = Se desarrolla en respuesta a un > EN LA SECRECIÓN DE FACTOR DE CRECIMIENTO DERIVADO DE LAS
PLAQUETAS
• Cual es el cc de la mielofibrosis?
1) Fatiga, distensión abdominal por esplenomegalia, DOLOR OSEO PRINCIPALMENTE RETROESTERNAL,
hepatomegalia.
2) Hemorragias por secuestro de plaquetas
3) HIPERTENSIÓN PORTAL Y VARICES ESOFÁGICAS POR HEMATOPOYESIS HEPÁTICA
II. LEUCEMIA DE CELULAS VELLOSAS
• Cual es la estirpe histológica de la que proviene la leucemia de células vellosas?
R = LINFOCITOS B
• Cual es el cc de LCV?
R = Fatiga, ESPLENOMEGALIA o asintomático
Que es el MIELOMA MÚLTIPLE?
1) Es una neoplasia de células plasmáticas que sustituye la MO.
2) Hay DESTRUCCIÓN ÓSEA con formación de PARAPROTEÍNAS plasmáticas IgG o IgA
3) Hay HIPERVISCOCIDAD debido a paraproteínas
4) Puede provocar IRA por las paraproteínas
Cual es el cc del mieloma múltiple?
1) DOLOR ÓSEO 70% que aumenta con los movimientos, principalmente en la espalda CON EVIDENCIAS DE FRACTURAS
VERTEBRALES
2) Edad > 65ª , ANEMIA, dolor óseo, infección
3) Amiloidosis manifiesta: macroglosia, neutropatia, ICC o hepatomegalia
Cuales son los datos de laboratorio del MM?
1) HIPERCALCEMIA, Sx anémico con ANEMIA NORMO-NORMO por efecto supresor de eritropoyetina por IL6
2) NEFRITIS intersticial con pérdida DE CADENAS LIGERAS
3) Biopsia de MO con infiltración normal o difusa
Cual es el manejo de mieloma múltiple?
1) Vincristina, doxorrubicina y dexametazona
2) TMO autologo
3) TALIDOMIDA + DEXAMETAZONA PARA RECAÍDAS
4) Dx dif. Con plasmocitoma solitario que responde bien a la radioterapia que no presenta la clínica típica del mieloma
Como estadificas la carga tumoral en el MM?
1) BAJA: con espiga IgG <5 gr/dl sin hipercalcemia ni IR
2) ALTA: con espiga IgG > 7 gr, Hto <25% y calcio > 12
Que es la MACROGLOBULINEMIA DE WALDESTROM?
R = Enfermedad MALIGNA DE LB, con hibrido de LB célula plasmática con PARAPROTEINA IgM
Cuales son las causas secundarias más comunes de PTI?
1) Fármacos como la heparina
2) LES y LLC
Cual es el mecanismo por el cual la heparina causa trombocitopenia?
R = Auto anticuerpos IgG vs factor 4 plaquetario
Cual es el manejo de la PTI?
1) PREDNISONA 1-2 mg/kg/dia con respuesta en una semana
2) INMUNOGLOBULINA iv para aumento de plaquetas
3) ESPLENECTOMÍA
Que es la PURPURA TROMBOCITOPENICA TROMBOTICA?
R = Es una enfermedad caracterizada por ANEMIA HEMOLÍTICA MICROANGIOHEPATICA,
TROMBOCITPENIA y AUMENTO NOTABLE DE DHL SÉRICA (a diferencia de la PTI donde no hay
elevación de la DHL)
Cual es la etiología de la PTT?
R = Deficiencia de “PROTEASA” que divide al factor de VON WILLEBRAND (VIII), por lo tanto se acumula
dicho factor originando aglutinación de plaquetas en endotelio vascular. TAMBIÉN LO PUEDE
DESENCADENAR EL EMBARAZO, LOS ESTRÓGENOS ELEVADOS Y LAS INFECCIONES.
Cual es el cc de la PTT?
1) PENTADA CLASICA: Anemia hemolítica microangiopatica, Trombocitopenia, Afección neurológica, Falla
Renal y Fiebre. ATARF
2) Fiebre, palidez, PETEQUIAS
3) Anemia, hemorragia.
4) Síntomas neurales como HEMIPARESIA QUE DURA MINUTOS
DEFICIT DE ADHESION PLAQUETARIO
I. Que es la tromboastenia de GLANZMAN?
A. Es un trastorno AR que produce hemorragias ( trastorno
plaquetario). LAS PLAQUETAS NO PUEDEN AGREGARSE por que no
hay RECEPTORES (GLUCOPROTEINAS IIB Y IIIA) para el fibrinógeno.
B. Se maneja con desmopresina
I. Que es el síndrome de BERNARD-SOULTIER?
A. Es un trastorno AR que OCASIONA HEMORRAGIAS. Las PLAQUETAS
NO SE ADHIEREN por FALTA DEL RECEPTOR IIB para el factor de vWF
B. Se maneja con transfusion de plaquetas
A que se debe la HEMOFILIA TIPO C y cual es su principal cc?
R = Déficit del FACTOR XI y se manifiesta con HEMORRAGIAS LEVES.
Que es la HEMOFILIA TIPO A?
R = Es un trastorno hereditario AR LIGADO AL X con déficit del FACTOR VIII y SOLO
AFECTA A VARONES
Cual es el cc de la hemofilia tipo A?
R = HEMORRAGIA EN ARTICULACIONES de rodilla, tobillo, codo, músculos y aparato GI
Cuales son los datos de lab en la hemofilia tipo A?
1) TPT SE PROLONGA únicamente
2) FACTOR VIII DISMINUIDO Y VWF NORMAL
Cual es el manejo de la hemofilia tipo A?
R = Concentrados de FACTOR VIII 4000 UI para 70 kg
Como puede transmitirse la enfermedad de anticuerpos vs el factor VIII?
R = Puede desarrollarse POST PARTO o sin una causa, hemofilia A que han recibido
concentrados plasmáticos.
Cual es el cc de la enfermedad de anticuerpos vs el factor VIII?
R = Hemorragia grave
Cual es el manejo de elección en la enfermedad de anticuerpos vs el factor VIII?
R = CICLOFOSFAMIDA
Que es la HEMOFILIA TIPO B o enfermedad de Christmas?
R = Trastorno AR LIGADO AL X con déficit de FACTOR IX en el cual se eleva el TPT
Cual es el manejo de la hemofilia tipo B o enfermedad de Christmas?
R = Concentrados de FACTOR IX 6000 para 70 kg
Que factores de coagulación se ven afectados en COAGULOPATIA POR ENFERMEDAD
HEPÁTICA?
1) Se fabrican todos los factores de coagulación en el hígado EXCEPTO EL FACTOR VIII
2) Primero se ven afectados los vitamina K dependientes II, VII, IX y X y el factor V
3) Aumenta la fibrinólisis por la disminución de plasmina
Cual es el manejo de la coagulopatia por enfermedad hepática?
R = PLASMA FRESCO CONGELADO
Cual es el cc de la coagulación intravascular diseminada?
1) HEMORRAGIAS Y TROMBOSIS
2) Hemorragia +++ por punción, heridas o espontanea
3) Trombosis + como isquemia digital o gangrena
Que es el SÍNDROME DE TROSEAU en la coagulación intravascular diseminada?
R = Coagulación intravascular diseminada manifiesta como TROMBOSIS VENOSA SUPERFICIAL Y PROFUNDA POR CÁNCER
principalmente de manera recurrente.
Cuales son los datos de lab para la coagulación intravascular diseminada?
1) HIPOFIBRINOLEMIA
2) Aumento de productos de degradación de la fibrina, TROMBOCITOPENIA y AUMENTO DEL TP Y TPT
3) DÍMERO D como producto de degradación de la fibrina que aumenta si hay fallo hepático.
Cual es el manejo para la coagulación intravascular diseminada?
1) TRATAR EL TRASTORNO SUBYACENTE
2) Heparina, S- AMINOCAPROICO (AUMENTA EL FIBRINÓGENO)
3) El ÉXITO del tx consiste en que AUMENTE EL FIBRINÓGENO
4) CRIOPRECIPITADOS PARA FIBRINÓGENO
Como se diagnostica la disfibrinolemia?
R = Aumento del TIEMPO DE REPTILASA
Que medicamento se utiliza de elección en los estados de hipercoagubilidad?
R = WARFARINA
PATOLOGIA CELULA PATOGNOMONICA
ANEMIA DE CELULAS FALCIFORMES Cuerpos de Howell-Jolly:Estructuras únicas o
dobles, pequeñas y redondas. Se observan
como gránulos densos y de color azul rojizo o
violeta. Localizados excéntricamente
DEFICIT DE 6 GPDH Cuerpos de Heinz: Citoplasma de los glóbulos
rojos , aparecen como pequeños puntos
oscuros bajo el microscopio
LINFOMA DE HODKING Reed Stemberg cells: Celula B modificada por
lo general es muy grande dando la pariencia de
ojos de Buho con nucléolos prominentes en
forma de inclusiones, la variedad mononuclear
tiene un solo nucleo y un nucléolo prominente.
LEUCEMIA AGUDA Bastones de Auer: Son estructuras filiformes
dentro de los blastos
ANEMIA SIDEROBLASTICA Cuerpos de Papenheimer: Son acúmulos
de hemosiderina unida a proteínas. Consisten
en gránulos basófilos, con las tinciones
habituales, que además, se tiñen también de
azul con el colorante de Perls (azul de Prusia).
PATOLOGIA CELULA PATOGNOMONICA
INTOXICACION POR PLOMO, LEUCEMIA Y
TALASEMIA.
Punteado basofilo: Pueden ser agregados
ribosómicos originados por una degeneración
vacuolar del citoplasma o precipitados de
cadenas globínicas libres. Consiste en puntitos
basófilos, con las tinciones habituales, de
tamaño variable y dispersos por toda la
superficie del hematíe.
ANEMIA MEGALOBLASTICA Anillos de cabot: Están formados por restos de
la membrana nuclear o de microtúbulos.
Consisten en una especie de hilos basófilos,
con las tinciones habituales, que adoptan una
forma de anillo o de ocho y que pueden
ocupar toda la periferia celular.
CÉLULA WARTHIN-FINKELDEY Sarampión: Célula gigante multinucleótica con
citoplasma eosinofílico e inclusiones nucleares
PATOLOGIA
• De que patología son característicos los CUERPOS DE HEINZ?
R = Déficit de glucosa 6PDH
• De que patología son patognomónicos los cuerpos de HOWELL-JOLLY?
R = Anemia de células falciformes
• De que patología son patognomónicos los BASTONES DE AUER?
R = Leucemia mieloide aguda en la que además la distingue la presencia de “MIELOPEROXIDASA”
• Que es la LEUCEMIA MIELOGENA CRÓNICA?
R = Se caracteriza por SOBREPRODUCCIÓN DE CÉLULAS MIELOIDES (principalmente LEUCOCITOS), presenta EL COMOSOMA FILADELFIA
(translocacion de los brazos 9 y 22) BCR/ABL que produce “TIROCINA CINASA”.
• Que diferencia la LEUCEMIA MIELOIDE AGUDA de la leucemia linfoblastica aguda?
R = Mieloperoxidasa que no se produce en la LLA
• Cual es el hallazgo patognomónico de linfoma de Hodking?
R = RED STEMBERG CELLS (CÉLULA B MODIFICADA por lo general es muy grande dando la apariencia de ojos de Búho con nucléolos prominentes
en forma de inclusiones, la variedad mononuclear tiene un solo núcleo y un nucléolo prominente).
• Que patología te da trombos hialinos en la biopsia?
R = PTT y SUH
• Citogenetica reconocible con T (18:14) de que enfermedad es patognomónico?
R = Linfoma no Hodking
ENF. DE HODKING
• Característica de la Enfermedad de Hodgkin?
• Linfadenopatia indolora, Síntomas constitucionales, no se conoce su origen, diseminación
ordenada
• Cuadro clínico?
• MASA INDOLORA (CUELLO), mas frecuente en varones, diseminación ordenada, fiebre de larga
evolución, DISMINUCIÓN DE PESO, PRURITO INTENSO, sudación nocturna, DOLOR GANGLIONAR
(ALCOHOL)
• Diagnostico definitivo?
• Biopsia del ganglio linfático afectado, CÉLULAS DE REED-STENBERG, no hay anormalidades
cariotipicas
• El virus Epstein Barr con que enfermedad tumoral se ha relacionado?
• Enfermedad de Hodgking
• En la Enfermedad de Hodgking cual es la variedad histológica mas frecuente?
• ESCLEROSIS NODULAR
• En la Enfermedad de Hodgking cual es la variedad de mejor pronostico?
• VARIEDAD LINFOCITICA
• En la Enfermedad de Hodgking cual es la variedad menos frecuente?
• Depleción linfocitaria
• En la Enfermedad de Hodgking que se relaciona con VIH?
• DEPLECIÓN LINFOCITARIA
• Tratamiento de la de la Enfermedad de Hodgking?
1) MOPP (mecloretamina, Vincristina, procarbacina, prednisona)
2) ABVD (adriamicina, bleomicina, Vincristina, Dacarbacina)
• Clasificación de ann-arbor?
• Etapa 1 una sola cadena ganglionar
• Etapa 2 2 regiones ganglionares (mismo lado del diafragma)
• Etapa 3 regiones ganglionares en ambos lados del diafragma
• Etapa 4 metástasis
• (A asintomático, B síntomas constitucionales)
ENF. NO HODKING
• Característica de la Enfermedad no Hodgking?
• No tiene cél de Reed Stemberg, si hay citogenética reconocible T (8:14), correlación con proto-
oncogen C-myc
• Cuadro clínico de la E no H?
• Linfadenopatia desordenada, a menudo diseminada al momento del diagnostico, síntomas
constitucionales
• En que consiste el termino de ganglio de Richter?
1) Es un ganglio generalmente SUPRACLAVICULAR .
2) GANGLIO que comparte una LLC q posteriormente se convertirá (EVOLUCIONARA) en LINFOMA NO
HODGKIN
• Tratamiento de los linfomas no Hodgkin?
1) 1 solo ganglio afectado: radioterapia local … si esta asintomático valorar conducta expectante
2) Tx: clorambucilo o CVP+fludarabina, CHOP
• Tratamiento para un MALTOMA?
• Terapia para HELICOBACTER PYLORI
LEUCEMIA AGUDA
• Cáncer mas frecuente en niños?
 Leucemia siendo la aguda la mas frecuente (80%)
• El cáncer de los ganglios linfáticos se llama?
 Linfoma
• El cáncer de medula ósea?
 Leucemia
• Característica de la leucemia Linfoblastica Aguda?
 Tienen blastos (20% en MO y 90% en sangre periférica) y su caract.
Es la capacidad de infiltrar órganos (encías, pericardio, testículos, MO,
SNC etc), 2da. Capacidad de producir Citopenias o Pancitopenias
• Cuadro Clínico de LLA?
 Por procesos de infiltración hay: Hemorragias (petequias, equimosis) Adenopatías,
Hepato-esplenomegalias, puede llegar a una CID
• Diferencia entre LLA y la hipoplasia medular y purpura trombocitopenica
idiopática
 En que puede haber trombocitopenia pero TANTO EN LA HIPOPLASIA COMO EN
LA PURPURA NUNCA habrá ganglios linfáticos palpables o hepatoesplenomegalias
• Características de la CID?
 Tiempo de protrombina alargado, dímero D presente, trombocitopenia.
• Que tipo de leucemia mielocitica aguda tiene la característica de presentarse
como una CID
 La promielocitica y la monocitica aguda (habitualmente son de buen pronostico
mientras que no se presente con CID)
• Citogeneticas desfavorables?
 Monosomia 5 y 7, Cromosoma Ph t(4;11) (Encontrar Cromosoma ph en LMA(2%) es
de muy mal pronostico habitualmente se observa en LMC (98%)
• Con respecto a marcadores de superficie:
 Determinación de una Enzima desoxinucleotidil transferasa terminal….. es una LLA (95%)
• Si es MIELOPEROXIDASA POSITIVA…. LMA
 Marcador CD19, CD10…. Es una LLA de células B (mas frecuente)
 Marcador CD2, CD5, CD7…. Es una LLA de células T (menos frecuente)
• Tratamiento de Leucemias Mielocitica Agudas?
 1ra etapa: Inducción de la Remisión Daunorrubicina o Idarrubicina mas citarabina
 2da etapa: Quimioterapia intensa de repetición mas transplante de MO autologa
• Tratamiento de la Leucemia Linfoblastica aguda?
 1ra etapa: Inducción de la Remisión Vincristina, Prednisona, Daunorrubicina, L
Asparaginasa (4 medicamentos) Metrotexate (profilaxis del SNC)
 2da etapa: Quimioterapia a dosis altas mas transplante de MO
• Leucocitosis con Desviación a la DERECHA:
 Cuando los LINFOCITOS son los que están elevados (arriba del 20%)
• Leucocitosis con Desviación a la IZQUIERDA:
 Cuando los NEUTRÓFILOS están elevados
• Característica Histológicas de LMA?
 Mieloperoxidasas positivas, cuerpos de AUER en citoplasma
• Características Histológicas de LLA?
 Antígeno CALLA (Antígeno común de LLA) y PAS Acido periódico de Shift
positivo
• Citogenética Favorables?
 En px con LMA t(8:21), t(15:17),inv (cr16), Inv (p13; q22) buen
pronostico
LEUCEMIA CRONICA
• Característica de una LLC?
1) Linfocitos mayores de 5000, linfocitos de apariencia normal (madura), con expresión de CD19, y CD5
2) Origen: malignidad de linfocitos B (95%) inmunoincopetentes
• Cuadro clínico de una LLC?
 Se presenta en px MAYORES DE 50 AÑOS (90%), linfocitosis impresionante, linfadenopatia y
Hepatoesplenomegalia
• Clasificación de la LLC (CLASIFICACION RAI)
 ETAPA O Linfocitosis mas de 5000
 ETAPA 1 Linfocitosis MAS LINFADENOPATIA (revisar al px)
 ETAPA 2 Linfocitosis MAS ORGANOMEGALIA HEPATOESPLENOMEGALIA
 ETAPA 3 Linfocitosis mas anemia
 Etapa 4 Linfocitosis , lo anterior mas trombocitopenia (mal pronostico)
• Laboratorios de una LLC?
 LINFOCITOSIS AISLADA (mas de 5000), Leucos mayor de 20000, inmunogenotipo: CD19 (LLC de Celulas
B), CD5 (LLC de Celulas T)
• Tratamiento de LLC con sintomatología?
 CLORAMBUCILO alternativa fludarabina en px jóvenes
• Característica de LMC?
1) Leucocitosis, Cromosoma Ph t(9:22) presentes en un 95%,
2) Leucocitosis con desviación a la izquierda (aumento delos
neutrófilos)
• Tx de LMC?
 EL TX no es urgente Mesilato de Imatinib, alternativa Hidroxiurea
o interferon alfa
• Cuadro clínico de LMC?
 Crecimiento impresionante de bazo, mas leucocitosis con desviación
a la izq con serie mieloide, sensibilidad esternal, cromosoma Ph (95%)
SINDROME PARANEOPLASICO/ AC
• Frecuentemente presentan pleocitosis linfocitaria en LCR
con aumento de proteínas y de IgG. Algunos pacientes
presentan anticuerpos característicos en suero y LCR que
proveen una fuerte evidencia de que el cuadro
neurológico es paraneoplásico y que además sugieren el
tipo de cáncer asociado, por ejemplo el anticuerpo anti-
Yo se asocia con degeneración cerebelosa y cáncer
ginecológico, el anticuerpo anti-Hu con encéfalomielitis
paraneoplásica y cáncer pulmonar de células pequeñas y
el anticuerpo anti-Ki con opsocionus y cáncer de mama
MICELANEAS
• En un paciente con anemia ferropénica que
esperaría encontrar en su estudio de
laboratorio?
• Ferritina, ferremia y saturación de transferrina
bajas y transferrina alta.
• Por cuanto tiempo se realiza el tratamiento
de la anemia ferropenica?
• R = 6 meses
CARDIOLOGIA
ENARM
EKG
EKG
 I N T E R V A L O S
 PR:
Se denomina así, al espacio que va del inicio de la Onda P al comienzo de la Onda R.
Valor normal: 120 a 200 milisegundos ( 0.12 – 0.20 segundos).
 COMPLEJO QRS:
Corresponde a la activación del miocardio ventricular.
Valor Normal: 60 a 100 milisegundos ( 0.06 – 0.10 segundos).
 QT:
Se mide desde el inicio del Complejo QRS hasta el final de la Onda T y corresponde a la duración total de la Sístole
Ventricular.
Valor normal: 240 a 480 milisegundos ( 0.24 – 0.48 segundos ).
( Varia de acuerdo a la frecuencia cardiaca )
 S E G M E N T O
 ST:
Es el intervalo normalmente iso-electrico entre el final del Complejo QRS y el inicio de la Onda T.
Valor normal: 60 a 160 milisegundos ( 0.06 a 0.16 segundos )
EKG
• I N T E R V A L O Q T
 P r o l o n g a d o:
 C a r d i o p a t í a I s q u é m i c a.
 I. C. C.
 M i o c a r d i t i s.
 Drogas: Quinidina, Amiodarona, Antidepresivos triciclicos...
 H i p o m a g n e s e m i a.
 H i p o c a l c e m i a.
 H i p o k a l e m i a ?.
 A c o r t a d o:
 R e p o l a r i z a c i ó n p r e c o z.
 D r o g a s: Digital...
 H i p e r c a l c e m i a.
 H i p e r k a l e m i a.
HIPOKALEMIA
• < 3 mEq/L: onda T
plana, depresión ST,
ondas U
• < 2,5 mEq/L: onda U
prominente,
inversión onda T, PR
y QT prolongado,
QRS ensanchado
HIPERKALEMIA : POTASIO SÉRICO > 5 mEq/L
LEVE 5.0 - 5.5 mEq/L
MODERADA 5.5 - 6.0 mEq/L
SEVERA > 6.0 mEq/L
K+ 5.0 - 6.5 mEq/L Cambios mínimos
K+ 6.5 - 8.0 mEq/L Onda T picuda
Aplanamiento y desaparición de la onda P
Depresión del segmento ST
K+ 8.0 a más Ensanchamiento del QRS
BAV
Arritmias cardíacas
Ritmo idioventricular
Paro cardíaco en asistolia.
HIPERPOTASEMIA Y EKG
EJES ELECTRICOS EKG
PULSO VENOSO YUGULAR
AXVAYC
• Se distinguen fundamentalmente dos ondas, la "a" y la "v". La primera, la onda
"a", ocurre justo antes del sístole, y se debe a la CONTRACCIÓN DE LA AURÍCULA
DERECHA (al final del diástole, cuando se termina de vaciar al ventrículo derecho).
El colapso de la vena después de la onda "a", es el descenso "x" y se debe a
la RELAJACIÓN DE LA AURÍCULA. AX
• La onda "v" se debe al LLENE PASIVO DE LA AURÍCULA DERECHA debido al
retorno venoso normal, mientras la VÁLVULA TRICÚSPIDE PERMANECE CERRADA
durante el sístole. Por lo tanto, es una onda que ocurre al mismo tiempo del sístole
y que se vería sobre el vena yugular. El colapso que se observa después de la onda
"v", se denomina el descenso "y", que corresponde al PASO DE LA SANGRE
DE LA AURÍCULA AL VENTRÍCULO DURANTE EL DIÁSTOLE, después que se abre la
válvula tricúspide. VAY
• Con registros muy finos, se describe una pequeña muesca ubicada en el descenso
de la onda "a", que se ha llamado la onda "c", ATRIBUIDA AL CIERRE DE LA
VÁLVULA TRICÚSPIDE, después que se ha terminado de contraer la aurícula
derecha y está comenzando el sístole, pero no es posible de ver a simple vista.
FARMACOLOGIA
ANGINA DE PECHO
ANGINA ESTABLE
• Es la más frecuente, aparece con el esfuerzo y
remite espontáneamente con el reposo y/o la
medicación. Posee una duración de pocos
minutos y presenta un patrón regular, por lo que
el paciente puede ser capaz de identificarla e
incluso predecir su aparición. Su origen se halla
primordialmente en una arteriopatía
aterosclerótica que causa la progresiva reducción
de la luz vascular, de uno o varios vasos
coronarios, en porcentajes del orden del 70% o
superior.
ANGINA INESTABLE
• La angina inestable no se relaciona con un mayor trabajo cardíaco,
es decir no deriva de un mayor consumo miocárdico de oxígeno. Su
causa debe buscarse en una disminución aguda del flujo cardíaco
coronario, que puede deberse a la complicación de una placa
aterosclerótica coronaria por erosión, fisura o rotura y trombosis
sobreañadida que cause una interrupción súbita del flujo coronario
o por causas extrínsecas al árbol coronario que produzcan
inestabilización. Su sintomatología clínica es muy similar a la que
registra el infarto agudo de miocardio, sin embargo, en la angina
inestable no se produce necrosis miocárdica.
• La angina inestable incluye diversos tipos de anginas caracterizadas
por su evolución imprevisible, aunque no siempre fatal y que se
apartan claramente del patrón típico de angina estable: angina de
reciente comienzo, angina progresiva, angina de reposo, angina
prolongada, angina vasoespástica o de Prinzmetal y angina
postinfarto, todas ellas consideradas urgencias médicas.
IAM
IAM
IAM
SICA
CORONARIAS INVOLUCRADAS EN IAM
DERIVACION CON
ELEVACION DEL ST
TIPO DE INFARTO
ARTERIA
CORONARIA
RESPONSIBLE
COMPLICACIONES
V1-V2 Septal
Descendente
anterior (ramos
septales)
Bloqueos de rama
V3-V4 Pared anterior
Descendente
anterior (ramos
diagonales)
Disfuncion VI, IC,
Bloqueo de rama
V5-V6 Lateral alto Circunfleja
Hipotension (evitar
nitroglicerina o
morfina)
DII, DIII y aVF
Posteroinferior con
extension al
ventriculo derecho
Derecha (ramos
proximales)
Hipotension
V1-V4 Posterior
Circunfleja o
descendente
posterior
Disfuncion VI
PREGUNTAS
Cual es la triada clásica del infarto del ventrículo
derecho?
Hipotensión, campos pulmonares limpios y
elevación de la PVY
Cual es el tratamiento de la taquicardia auricular
multifocal?
Suele asociarse a enfermedad pulmonar grave,
mejora con la ventilación mecánica y la
oxigenación
TIPOS DE SHOCK
VALVULOPATIAS
RESUMEN SOPLOS
• ESTENOSIS MITRAL; SINTOMAS Y SOPLO
 DISNEA de esfuerzo, ORTOPNEA, EAP, disfonía por compresión del NLR.
Imagen de 4 arcos en Rx por congestion venocapilar.
 Soplo diastólico
• INSUFICIENCIA MITRAL; SINTOMAS Y SOPLO
 DISNEA de esfuerzo y fatiga, ORTOPNEA, DISNEA PAROXÍSTICA
NOCTURNA. Por prolapso valvular dolor torácico (signo de Barlow) dx dif con
SICA.
 Soplo sistólico
• ESTENOSIS AORTICA; SINTOMAS Y SOPLO
 Triada clásica, ANGINA DE PECHO - IC – SINCOPE. Palpitaciones, visión
borrosa. HV.
 Soplo sistólico
• INSUFICIENCIA AORTICA; SINTOMAS Y SOPLO
 Disnea que va de esfuerzo a DISNEA PAROXÍSTICA NOCTURNA, estertores.
 Soplo diastólico.
BRI
BRD
B AV I
• Prolongacion del intervalo PR en forma
continua
• >20” adultos
• >16” en infantes
B AV II/ M1
B AV II/M2
B AV C
RESUMEN BLOQUEO AV I-III
BLOQUEOS AV MANEJO
FA
QT LARGO
TRAZOS EKG PATOLOGICOS
SX WPW
SX BRUGADA
INSUFICIENCIA CARDIACA
INSUFICIENCIA CARDIACA
ENDOCARDITIS INFECCIOSA
Endocarditis infecciosa
A. Amoxicilina 2gr 1 hora antes
B. Ampicilina 2gr IM 30 minutos antes
(en intolerancia oral)
Alergia a penicilina
1) Claritromicina 500 mg 1 hora antes
2) Clindamicina 600 mg 1 hora antes
3) Cefalexina 2gr 1 hora antes
Profilaxis antibiótica en procedimientos dentales,
cavidad oral, respiratorio y esófago
Endocarditis infecciosa
• Ampicilina 2 gr IM o IV + Gentamicina
1,5mg/Kg/ 30 minutos antes, 6 h después
Ampicilina 1gr IM/IV o Amoxicilina oral 1gr
Alérgicos a penicilina
• Vancomicina 1gr IV en 1-2 h + Gentamicina
1,5 mg/Kg IV/IM terminando la perfusión 30
minutos después del procedimiento
Profilaxis antibiótica en procedimientos genitourinarios
y gastrointestinales
LESIONES DE JANEWAY
MANCHAS DE ROTH
NODULOS DE OSLER
TAMPONADE
JOVEN + DOLOR PRECORDIAL, FIEBRE Y ST (LESIÓN)
EXTENSA = PERICARDITIS AGUDA. LO EXACERBA LA RESPIRACIÓN.
ES FRECUENTE ANTECEDENTE DE IRAS VIRALES
DE NO CONOCERSE LA ETIOLOGÍA SE DAN AINES (AAS)
PERICARDITIS
• El supradesnivel del ST en la pericarditis se diferencia del infarto
agudo del miocardio porque en éste el supradesnivel es convexo y
más localizado, pueden coexistir ondas T negativas al mismo tiempo
del supradesnivel, presencia de ondas Q cuando es un infarto Q y el
EKG no normaliza antes de hacerse negativa la onda T.
• De la angina de Prinzmetal se distingue porque el supradesnivel es
transitorio y sólo durante el dolor; además, compromete
derivaciones más localizadas.
• La imagen de la repolarización precoz es parecida pero nunca hay
depresión del PR, no evoluciona y no hay síntomas.
MIOCARDIOPATIAS
Adulto + anasarca + complejo bajos =
pericarditis constrictiva
Rx de mujer , imagen de hiperflujo pulmonar, ECG con
BRD y eje a la derecha= Comunicación Interauricular
(OS)
CARDIOPATIAS CIANOGENAS Y
ACIANOGENAS
CIV
CIA
COARTACION AORTICA
ANEURISMAS
CARDIOPATIAS CONGENITAS
DISECCION AORTICA
EMBOLIA VS TROMBOSIS
OCLUSION ARTERIAL CRONICA
HAS
HAS
HAS
HAS
• Cual es el signo electrocardiografico mas común en tromboembolismo pulmonar?
• R = Taquicardia sinusal.
• -A 42-year-old man develops shortness of breath (SOB) and chest pain 7 days after an open cholecystectomy.
His blood pressure is 145/86 mm Hg, pulse is 120/min, respirations 24/min, and oxygen saturation of 97%.
Pulmonary embolism is clinically suspected. Which of the following is the most common ECG finding of
pulmonary embolism?
• (A) a deep S wave in lead I
• (B) depressed ST segments in leads I and II
• (C) prominent Q wave in lead I, and inversion of T wave in lead III
• (D) sinus tachycardia
• (E) clockwise rotation in the precordial leads
• Correcta D
• Cuales son los síntomas de estenosis mitral?
• R = DISNEA, ortopnea y disnea paroxística nocturna
• Como es el soplo en insuficiencia mitral?
• R = PanSISTOLICO en vértice e irradiado a axila.
• -A 25-year-old woman is found to have a midsystolic murmur on routine evaluation. The murmur does not
radiate but it does increase with standing. She otherwise feels well and the rest of the examination is normal.
• R = Mitral valve prolapsed.
• -A 65-year-old man with a previous history of an anterior MI comes for follow-up. On examination, he has a
systolic murmur heard best at the apex and radiating to the axilla. Transient external compression of both arms
with blood pressure cuffs 20 mm Hg over peak systolic pressure increases the murmur.
• R = Regurgitación mitral
• Que caracteriza al signo de MUSSET y en que patología se presenta?
• R = Hay movimientos de la cabeza en cada pulsación debido a INSUFICIENCIA DE LA VÁLVULA AORTICA.
• -A 75-year-old man is bought to the hospital because of a syncopal episode. There was no incontinence or post-event confusion. On examination, his blood
pressure is 140/80 mm Hg, pulse 72/min with no postural changes. His second heart sound is diminished and there is a systolic ejection murmur that radiates
to the carotids. With the Valsalva maneuver, the murmur decreases in length and intensity.
• R = Aortic stenosis.
• En que patología se escucha el soplo de AUSTIN FLINT?
• Insuficiencia aortica
• Que sucede cuando hay regurgitación aortica en forma aguda?
• Insuficiencia ventricular izquierda manifestada como EDEMA AGUDO PULMONAR.
• -A 32-year-old asymptomatic woman has a rapidly rising, forceful pulse that collapses quickly. Which of the following is the most likely diagnosis?
• (A) mitral stenosis
• (B) mitral regurgitation
• (C) aortic stenosis
• (D) aortic regurgitation
• Respuesta correcta D
• Que medicamento es de elección en el síndrome de Marfan con regurgitación aortica?
• R = BETABLOQUEADORES
• Que medicamentos pueden disminuir la dosis de homocisteina?
• R = Acido fólico, B6 y B12.
• Cuál es el fenómeno de hibernación y aturdimiento miocardico?
• R = Son las áreas del miocardio que se encuentran subperfundidas que se adaptan para ser viables con disfunción contráctil sostenida.
• Que es el síndrome de TIETZE?
• R = Inflamación de uniones condrocostales tumefactas, rojas que causa dolor retroesternal
• Que antianginosos han demostrado prolongar la vida?
• R = Los BETABLOQUEADORES a excepción de el pindolol que exacerba la angina
RESUMEN SOPLOS
• ESTENOSIS MITRAL; SINTOMAS Y SOPLO
 Disnea de esfuerzo, ortopnea, EAP, disfonía por compresión del NLR.
Imagen de 4 arcos en Rx por congestión venocapilar.
 Soplo diastólico
• INSUFICIENCIA MITRAL; SINTOMAS Y SOPLO
 Disnea de esfuerzo y fatiga, ortopnea, disnea paroxística nocturna. Por
prolapso valvular dolor torácico (signo de Barlow) dx dif con SICA.
 Soplo sistólico
• ESTENOSIS AORTICA; SINTOMAS Y SOPLO
 Triada clásica, angina de pecho - IC – Sincope. Palpitaciones, visión borrosa.
HV. Signo de Musset.
 Soplo sistólico
• INSUFICIENCIA AORTICA; SINTOMAS Y SOPLO
• Disnea que va de esfuerzo a disnea paroxística nocturna, estertores. PULSO
DE CORRIGAN , QUINQUE Y DUROZEIZ . Soplo de Austin Flint.
 Soplo diastólico.
• En cuanto tiempo tiene que estar el TTP cuando se da tratamiento anticoagulante?
• R = 50-70 segundos.
• En que pacientes se utiliza la profilaxis antiarrítmica?
• R = LATIDOS ECTÓPICOS FRECUENTES y taquicardia ventricular.
• Como se trata una fibrilación ventricular que no responde a una cardioversión?
• R = Amiodarona y RCP
• Como se dx infarto del ventrículo derecho?
• R = Con la ELEVACIÓN DEL ST DEL HEMITORAX DERECHO
• -A 69-year-old woman complains of some atypical chest pain 2 days prior to presentation. On examination, the
JVP is at 8 cm, positive Kussmaul’s sign, and normal heart sounds. The lungs are clear. The ECG is abnormal, and
the CXR shows a normal cardiac silhouette.
• Que tan frecuente se presenta un AUNERISMA VENTRICULAR IZQUIERDO POST IAM?
• R = 20% y se da el dx por medio de la ELEVACIÓN PERSISTENTE DEL ST DURANTE 4-8 SEMANAS.
• -Three months after an anterior MI, a 73-yearold man has a follow-up ECG. He is clinically feeling well with no
further angina symptoms. His ECG shows Q waves in the anterior leads with persistant ST-segment elevation.
The current ECG is most compatible with which of the following diagnosis?
• (A) ventricular aneurysm
• (B) hibernating myocardium
• (C) acute infarction
• (D) silent infarction
• (E) early repolarization
• Que caracteriza al SÍNDROME DE DRESSLER?
• R = SE PRESENTA POST IAM, debido a un FENÓMENO AUTO INMUNITARIO CON PERICARDITIS, FIEBRE,
LEUCOCITOSIS y ocasionalmente derrame pericardico o pleural.
• -A 67-year-old man presents with an anterior myocardial infarction (MI) and receives thrombolytic therapy.
Three days later, he develops chest pain that is exacerbated by lying down, and his physical findings are normal
except for a friction rub. His ECG shows evolving changes from the anterior infarction but new PR-segment
depression and 1-mm ST-segment elevation in all the limb leads. Which of the following is the most likely
diagnosis?
• (A) reinfarction
• (B) pulmonary embolus
• (C) viral infection
• (D) post-MI pericariditis
• (E) dissecting aneurysm
• Respuesta correcta D
• Cual es el manejo del infarto agudo del VD?
• R = CARGA LIQUIDA para mejorar el llenado ventricular derecho e INOTRÓPICOS.
• Cual es el mecanismo de acción de los antiarrítmicos clase Ia y cuales son?
• R = BLOQUEADORES DE LOS CANALES DE SODIO. Procainamida, quinidina y disopiramida.
• Cual es el mecanismo de acción de los antiarrítmicos clase Ib y cuales son?
• R = BLOQUEADORES DE LOS CANALES DE SODIO. Lidocaina y difenilhidantoina.
• Cual es el mecanismo de acción de los antiarrítmicos clase II y cuales son?
• R = BETABLOQUEADORES, retardando la conducción AV. Propanolol, metoprolol y esmolol.
• Cual es el mecanismo de acción de los antiarrítmicos clase III y cuales son?
• R = PROLONGAN EL POTENCIAL DE ACCIÓN. Amiodarona, sotalol, dofelitida.
• Cual es el mecanismo de acción de los antiarrítmicos clase IV y cuales son?
• R = BLOQUEADORES DE LOS CANALES LENTOS DE CALCIO. Verapamil, diltiazem, digoxina y adenosina.
• Con que fenómenos se relaciona la taquicardia paroxística supraventricular, la cual suele cursar asintomática.
• R = Intoxicación por digitalicos, bloqueos AV y fenómeno de reentrada.
• Que tratamiento se utiliza en taquicardia paroxística supraventricular en caso de estar contraindicados los
antiarrítmicos clase IV y que ha demostrado 100% de éxito?
• R = Cardioversion.
• Cual es el fármaco de elección en caso de prevención de ataques de taquicardia paroxística supraventricular?
• R = DIGOXINA
• Que caracteriza al síndrome de LOWN-GANONG-LEVINE?
• R = INTERVALO PR CORTO Y MORFOLOGÍA NORMAL DE QRS
• En que casos no necesita tratamiento las taquicardias supraventriculares causadas por vías accesorias o síndrome de
preexitacion?
• R = En casos de no presentar palpitaciones, mareos o sincope.
• Cual es el procedimiento de elección en pacientes con síndrome de preexitacion?
• R = ABLACIÓN POR RADIOFRECUENCIA
• Que agentes farmacológicos deben evitarse en el síndrome de preexitacion?
• R = Digoxina, betabloqueador y bloqueador de los canales de calcio.
• Cual es el tratamiento a largo plazo de el síndrome de preexitacion?
• R = Procainamida, verapamil y digoxina.
• Cuales son los fármacos de elección en extrasístoles ventriculares sintomáticas?
• R = BETABLOQUEADORES.
• Que medida se utiliza cuando la taquicardia ventricular es recurrente?
• R = Marcapasos
• - A22-year-old woman complains of palpitations and has a regular heartbeat at a rate of 170/min, with a blood pressure of 110/70 mm Hg. The rate
abruptly changes to 75/min after applying carotid sinus pressure. Which of the following is the most likely diagnosis?
• (A) sinus tachycardia
• (B) paroxysmal atrial fibrillation
• (C) paroxysmal atrial flutter
• (D) paroxysmal supraventricular tachycardia (PSVT)
• Respuesta correcta D
• Cual es el mecanismo de elección para la prevención de muerte súbita en un paciente con factores de riesgo?
• R = Desfibrilador implantado
• Que caracteriza al síndrome QT PROLONGADO?
• R = SINCOPE RECURRENTE, arritmias ventriculares y muerte súbita. Es una ANOMALÍA CONGÉNITA hereditaria que afecta los CANALES DE Na y K.
• Cual es la terapéutica mas eficaz en el síndrome de QT prolongado?
• R = BETABLOQUEADORES Y DESFIBRILADOR IMPLANTADO.
• Que hallazgo EKG caracteriza al bloqueo AV tipo I?
• R = PROLONGACIÓN DEL PR > .20” EN ADULTOS Y > .16”. LA ALTERACIÓN ES NODAL.
• Que hallazgo EKG caracteriza al bloqueo AV tipo II?
1) - 1er grado o fenómeno de WENCKEBACH, MOBITZ I: prolongación progresiva del PR hasta que 1 onda P no conduce.
2) - 2do grado o MOBITZ II: intervalo PR, sin embrago hay onda P que no conduce con un patrón 2:1 o 3:1
• En que casos se diagnostica el síndrome de SENO ENFERMO?
• R = PARO SINUSAL, bloqueo sinoauricular, BRADICARDIA SINUSAL PERSISTENTE o bradiarritmias.
• Cual es el manejo del síndrome de seno enfermo?
• R = TEOFILINA ORAL Y MARCAPASOS.
• Cual es la indicación terapéutica del Mobitz III?
• R = MARCAPASOS
• Cuales son los síntomas inadvertidos de ICC?
• R = Tos crónica, nicturia
• Que medicamento se utiliza en caso de intoxicación por digoxina?
• R = Anticuerpos FAP
• Cual es el cuadro clínico característico de alguien con ICC con edema agudo pulmonar?
• R = DISNEA, tos, ESPUTO ROSADO, diaforesis, cianosis.
• En que consiste el fenómeno de reentrada?
• R = Involucra un circuito que conduce de forma anterograda al ventrículo y de forma retrograda a la aurícula
utilizando vías accesorias.
• Que tratamiento se utiliza en ICC después de que los diuréticos y nitratos no mejoran la sintomatología?
• R = NISERITIDA, forma recombinante de péptido atrial natriuretico del cerebro humano.
• Que agente antihipertensivo se debe evitar en ICC?
• R = CALCIOANTAGONISTAS
• Como es típicamente la Rx de torax en edema agudo pulmonar con ICC?
• R = PATRÓN EN MARIPOSA en la distribución del edema alveolar.
• En el EAP la presión capilar en cuna se encuentra siempre elevada, a que cantidad aproximadamente?
• R = > 25 mm Hg
• La morfina es eficaz en el manejo de derrame pleural cardiogenico, debido a que mecanismo?
• R = AUMENTA LA CAPACITANCIA VENOSA, disminuye la presión auricular izquierda y disminuye la ansiedad.
• Cuál es la diferencia entre cardioversión y desfibrilación?
• R = La primera se encuentra sincronizada con complejo QRS y la 2da no hay sincronización.
• Cuáles son los agentes más comunes para miocarditis aguda?
• R = VIRAL (COXACKIE) bacteriano, riketsias, espiroquetas, micoticos y parasitarios.
• -A 23-year-old man develops sharp left-sided chest pain, fever, and a friction rub heard at the lower left sternal
border, unaffected by respiration. The pain is also aggevated by lying down and relieved by sitting up. He is
otherwise well with no other symptoms and the remaining physical examination is normal. Which of the
following is the most likely cause for his symptoms?
• (A) rheumatic fever
• (B) tuberculosis (TB)
• (C) herpes simplex virus
• (D) MI
• (E) coxsackievirus
• Respuesta correcta E
• La miocarditis Riketsial , por que enfermedad es originada?
• R = TIFO, fiebre de las montanas rocallosas y fiebre Q.
• Cual es el agente causal y cual es el periodo de incubación de la ENFERMEDAD DE CHAGAS?
• R = Es causado por el TROFOSZOITO TRIPANOSOMA CRUZI y su principal manifestación clínica aparece
en 10 años.
• Que afección sistémica es la regla para un individuo que padece Chagas?
• R = MEGAESOFAGO O MEGACOLON.
• Entre las afecciones parasitarias cual es la mas frecuente de afección cardiaca?
• R = La triquinosis.
• Cual es la fisiología de la maniobra de valsalva?
• R = Se produce un aumento del tono parasimpático por liberación de acetilcolina, lo que genera un
retardo en la conducción AV causando un bloqueo AV transitorio.
• Que dato puede ser único o inicial en el EKG en MIOCARDITIS INFECCIOSA?
• R = Ectopia ventricular
• Que medicamento esta indicado en el espasmo coronario inducido por la COCAÍNA?
• R = BETABLOQUEADORES Y CALCIOANTAGONISTAS
• Cuales son los síntomas de MIOCARDIOPATIA DILATADA?
• R = Disnea, dolor torácico .
• Cual es el manejo del paciente con CARDIOPATÍA HIPERTRÓFICA?
• R = BETABLOQUEADORES, CALCIOANTAGONISTAS Y MARCAPASOS.
• -A 45-year-old woman has developed increasing SOB on exertion and fatigue. She has a loud systolic ejection murmur heard best at the left
sternal border, and the murmur increases with standing. A double apical impulse is also felt.
• R = Cardiomiopatia hipertrofica.
• 66. Cuales son las causas principales de MIOCARDIOPATIA RESTRICTIVA?
• R = AMILOIDOSIS, RADIACIÓN, fibrosis por cirugía, sarcoidiosis, hemocromatosis y síndrome carcinoide.
• -57 A 63-year-old man develops edema, and dyspnea on exertion. He has no prior cardiac or renal conditions, and his examination is
significant for macroglossia, elevated jugular venous pressure (JVP), hepatomegaly, and 3+ pedal edema. His investigations reveal 3.5 g/d of
protein in the urine, anemia, normal fasting glucose, and serum immunoelectrophoresis is positive for a monoclonal immunoglobulin.
Which of the following is the most characteristic neurologic finding associated with this condition?
• -Peripheral motor and sensory neuropathy: In addition to peripheral motor and sensory neuropathy, cardiac involvement, tongue
enlargement, gastrointestinal (GI) manifestations, and carpal tunnel syndrome are also seen in amyloidosis. The specific diagnosis requires
tissue biopsy with presence of amyloid with specific stains. In primary amyloidosis and myeloma, the amyloid protein is of the ALtype. In
reactive amyloidosis, the protein is of the amyloid Aprotein (AA) type.
• Cuales son los criterios mayores de Jones de CARDIOPATÍA REUMÁTICA?
1) CARDITIS: pericarditis, ICC, cardiomegalia y soplos.
2) ERITEMA MARGINADO
3) NÓDULOS SUBCUTÁNEOS: los nódulos subcutáneos SON INDOLOROS y menores de 2 cm
4) COREA DE SHYDENHAM: Movimientos coreoatetoides
5) ARTRITIS: dura de 1-5 semanas y responde a AINES
• Cuales son los criterios menores de cardiopatía reumática?
• R = Fiebre, VSG elevada, prolongación del PR.
• Cual es el manejo de cardiopatía reumática?
• R = PENICILINA BENZATINICA 1.2 millones DU o PEN-PRO 600, 000 x 10 dias.
• Como se previene la fiebre reumática?
• R = PEN-BEN 1.2 cada 4 semanas
• En los pacientes con Wolf-Parkinson-Withe. Que medicamentos están contraindicados?
• R = ADENOSINA, DIGOXINA, CALCIOANTAGONISTAS Y BETABLOQUEADORES YA QUE BLOQUEAN EL NODO AV.
• En los pacientes con Wolf-Parkinson-Withe. Cual es el manejo indicado?
• R = Cardioversión eléctrica y ablación por radiofrecuencia de vías accesorias.
• Cuales son los síndromes de preexitacion?
• R = Síndrome de WPW y Lown-Ganong-Levine
• Cual es el microorganismo causal de la ENFERMEDAD DE LYME?
• R = Borrelia Burgdorjeri
• Que características tiene la pericarditis secundaria a síndrome urémico?
• R = Pericardio afelpado, derrame hemorrágico y exudativo.
• Que caracteriza al síndrome de Dressler?
• R = Pericarditis post IAM o cirugía del corazón abierto y de etiología autoinmune.
• Como se define la hipertensión pulmonar primaria?
• R = Como aumento de la resistencia vascular pulmonar sin enfermedad subyacente.
• -Auscultation of the heart of a 17-year-old boy reveals an increased intensity of the pulmonary component of the
second heart sound. He complains of dyspnea on exertion but no other cardiac or pulmonary symptoms. Which of the
explanations is the most likely cause of his dyspnea?
• (A) pulmonary stenosis
• (B) aortic stenosis
• (C) MI
• (D) pulmonary hypertension
• (E) systemic hypertension
• Respuesta correcta D
• Cual es el manejo de hipertensión pulmonar primaria?
• R = CALCIOANTAGONISTAS, anticoagulantes, prostaciclina (potente vasodilatador pulmonar)
• Cual es el tratamiento de elección de la hipertensión primaria?
• R = Endotelina
• A que hace alusión el termino Cor-pulmonale?
• R = Hipertrofia ventricular derecha e insuficiencia por enfermedad pulmonar, mas frecuente en EPOC
• Cual es el dato de laboratorio mas frecuente en el Cor-pulmonale?
• R = Policitemia
• Cuales son los tumores que afectan al corazón?
• R = Carcinoma broncogenico, carcinoma mamario, melanoma maligno, linfomas, carcinoma de células renales y sarcoma de kaposi.
• Cuales son los tumores primarios del corazón mas frecuentes?
• R = MIXOMA AURICULAR el cual es un tumor benigno que puede embolizar sistémicamente y se ubica comúnmente en la aurícula derecha.
• -A 47-year-old woman has new-onset transient right arm weakness and word finding difficulty symptoms lasting 3 hours. She is also
experiencing exertional dyspnea, and had a syncopal event 1 month ago. Her echocardiogram reveals a cardiac tumor in the left atrium, it
is pendunculated and attached to the endocardium. Which of the following is the most likely cause of this lesion?
• (A) myxoma
• (B) sarcoma
• (C) rhabdomyoma
• (D) fibroma
• (E) lipoma
• Respuesta correcta A
• En que consiste la ENFERMEDAD DE BURGUER?
• R = Es un PROCESO INFLAMATORIO Y TROMBOTICO de las arterias y venas periféricas PRODUCIENDO CLAUDICACIÓN, DOLOR Y NECROSIS.
• Cuales son los datos clínicos mas frecuentes de la enfermedad de Takayasu?
• R = Soplos vasculares, PULSOS PERIFÉRICOS DISMINUIDOS y ASIMETRÍA DE LA PA DE LAS EXTREMIDADES.
• Cual es el tratamiento de la enfermedad de Takayasu?
• R = ESTEROIDES para la inflamación y después COLOCAR STENT O PUENTES.
• Cual es el tratamiento para pacientes con displasia fibromuscular de la intima?
• R = ASA
• Que fenómenos deben descartarse en una persona que haya cursado con enfermedad de Raynaud?
• R = 80% ESCLERODERMIA, MIOSITIS, LES, AR.
• Cual es el síndrome doloroso regional o complejo tipo 1 distrofia simpática refleja?
• R = DOLOR ARDOROSO O QUEMANTE de mas duración de lo esperado POR TRAUMATISMO EN EXTREMIDAD
secundario a aplastamiento o quemadura.
• Cuales son los datos clínicos de la distrofia simpática refleja?
• R = DOLOR LOCAL, hiperestesia, calor, RIGIDEZ MUSCULAR, RIGIDEZ ARTICULAR, con consiguiente DESUSO Y
OSTEOPENIA RADIOLÓGICA
• Que tratamientos se utilizan en distrofia simpática refleja?
• R = OPIOIDES Y GABAPENTINA
• Cual es el tipo mas común de síndrome de salida torácica?
• R = NEUROGENICO 90%, venoso y arterial.
• En el síndrome de salida torácica , que es el síndrome de PAGET-SCHROETTER O TROMBOSIS DE ESFUERZO?
• R = Se presenta como EDEMA UNILATERAL AGUDO DEL BRAZO, pesantez de axila, CIANOSIS DE LA MANO E
INGURGITACIÓN DE LAS VENAS DEL HOMBRO Y TÓRAX.
• Qué medidas se toman en un sujeto con choque que no responde a estímulos?
• R = DEXTROSA AL 50%, naloxona 2 ml iv. Manteniendo la diuresis horaria mayor de .5
• Que te indica la PVC?
• R = MENOR DE 5 mm hg indica HIPOVOLEMIA, MAYOR DE 18 SOBRECARGA VOLUMEN.
• Cuáles son los datos clínicos de taponamiento cardiaco?
• R = Taquicardia, HIPOTENSIÓN, pulso paradójico, AUMENTO DE LA PVY. Excepto en pacientes urémicos o con
hipotiroidismo.
• -A 25-year-old man complains of left precordial chest pain that radiates to the left shoulder but not down the
left arm. The pain is accentuated by inspiration and relieved by sitting up. The pain is accompanied by fever and
chills. His blood pressure is 105/75 mm Hg, pulse 110/min and regular, and temperature 37.5°C. Aside from the
tachycardia, there are no abnormal physical findings in the heart or lungs. The ECG shows STsegment elevation
in all leads except aVR and VI. On the third hospital day, the patient’s blood pressure falls, JVP rises, and he
goes into CHF. Which of the following is the most likely diagnosis?
• (A) a second pulmonary embolus
• (B) extension of a myocardial infarct
• (C) cardiac tamponade
• (D) secondary bacterial infection
• (E) rupture of a chordae tendineae
• Respuesta correcta C
• -A 56-year-old man presents with SOB, fatigue, and edema. He has also noticed weight gain, abdominal
discomfort, and distension. He has a prior history of lung cancer treated with radiotherapy to the chest. There
is no history of liver or cardiac disease in the past. On examination, he has an elvated JVP, prominent y descent
of neck veins, and positive Kussmaul’s sign. The heart sounds are normal. The CXR shows a normal cardiac
silhouette and the ECG has low voltages.
• -A 55-year-old woman with metastatic lung cancer presents with dyspnea and pedal edema. On examination,
the JVP is at 10 cm, with a negative Kussmaul’s sign. The heart sounds are diminished and the lungs have
bibasilar crackles. The ECG shows QRS complexes of variable height
• Como afecta el cliostazol en la enfermedad arterial periférica?
• R = Es un INHIBIDOR DE LA FOSFODIESTERASA 3 que disminuye la agregación plaquetaria, activa la lipoproteína
lipasa y causa vasodilatación.
• Cuál es el objetivo del tratamiento en un paciente con choque?
• R = PVC 8-12, PA ½ 65-90, índice cardiaco 2-4 lts/min, y O2 venoso > 70%.
• Cuáles son los efectos de la dopamina?
• R = 2-3 ug/kg/min estimula RECEPTORES DOPA y B AGONISTAS con AUMENTO DEL FG, FC Y IONOTROPISMO. >5 ug/kg/min
estimula receptores ALFA adrenergicos produciendo VASOCONSTRICCIÓN PERIFÉRICA.
• Cual es el medicamento de elección para choque cardiogenico?
• R = DOBUTAMINA con aumento del ionotropismo y disminución de la poscarga
• Que efecto tiene la desmopresina utilizada en choque distributivo?
1) Vasoconstricción periférica, disminución de la FC, vasodilatación coronaria, pulmonar y cerebral.
2) La vasoconstricción es mediada por oxido nítrico.
• Cual es la causa de la miocardiopatía hipertrófica?
• R = DEFECTO CONGÉNITO en los sarcomeros.
• Cual es la causa de miocardiopatía dilatada?
• R = Isquemia, cardiotoxicos. EMBARAZO.
• Cual es la causa de la miocardiopatía restrictiva?
• R = Primaria: fiebre endomiocardica, endocarditis de coffler o ideopatica, Secundaria. RADIACION.
• -A 55-year-old woman is recently diagnosed with amyloidosis. She is now noticing increasing SOB, fatigue, and edema. On
examination, the JVP is at 10 cm with a negative Kussmaul’s sign but prominent x and y descent. The blood pressure is 90/70
mm Hg, no pulsus paradoxus, pulse 100/min with low volume, and normal heart sounds.
• -A 60-year-old man presents with SOB, increasing abdominal distention, and lower leg edema. He has no prior history of
cardiac, renal, or liver disease. On examination, the JVP is at 8 cm with a negative Kussmaul’s sign but prominent x and y
descent. The blood pressure is 95/75 mm Hg, no pulsus paradoxus, pulse 100/min with low volume, and normal heart sounds.
There is shifting dullness of the abdomen and pedal edema. His blood glucose and hemoglobin A1C are elevated.
• Que medicamentos están contraindicados en miocardiopatía restrictiva?
• Digitalicos y agonistas B adrenérgicos.
• Cuales son los criterios para ENDOCARDITIS BACTERIANA DE DUKE.
• CRITERIOS MAYORES:
• 1. Hemocultivos positivos para EI
1.1. Microorganismos típicos de EI en dos hemocultivos separados
1.1.1 ESTREPTOCOCO VIRIDANS
S. BOVIS
HACEK
1.1.2. S. Aureus o Enterococus adquiridos en la comunidad en ausencia de foco primario
1.2. Hemocultivos persistentes positivos
1.2.1. Hemocultivos extraidos con más de 12 horas de separación
1.2.2. La totalidad de tres, o la mayoría de cuatro o más hemocultivos separados
siempre que entre el primero y el último haya al menos una hora
• 2. Evidencia de afectación miocárdica
2.1. Ecocardiograma positivo
2.1.1. VEGETACIÓN EN VÁLVULA O ESTRUCTURAS ADYACENTES o en el choque del jet, o
sobre dispositivos protésicos en ausencia de otra explicación anatómica
2.1.2. Absceso
2.1.3. Nueva dehiscencia parcial de una válvula protésica
2.2. Nueva regurgitación valvular (incremento o cambio en un soplo preexistente no
es suficiente)
• CRITERIOS MENORES
1. Predisposición. Una cardiopatía predisponente o ser ADVP.
2. FIEBRE > 38ºC
3. Fenómenos vasculares: émbolos en arterias mayores, infartos pulmonares, sépticos, aneurismas micóticos,
hemorragia intracraneal, hemorragia conjuntival y LESIONES DE JANEWAY
4. Fenómenos inmunológicos (glomerulonefritis, NÓDULOS DE OSLER, MANCHAS DE ROTH Y factor
reumatoide).
5. Ecocardiograma: sugestivos de eI sin alcanzar los criterios mayores antes mencionados.
6. Evidencia microbiológica (hemocultivos positivos que no cumplen los criterios mayores) o evidencia
serológica de infección activa con un microorganismo que produce EI.
• -A 28-year-old man develops viridans group streptococci septicemia. Which of the following cardiac lesions has
the highest risk of developing endocarditis?
• (A) ventricular septal defect
• (B) atrial septal defect, secundum type
• (C) mitral valve prolapse with regurgitation
• (D) pure mitral stenosis
• (E) asymmetric septal hypertrophy
• Respuesta correcta A
• En que consiste la PERICARDITIS CONTSTRICTIVA?
• R = Constrictive pericarditis is characterized by a prominent y descent of the neck veins and LOW VOLTAGE ON
ECG. THE PRESENCE OF A POSITIVE KUSSMAUL’S SIGN HELPS DIFFERENTIATE THE SYNDROME FROM COR
PULMONALE AND RESTRICTIVE CARDIOMYOPATHIES. Apericardial knock is characteristic of constrictive
pericarditis. It is in fact an early S3, occurring 0.06–0.12 seconds after aortic closure. S1 and S2 are frequently
distant.
• -A 56-year-old man presents with SOB, fatigue, and edema. He has also noticed weight gain, abdominal
discomfort, and distension. He has a prior history of lung cancer treated with radiotherapy to the chest. There
is no history of liver or cardiac disease in the past. On examination, he has an elvated JVP, prominent y descent
of neck veins, and positive Kussmaul’s sign. The heart sounds are normal. The CXR shows a normal cardiac
silhouette and the ECG has low voltages.
• -A 64-year-old presents with dyspnea and edema. He had previous coronary
bypass surgery 5 years ago, which was uncomplicated. Since then he has
had no further chest pain. On examination, his JVP is at 8 cm, with
prominent Kussmaul’s sign. The heart sounds are easily heard but there is
an early diastolic filling sound (pericardial knock).
• Cuál es el diagnostico diferencial entre miocardiopatia constrictiva y
pericarditis restrictiva?
1) PERICARDITIS RESTRICTIVA: Puede ser causada por tuberculosis, Rx del
tórax y cirugía cardiaca.
2) PERICARDITIS CONSTRICTIVA: Causada por un tamponade.
3) MIOCARDIOPATIA CONSTRICTIVA: Lo causa la amiloidosis,
hemocromatosis, sarcoidiosis, esclerodermia, carcinoide.
REUMATOLOGIA
ENARM
ARTRITIS CRONICA JUVENIL
ESPONDILITIS ANQUILOSANTE
• Que espondiloartropatias se asocian al HLA-B27?
 R = ESPONDILITIS ALQUILOSANTE, ARTRITIS REACTIVA, ESPONDILOARTROSIS
PSORIASICA, uveítis anterior aguda.
• Cual es el síntoma inicial de la espondilitis alquilosante (EA) y que hallazgos son frecuentes en la ex fis?
 R = DOLOR en regiones SACROILIACAS Y LUMBARES. En la ex fis HAY DISMINUCIÓN DE LA
MOVILIDAD DE LA COLUMNA LUMBAR, dolor al presionar las regiones sacroiliacas y disminución de la
expansión del tórax.
• Cual es la manifestación extrarticular mas frecuente de la EA?
 R = UVEITIS ANTERIOR, 2/3 de px tiene CAMBIOS EN LA MUCOSA Y SUBMUCOSA DEL COLON E ÍLEON
TERMINAL
• Cuales son los hallazgos más comunes en imagen obtenidas en la espondilitis anquilosante?
 R = Erosiones y esclerosis sacroiliacas, IMAGEN EN COLUMNA DE BAMBU
• Cual es el manejo de la EA?
 R = AINES, COX-2, Infliximab, Etanercept mas terapia física. Indometacina, INFLIXIMAB que disminuye el FNTa.
ENFERMEDAD DE REITER
• Cuales son los datos clinicos más comunes de ARTRITIS REACTIVA
o enfermedad de Reiter y con que HLA se relaciona?
 R = Ligado al HLA B27, oligoartritis, CONJUNTIVITIS, URETRITIS,
CERVICITIS, ULCERAS ORALES x un CUADRO INFECCIOSO PREVIO.
• Que situación precede a una manifestación de ARTRITIS
REACTIVA o síndrome de Reiter y cual es el patógeno > común?
 R = Alguna INFECCIÓN por SALMONELLA, SHIGELLA, YERSINIA,
CLAMYDIA TRACHOMATIS +++o
CAMPYLOBACTER que PRECEDE a la MANIFESTACIÓN ARTICULAR
3-4 semanas después.
• Cual es el estudio de laboratorio o gabinete de elección del
síndrome de Reiter?
 R = CULTIVO CON ANTIBIOGRAMA, se puede encontrar también Ac
anti bacteriano en el suero o liquido sinovial o DNA bacteriano.
ENFERMEDAD DE STILL
• Que caracteriza clínicamente a la enfermedad de Still y que la precede?
 R = VARIANTE DE LA ARTRITIS REUMATOIDE en la cual hay FIEBRE DE
MAS DE 40 y con precipitación posterior varios grados debajo,
ODINOFAGIA Y POLIADENOPATIAS. LO PRECEDE
FARINGITIS.
• Cual es una manifestación clínica característica de la enfermedad de Still?
 R = EXANTEMA NO PRURIGINOSO, maculopapular de COLOR SALMÓN
en TÓRAX Y ABDOMEN. (EL CUAL NO HAY EN REITER)
• Cuales son los laboratorios distintivos de la enfermedad de Still, que
inmunoglobulina se eleva y como afecta al FR?
 R = LEUCOCITOSIS > 40, VSG AUMENTADA, hipergamaglobulinemia IgG y anemia.
NO se detectan AUTOANTICUERPOS NI FR.
• Cual es el manejo de la enfermedad de Still?
 R = AINES
ARTRITIS SORIASICA
• Que es la artritis soriasica?
 R = Ligado al HLA B27. Se define como una ARTRITIS usualmente
SERONEGATIVA que se ASOCIA A SORIASIS.
• Cual es el cuadro clínico de la artritis soriasica?
 R = PRECEDE PSORIASIS A ARTRITIS, ARTRITIS ASIMÉTRICA con
DEDOS EN APARIENCIA DE SALCHICHA (DACTILITIS) de los dedos de manos
y pies.
• Como diagnosticas artritis soriasica?
 R = HC, y RX con evidencia de SACROILEITIS ASIMÉTRICA, alteraciones de la
conformación de hueso nuevo con IMAGEN DE PUNTA DE LÁPIZ EN UNA
COPA.
• Cual es el manejo de la artritis soriasica?
 R = AINES E INFLIXIMAB. Tratamiento de soriasis.
RESUMEN ESPONDILOARTROPATIAS
SERONEGATIVAS
LUPUS ERITEMATOSO SISTEMICO
• Que HLA tienen relación con el LES?
1) HLA B8, HLA DR2 y DR3.
2) Los alelos de HLA están en el brazo corto del CROMOSOMA 6.
• Que fármacos inducen la producción de anticuerpos antinucleares y LES?
 R = HIDRALAZINA, procainamida, clorpromazina, ISONIAZIDA y diversos anticonvulsivantes.
• Cual es el cc del LES en cada aparato – sistema y con que padecimientos se relaciona?
 1. Se relaciona con: PTI, ANEMIA HEMOLITICA Y TIROIDITIS.
 2. Constitucionales: la FATIGA es lo mas común
 3. Mucocutaneas: 80%, LESIONES DISCOIDES observándose en cara, pabellones auriculares y el
AREA DE V DEL ESCOTE. Eritema malar o en ALAS DE MARIPOSA QUE ES FOTOSENSIBLE. EL
neonatal QUE SE PRESENTA EN LOS HIJOS DE PX CON LES POR EL PASO DE IgG
CARACTERIZÁNDOSE POR LESIONES CUTÁNEAS Y BLOQUEO CARDIACO CONGÉNITO EN
LOS PRIMEROS 6 MESES DE EDAD. ALOPECIA.
 4. Musculoesqueletico: ARTRALGIAS con artritis. Osteonecrosis en la cabeza femoral, humeral,
cóndilos femorales, carpo, tarso manifestándose clínicamente con dolor que mejora con el reposo.
 5. Renal: nefritis lupica siendo la mas común la TIPO IV ¨GLOMERULONEFRITIS PROLIFERATIVA
DIFUSA¨.
 6. SNC: Meningitis aséptica, EVC, síndromes desmielinizantes y cefaleas.
 7. Cardiopulmonar: En corazón con VEGETACIONES ENDOCARDICAS de LIBMAN-SACKS, en
pleura lo mas común es pleuritis con o sin derrame pleural con niveles altos de proteínas DHL,
complejos inmunes, ANA y células LE. LA VALVULA MAS AFECTADA ES LA MITRAL.
 8. Gastrointestinal: Se puede presentar con SEROSITIS, disfagia y dispepsia muy a menudo.
 9. Hematológico: PANCITOPENIA Y >TTP. Anemia por deficiencia de hierro MICROCITICA-
HIPOCROMICA.
ESCLERODERMIA
SX SJOGREN
POLIARTERITIS NODOSA
• Cuales son los signos y síntomas MAS COMUNES de poliarteritis nodosa?
 R = DOLOR EN EXTREMIDAD POR ARTRALGIA, MIALGIA QUE AFECTA
PANTORRILLAS, puede afectar riñón causando HAS, IRA O HEMORRAGIA
POR MICROANEURISMAS. MONONEURITIS MÚLTIPLE
(PIE CAÍDO), corazón con IAM. Livides reticular, nódulos subcutáneos, y ulceras en
piel. Son necesarios 3 de 10 criterios para su diagnostico.
• Cuales son las pruebas de laboratorio en la poliarteritis nodosa y con que otro
virus se relaciona habitualmente?
 R = DESCARTAR VHBQUE SE ASOCIA EN 20-30%. ANCA (-) Y ANA (-) .Anemia,
leucocitosis y trombocitosis. Se confirma diagnostico con toma de BIOPSIA DE LA
LESIÓN.
• Que datos se obtienen al tomar la biopsia en la poliarteritis nodosa?
 R = BIOPSIA DE ARTERIA de mediano o pequeño calibre con PRESENCIA DE
INFILTRADO GRANULOCITICO, PMN Y CÉLULAS MONONUCLEARES EN LA PARED DEL
VASO.
• Cual es el manejo de la poliarteritis nodosa?
 R = ESTEROIDES, pero SI SE ASOCIA CON VHB SE AGREGAN RETROVIRALES.
ARTERITIS DE LA TEMPORAL
VASCULITIS
ARTRITIS SEPTICA PIOGENA
• Cuales son los factores de riesgo para la artritis séptica piógena (gonocócica o no)?
 R = Abuso de drogas IV, jóvenes sexualmente activos, enfermedades concomitante como DM, AR o lupus,
antecedentes de cirugía o prótesis.
• Cual es el cc de la artritis séptica piógena gonocócica o no?
 R = Es una entidad de inicio agudo y progresivo caracterizado por dolor e inflamación local (salvo en el primer
año de vida donde es similar a la sepsis). El dolor es moderado a intenso con limitación funcional, aumento de
la temperatura local y rubor es monoarticular, cuando es poliarticular es en paciente inmunocomprometido
siendo la rodilla la mas afectada.
• Cuales son los hallazgos radiológicos en la artritis séptica piógena?
 R = Edema de tejidos blandos y derrame articular. EN AFECCIÓN POR ANAEROBIOS SE VE GAS EN TEJIDOS
BLANDOS. El gamagrama óseo con tecnecio (valora flujo) o con galio (valora actividad inflamatoria) pueden
utilizarse de manera temprana.
• Cual es el manejo de la artritis séptica piógena?
1) Consiste en la administración de antibióticos y drenaje de la articulación afectada. En caso de ARTRITIS
PIÓGENA debe administrarse de 3 A 4 SEMANAS y en caso de ARTRITIS GONOCÓCICA DE 7 A 14 DÍAS.
2) En menores de 2M: DICLOXACILINA mas CEFOTAXIMA
3) En LACTANTES: DICLOXACILINA mas CEFOTAXIMA
4) En niños MAYORES DE 3ª: DICLOXACILINA
5) Artritis séptica GONOCÓCICA: 1g por via IM o IV cada 24 hrs.
6) Artritis séptica en DROGADICTOS por via IV: DICLOXACILINA mas GENTAMICINA
7) Artritis séptica en INMUNOCOMPROMETIDOS: CEFTAZIDIMA mas AMIKACINA
8) Artritis séptica POSTOPERATORIA: VANCOMICINA mas AMIKACINA
9) Artitis séptica en FRACTURA ABIERTA: AMOXICILINA con ACIDO CLAVULANICA
ARTRITIS SEPTICA PIOGENA
RESUMEN DE LAS ENFERMEDADES
METABOLICAS OSEAS
CPK
• Las siguientes patologías cursan con aumento
de la CPK
1) HIPOTIROIDISMO
2) RAZA NEGRA
3) POLIMIOSITIS Y
4) DERMATOMIOSITIS
GASTROENTEROLOGIA
ENARM
ACALASIA
• Que es la acalasia?
 R = Es un trastorno por motilidad inadecuada que se caracteriza por una RELAJACIÓN INCOMPLETA y
AUMENTO DE LA PRESIÓN BASAL DEL EEI, además de presentar APERISTALSIS DEL CUERPO.
• Que causa la acalasia?
 R = Es IDIOPÁTICO, aunque una de las hipótesis indica que es de origen central con DESTRUCCIÓN
PARCIAL DEL NÚCLEO DORSAL DEL VAGO o sus fibras nerviosas a nivel del esófago con la siguiente
alteración de la función.
• Cuales son los síntomas más comunes de la acalasia?
 R = DISFAGIA EN EL 90% DE LOS CASOS, INICIALMENTE A SÓLIDOS Y DESPUÉS A LÍQUIDOS. El segundo
síntoma mas frecuente es la REGURGITACIÓN EN 75% mas acentuada en decúbito, EL DOLOR 20-
30% ES MENOS COMÚN, DE LOCALIZACIÓN EPIGÁSTRICA, RETROESTERNAL, E IRRADIADO HACIA
CUELLO, DORSO Y EXTREMIDADES SUPERIORES.
• Cuales son los estudios de gabinete necesarios para el diagnostico de acalasia?
 R = La MANOMETRIA ES DE ELECCIÓN la cual nos muestra una PRESIÓN DEL EEI AUMENTADA (>69
MM/HG), SIN PRESENCIA DE ONDAS PERISTÁLTICAS A LA DEGLUCIÓN. La ESOFAGOGRAFIA CON BARIO
MUESTRA ALTERACIONES RELACIONADAS CON LAS ONDAS PERISTÁLTICAS Y LA AUSENCIA DE
RELAJACIÓN DEL EEI DANDO UNA IMAGEN EN PICO DE PÁJARO O EN PUNTA DE LÁPIZ característica.
• Cual es el manejo de la acalasia?
1) MEDICO: A base de DILATACIONES HIDROSTÁTICAS O NEUMÁTICA, además de LA INYECCIÓN CON
TOXINA BOTULINICA.
2) Quirúrgico: MIOTOMIA DE HELLER con una tasa de curación del 95%.
ESPASMO ESOFAGICO DIFUSO
• Que es el espasmo esofágico difuso?
 R = Se trata de un trastorno motor esofágico que afecta fundamentalmente a la musculatura lisa y que se caracteriza
por LA APARICIÓN DE ACTIVIDAD CONTRÁCTIL NO PROPULSIVA que alterna con episodios de peristalsis normal.
• Cual es el cuadro clínico del espasmo esofágico difuso?
 R = DOLOR TORAXICO CON LOCALIZACIÓN fundamentalmente RETROESTERNAL, aunque a veces SE IRRADIA A
CUELLO Y A LOS HOMBROS SIMULANDO UNA ENFERMEDAD CARDIACA aumentando cuando están bajo estrés
sicológico o emocional. La DISFAGIA es tan frecuente o mas que el dolor torácico. También es intermitente SE
DESENCADENA CON FRECUENCIA CON LA TOMA DE SÓLIDOS O BEBIDAS FRÍAS.
• Cuales son las alteraciones histológicas que esperas encontrar en el espasmo esofágico difuso?
 R = HIPERPLASIA DE LAS CÉLULAS musculares y ocasionalmente mínimas ALTERACIONES a nivel de las fibras nerviosas
terminales del PLEXO DE AUERBACH, no existiendo alteraciones significativas en las células ganglionares y nervios o en
todo caso estas son mínimas.
• Cuales son los hallazgos al gabinete en el espasmo esofágico difuso?
 R = ESOFAGOGRAMA: Que nos muestra hallazgos característicos con APARICIÓN DE FRECUENTES ONDAS NO
PROPULSIVAS en el musculo liso esofágico QUE INDENTAN LA COLUMNA DE BARIO y retrasan su evacuación, lo
cual se describe como ESÓFAGO EN SACACORCHOS. MANOMETRIA: Alteraciones segmentarias, apareciendo
generalmente en los DOS TERCIOS INFERIORES DEL CUERPO ESOFÁGICO. PHmetria obligatoria para descartar ERGE
como causa del espasmo.
• Cual es el manejo MEDICO Y QUIRURGICO del espasmo esofágico difuso?
• MEDICO: NIFEDIPINO, ANTIDEPRESIVOS Y ANSIOLÍTICOS.
• QUIRURGICO: La ESOFAGOMIOTOMIA AMPLIADA hasta el cayado aórtico puede ser una alternativa muy acertada EN
CASOS GRAVES REFRACTARIOS a cualquier otro tratamiento.
PERISTALSIS ESOFÁGICA SINTOMÁTICA O ESÓFAGO EN CASCANUECES
• Que es la peristalsis esofágica sintomática o esófago en cascanueces y con que se relaciona?
 R = Trastorno de la motilidad esofágica que se caracteriza por la PRESENCIA DE CONTRACCIONES DE
GRAN AMPLITUD, SIN ALTERACIONES EN EL PERISTALTISMO del cuerpo esofágico y relajación del
esfínter esofágico inferior. Se RELACIONA CON ERGE y a la histología no presenta alteraciones.
• Cual es el cuadro clínico de la peristalsis esofágica sintomática o esófago en cascanueces?
 R = DOLOR TORÁCICO RECURRENTE y con menor frecuencia disfagia.
• Cuales son los hallazgos de gabinete en la peristalsis esofágica sintomática o esófago en cascanueces
en la manometria o en el esofagograma?
 R = El diagnostico se establece por la MANOMETRIA y se caracteriza por la aparición de ONDAS DE
GRAN AMPLITUD (180 MM/HG), QUE OCASIONALMENTE TAMBIÉN SON DE LARGA DURACIÓN
(6SEG), siendo condición indispensable que EL PERISTALTISMO ESOFÁGICO SEA NORMAL.
Siempre se debe descartar ERGE realizando una PHmetria. NO HALLAZGOS CON ESOFAGOGRAMA.
• Cual es el manejo de la peristalsis esofágica sintomática o esófago en cascanueces?
1) MEDICO: Si existe ERGE se recomiendo uso de OMEPRAZOL y evitar MIORRELAJANTES (nifedipino,
diltiazem o nitratos) POR QUE EMPEORARÍAN EL ERGE. BENZODIAZEPINAS Y SOBRE TODO LOS
ANTIDEPRESIVOS TRICICLICOS parecen útiles.
2) QUIRURGICO: ESOFAGOMIOTOMIA AMPLIADA HASTA EL CAYADO AÓRTICO puede ser una alternativa
en pacientes intratables. La inyección de toxina botulinica tiene resultados muy pobres.
EEI HIPERTENSO
• Como se define el EEI hipertenso?
 R = Como la existencia DE UN TONO O PRESIÓN media máxima basal DEL
EEI IGUAL O SUPERIOR A DOS DEVIACIONES ESTÁNDAR, mas de 40-45
mm/Hg, siendo además condicionantes indispensables que existan
RELAJACIONES COMPLETAS DEL EEI y que EL PERISTALTISMO ESOFÁGICO
SEA NORMAL.
• Cual es el cuadro clínico del EEI hipertenso?
 R = DOLOR TORÁCICO O DISFAGIA. Mejorando generalmente el dolor tras el
tratamiento, siendo la disfagia la mas difícil de tratar.
• Cual es el manejo del EEI hipertenso?
 R = RELAJANTES DE LA MUSCULATURA LISA
SX DUMPING
CÁNCER GÁSTRICO
• Cual es la estirpe etiológica mas común del cáncer gástrico?
 R = ADENOCARCINOMA.
• Cual es el cuadro clínico de un paciente con cáncer gástrico?
 R = PERDIDA DE PESO, dolor abdominal, SACIEDAD TEMPRANA. Generalmente se presentan
con palidez. Entre las manifestaciones dermatológicas mas frecuentes se encuentran la
ACANTOSIS NIGRICANS y SIGNO DE LESER-TRELAT (queratosis seborreica de aparición
súbita)
• Como se manifiesta la metastasis comúnmente el cáncer gástrico?
 R = GANGLIOS EN SUPRACLAVICULAR (Virchow), GANGLIOS PERIUMBILICALES (Hermana
Maria Jose), estructuras rectales (signo del anaquel de blúmer), OVARIOS (tumor de
Krukenberg).
• Cual es el estándar de oro para el diagnostico del cáncer gástrico?
 R = PANENDOSCOPIA con toma de 7 BIOPSIAS de la lesión y después TAC PARA
ESTADIFICACION en caso de metástasis.
• Cual es el manejo del cáncer gástrico?
 R = Quirúrgico. La gastrectomía total se recomienda en caso de lesiones en el estomago
proximal. La QUIMIOTERAPIA no mejora la sobrevida sobre la cirugía, sin embargo
COMBINADA CON LA RADIOTERAPIA si la mejora.
LINFOMA GASTRICO
• 60. Cual es la estirpe etiológica mas común en el linfoma gástrico?
 R = No HODKING 95%
• 61. Cual es el cuadro clínico del linfoma gástrico?
 R = Sintomatología no especifica como DOLOR ABDOMINAL, nausea,
vomito, PERDIDA DE PESO, SUDORES NOCTURNOS y diarrea.
• 62. Cual es la apariencia del linfoma gástrico a la endoscopia?
 R = Aparecen como LESIONES FUNGOIDES, PÓLIPOS, ENGROSAMIENTO DE
PLIEGUES.
• 63. Cual es el manejo del linfoma gástrico?
 R = Quirúrgico
MALABSORCION
• Cuales son los datos clínicos de malabsorción?
 R = Diarrea, DISMINUCIÓN DE PESO y desnutrición, acompañados de
DISTENCIÓN ABDOMINAL, síntomas y signos secundaros a los déficit
específicos, ESTEATORREA.
• Que datos de laboratorio se encuentran en un paciente con malabsorción?
 R = Hipocolesterolemia, HIPOALBUMINEMIA, alteraciones electrolíticas y PH.
• Cual es la posología de la prueba con D-Xilosa en malabsorción?
 R = Se da EN AYUNAS midiendo su nivel en suero 1hr después y la excreción en
orina en 6hr, este azúcar SE ABSORBE EN FORMA PASIVA SI LA MUCOSA
INTESTINAL ESTA INTEGRA.
• Cuales son las principales causas de malabsorción?
 R = DEFICIENCIA DE LACTASA, ESPRUE celiaco, ESPRUE TROPICAL,
ENFERMEDAD DE WIPPLE, sobre crecimiento bacteriano y síndrome de
intestino corto.
ESPRUE CELIACO
• Que es el esprue celiaco?
 R = Causa malabsorción debido a lesión ocasionada por el GLUTEN a nivel intestinal, el cual
es contenido en el trigo, CEBADA, CENTENO Y AVENA.
• Cual es el cuadro clínico del esprue celiaco?
 R = Hay malabsorción de CHBTS, PROTEÍNAS Y GRASAS. Perdida progresiva de peso,
distensión abdominal, flatulencia y diarrea, evacuaciones voluminosas y de mal olor.
• Cuales estudios de diagnostico hay para el esprue celiaco y cuales son los anticuerpos
especificos?
1) La biopsia en LA UNIÓN DUODENOYUYENAL que suele REPORTAR ATROFIA O
ACORTAMIENTO DE LAS VELLOSIDADES, HIPERPLASIA DE LAS CRIPTAS y aumento en la
mitosis CON INCREMENTO DE LINFOCITOS, NEUTROFILOS, MASTOCITOS Y EOSINOFILOS EN
LAMINA PROPIA.
2) Cuantificación de grasa en heces de 24 hrs mayor a 7g/dia , la ABSORCIÓN DE D-XILOSA
SUELE SER ANORMAL, los ANTICUERPOS SÉRICOS ANTIGLIADINA SON POSITIVOS EN 90%,
el anticuerpo IgA es el mas especifico.
3) Los anticuerpos IgA ANTIENDOMISIO Y ANTIRRETICULINA son los
MAS ESPECÍFICOS.
• Cual es el manejo del esprue celiaco?
 R = Dieta libre de gluten
ESPRUE TROPICAL
• Cual es la etiología del esprue tropical?
 R = Es producido por la COLONIZACIÓN PERSISTENTE del intestino delgado por KLEBSIELLA
PNEUMONIAE, ENTEROBACTER CLOACAE o E. COLI.
• Cual es el cuadro clínico del esprue tropical?
1) CRISIS DE DIARREA ACUOSA, acompañada de DOLOR ABDOMINAL y FLATULENCIA, los
cuales disminuyen de intensidad a la semana de evolución y posteriormente se hacen
crónicos.
2) ANEMIA MEGALOBLASTICA.
• Cuales son los estudios diagnósticos del esprue tropical?
1) La biopsia de intestino delgado reporta ALARGAMIENTO DE LAS CRIPTAS, ensanchamiento
y acortamiento de las vellosidades e INFILTRACIÓN DE LA LAMINA PROPIA POR CÉLULAS
INFLAMATORIAS.
2) Se encuentra esteatorrea, ABSORCIÓN DE D-XILOSA ANORMAL.
• Cual es el manejo del esprue tropical?
 R = TETRACICLINA y ACIDO FÓLICO para mejorar ANEMIA MEGALOBLASTICA.
ENFERMEDAD DE WHIPPLE
• Cual es el agente causal de la enfermedad de Whipple y con que HLA se relaciona?
 R = Es secundaria a la infección por TROPHERYMA WHIPELLII y se relaciona con HLA B27 + en 25%.
• Cual es el cuadro clínico de la enfermedad de Whipple?
 R = Perdida de peso, diarrea o esteatorrea, artralgias, FIEBRE, dolor abdominal y HEMORRAGIA
INTESTINAL, LAS ARTRALGIASSON MIGRATORIAS, no destructivas y afectan las grandes
articulaciones, se presenta SACROILEITIS EN 25%. Un signo característico es la MIORRITMIA
OCULOMASTICATORIA, la cual consiste en la convergencia rítmica de los ojos asociada a
contracciones sincrónicas de los músculos de la masticación. A nivel ocular se manifiesta como
UVEÍTIS, vitreitis, retinitis y papiledema. A la ex fis se encuentra LINFADENOPATIA
periférica.
• Cuales son los hallazgos de laboratorio en la enfermedad de Whipple?
 R = La biopsia de intestino delgado muestra MACRÓFAGOS POSITIVOS CON LA TINCIÓN PAS. Sin
embargo se debe demostrar el AGENTE CAUSAL EN EL LCR, tejido cerebral o GANGLIOS LINFÁTICOS.
Anemia secundaria a enfermedad crónica, deficiencia de hierro, folato o VB12, asociada a neutrofilia.
Prueba de D-Xilosa anormal, LINFADENOPATIA ABDOMINAL detectada por ultrasonido.
Radiografía de TORAX CON INFILTRADOS, FIBROSIS O DERRAME PLEURAL.
• Cual es el manejo de la enfermedad de Whipple y por cuanto tiempo se realiza?
 R = TMP/SMZ durante 1ª, si hay ALERGIA A SULFAMIDAS esta indicada la PENICILINA por vía oral
durante el mismo periodo y se deben tratar los trastornos nutricios.
SX COLON IRRITABLE
• Cuales son los criterios de ROMA II para integrar el síndrome de colon irritable?
 R = Que los síntomas hallan durado POR LO MENOS 12 SEMANAS (no necesariamente
continuas) durante los 12m previos de malestar o dolor abdominal que tiene 2 de las
siguientes características. Se dice que el síndrome de intestino irritable se diagnostica
cuando cumple al menos con 1 o mas de los criterios 2, 4 y 6 o bien 1, 3 y 5 DEPENDIENDO
DE SI ES DIARREA LA QUE PREDOMINA O CONSTIPACIÓN. Se exacerba con el estrés y es
MAS FRECUENTE EN MUJERES.
1. Menos de 3 evacuaciones por semana
2. Mas de 3 evacuaciones al día
3. Evacuaciones de consistencia dura
4. Evacuaciones sueltas o duras
5. Pujo al evacuar
6. Urgencia al evacuar
7. Sensación de evacuación completa
8. Moco durante la evacuación
9. Distención abdominal
• Cual es el manejo del síndrome de colon irritable?
 R = Debe ser INDIVIDUALIZADO a los síntomas. PInaverio con dimeticona.
POLIPOS DEL COLON
• Cual es la clasificación de los pólipos neoplásicos de colon?
 R = A) TUBULARES 75% que corresponde a ADENOMAS. B) VELLOSOS 10% de los adenomas, mas
relacionado con cáncer colonico y C) TUBULOVELLOSOS 15% de los adenomas. Requiriéndose un periodo
mínimo de 10ª para que un adenoma se convierta a cáncer.
• Cuales son las alteraciones genéticas implicadas en los adenomas y el cáncer de colon?
 R = Mutación del GEN RAS y lesiones de los cromosomas 5, 17 y 18.
• Cual es el cuadro clínico de un paciente con SÍNDROME DE PEUTZ-JEGHERS?
 R = Es un TRASTORNO AD caracterizado POR MANCHAS HIPERPIGMENTADAS en los labios, mucosa bucal, cara
y dedos, PRESENTANDO HAMARTROMAS EN TODO EL TUBO DIGESTIVO e INTUSUCEPCION CON STDB.
• Cual es el manejo del Síndrome de Peutz-Jeghers?
 R = Resección de pólipo sintomático por endoscopia
• Cual es el cuadro clínico de la poliposis juvenil difusa?
 R = Es un TRASTORNO AD caracterizado por una población homogénea de pólipos, tanto hamartromas como
adenomas, los pólipos pueden ocasionar hemorragia e intususcepción en este síndrome. 10% RIESGO DE
DESARROLLO DE CA DE COLON.
• Cual es el manejo de la poliposis juvenil difusa?
 R = RESECCIÓN del colon con y anastomosis ileorectal. SEGUIMIENTO CON PROCTOSCOPIA CADA 6M y
extirpación de cualquier pólipo nuevo.
• Cual es el cuadro clínico del SÍNDROME DE COWDEN?
1) Enfermedad AD caracterizada por HAMARTROMAS en todo el tubo digestivo, PAPULAS FACIALES Y
BUCALES, CRECIMIENTOS QUERATOSICOS EN MANOS Y PIES.
2) Estos pacientes pueden presentar cáncer de mama, tiroides o de útero.
• Cual es el cuadro clínico del SÍNDROME DE CRONKHITE-CANADA?
 R = NO ES HEREDITARIO y se caracteriza por HAMARTROMAS INTESTINALES generalizados, ALOPECIA,
PIGMENTACIÓN CUTÁNEA y ATROFIA DE NAILS. Los SÍNTOMAS INCLUYEN VOMITO, DIARREA,
MALABSORCIÓN y enteropatía con perdida de proteínas, LA MAYORÍA DE LOS PACIENTES MUERE POCO
DESPUÉS DEL DIAGNOSTICO.
• Cual es el cuadro clínico de la poliposis adenomatosa familiar?
 R = TRASTORNO AD y su defecto se localiza en el CROMOSOMA 5. Se caracteriza por la presencia de mas de
100 POLIPOS en todo el colon y recto, 100% DESARROLLA CÁNCER.
• Cual es el manejo de la poliposis adenomatosa familiar?
 R = Resección con RECTOCOLECTOMIA total con ileostomía.
• Cual es el cuadro clínico de la ENFERMEDAD DE GARDNER?
 R = POLIPOSIS ADENOMATOSA familiar que se asocia a OSTEOMIELITIS, QUISTES EPIDERMOIDES Y
FIBROMAS DE LA PIEL.
• Que es el SÍNDROME DE TURCOT?
 R = POLIPOSIS ADENOMATOSA FAMILIAR que se asocia con NEOPLASIAS DEL SNC.
• Como se manifiesta un Adenocarcinoma de colon ubicado en el lado derecho?
 R = Perdida de peso, HEMATOQUZIA, anemia por déficit de hierro y UNA MASA ABDOMINAL EN EL CID.
• Como se manifiesta un Adenocarcinoma de colon ubicado en el lado izquierdo?
 R = Perdida de peso, cambios en hábitos intestinales, RECTORRAGIA y dolor abdominal tipo cólico. Se debe
realizar una colonoscopia con biopsia.
TROMBOSIS MESENTERICA
• Que pacientes tienen el riesgo de desarrollo de trombosis venosa
mesentérica?
 R = Pacientes traumatizados, hipertensión portal, peritonitis o con
hipercoagubilidad
• Cual es el cuadro clínico de la trombosis venosa mesentérica?
 R = DOLOR ABDOMINAL DIFUSO, DISTENCIÓN ABDOMINAL, nauseas,
vómitos y signos de deshidratación
• Cual es el estándar de oro para el dx de la trombosis venosa mesentérica?
 R = TAC EN ESPIRAL con medio de contraste oral e IV
• Cual es el manejo de la TVM?
 R = ANTI COAGULACIÓN E HIDRATACIÓN.
STDB
TIPS STDA Y STDB
o Los pacientes que requieran mas de 10u de paquetes globulares deberán recibir
también PLASMA FRESCO CONGELADO, PLAQUETAS O AMBOS.
o La colocación de BANDAS ELÁSTICAS es el tratamiento de elección en la rotura de
VARICES ESOFÁGICAS.
o El LAVADO NASO GÁSTRICO se utiliza para DIFERENCIAR entre el STDA Y STDB.
o La POSITIVIDAD EN EL ASPIRADO del tubo naso gástrico, RUIDOS INTESTINALES
HIPERACTIVOS y elevación de los niveles del BUN apoyan el diagnostico de STDA.
o En la hematemesis secundaria a la ingesta de AINES se debe realizar una
esofagogastroscopia y administrar un inhibidor de la bomba.
o El hematoma retroperitoneal produce una IMAGEN DE VIDRIO DESPULIDO en la
radiografía de abdomen.
ABSCESO ANAL
• Cuales son las causas de absceso anorrectal especificas y no especificas?
 R = ESPECIFICAS: Chron, CUCI, Tb, actinomicosis, ca, leucemias. NO
ESPECIFICAS: Infección bacteriana de glándulas anales
• Cual es la complicación principal que sucede al drenar un absceso anal?
 R = Posterior formación de FISTULAS
• Cual es el cuadro clínico en un px que presenta un absceso anal?
 R = DOLOR ANAL INTENSO o una TUMORACIÓN ANAL MUY DOLOROSA que
apareció en forma reciente.
• Cual es el manejo de un absceso perianal?
 R = DRENAJE Y METRONIDAZOL O CIPROFLOXACINO.
TUMORES DEL ANO
FISTULA ANAL
• Cual es la etiología de una fistula anal?
 R = Abscesos crónicos, CHRON Y CUCI.
• Cual es el cuadro clínico de las fistulas anales?
 R = Es característico la historia de un ABSCESO PREVIAMENTE DRENADO en la
misma localización, DOLOR PERIANAL, la principal molestia es la PERMANENTE
SALIDA DE MATERIAL PURULENTO a través de un orificio que MANCHA LA ROPA
INTERIOR.
• Cual es el estudio de imagen utilizado para el diagnostico de las fistulas anales?
 R = ULTRASONIDO RECTAL
• Cual es el manejo de las fistulas anales?
 R = La forma para predecir por donde drena la fistula es la regla de GOODSALL-
SALMON y una vez identificada se realiza FISTULOTOMIA.
FISURA ANAL
• Cual es la etiología de las fisuras anales?
 R = ISQUEMIA EN MUCOSA POR ESFÍNTER HIPERTÓNICO.
• Cual es el cuadro clínico de las fisuras anales?
 R = DOLOR intenso TIPO ARDOROSO que se relaciona al inicio
de la evacuación con una SENSACIÓN DE DESGARRO
al pasar el bolo fecal. SANGRADO FRECUENTE, suele ser en
POCA CANTIDAD.
• Cual es el manejo de las fisuras anales?
 R = Baños de asiento, laxante, RELAJANTE DE MUSCULATURA
PÉLVICA COMO CREMA DE ISOSORBIDE, DILTIAZEM O
NIFEDIPINO.
HEMORROIDES
HEMORROIDES
HERNIAS ABDOMINALES
• De donde emerge la hernia inguinal directa?
 R = Emerge DENTRO DE LOS VASOS EPIGÁSTRICOS originándose en el piso del
conducto inguinal
• De donde emerge la hernia inguinal indirecta?
 R = Por FUERA DE LOS VASOS EPIGÁSTRICOS y son DE ORIGEN CONGÉNITO siendo
mas FRECUENTES DEL LADO DERECHO.
• De que manera se distingue a la ex fis una hernia inguinal directa de indirecta?
 R = Una HERNIA INDIRECTA EJERCE CIERTA FUERZA CONTRA LA PUNTA DEL DEDO,
en tanto que la DIRECTA PRESIONA LA PULPA DEL DEDO. Además cuando es
colocado el dedo en el orificio inguinal y pedirle al paciente que tosa es posible
parar la hernia indirecta y no así la directa.
• Cual es la localización de la hernia crural?
 R = DEBAJO DEL LIGAMENTO INGUINAL, medial a los vasos femorales
manifestándose CON DOLOR EN LA REGIÓN INGUINAL, progresivo e incapacitante.
CAUSAS DE ELEVACION DEL AST O ALT
HEPATITIS VIRAL
VHA-PICORNAVIRUS
• Cual es el dato de laboratorio que orienta al VHA?
 R = IgM VS VHA AGUDO E IgG CUANDO YA HABÍA SIDO INFECTADO EL
PACIENTE
• Cual es el manejo del VHA?
 R = Sintomático, Inmunización pasiva como profilaxis postexposicion.
• Cuales son las indicaciones de la VACUNA PASIVA del VHA?
 R = PERSONAS QUE PLANEAN UN VIAJE, contactos íntimos, PERSONAL DE
GUARDERÍAS, ASILOS E INSTITUCIONES. NO esta INDICADA en px que tienen
DATOS CLÍNICOS DEL VHA
• Cuales son las indicaciones de la VACUNA ACTIVA del VHA?
 R = ENFERMEDAD HEPÁTICA CRÓNICA, VARONES HOMOSEXUALES,
DROGADICTOS.
VHB- HEPADNAVIRIDAE
• Cual es el dato de laboratorio que te orienta a VHB?
1) HBs Ag, la persistencia de >6m sugiere infección crónica.
2) Anti HBsconfiere inmunidad protectora ante el virus.
3) Hbe Ages el antígeno que refiere la REPLICACIÓN ACTIVA del virus y la
CONTAGIOSIDAD, en px con infección crónica permanece constante.
4) IgM antiHBc infección aguda.
5) - 10% EVOLUCIONA A CRONICIDAD, BUEN PRONOSTICO. TX EN CRONICO
• Cual es el manejo del VHB?
1) INTERFERON ALFA
2) LAMIVUDINA (análogo de nucleosidos que inhibe la TRANSCRIPTASA REVERSA),
3) ADEFOVIR (análogo de nucleótidos sintético que inhibe a la DNA polimerasa del VHB),
4) ENTECAVIR ( análogo nucleosido deoxiguanina que inhibe la actividad del DNA
polimerasa del VHB).
VHD
• Que caracteriza al VHD?
 R = Es EL VIRUS MAS PEQUEÑO con 36 nm, NECESITA DE LA ENVOLTURA DEL VHB para
sobrevivir y su medio de CONTAGIO ES EL MISMO QUE EL DEL VHB.
• Cual es el curso del VHD en coinfección aguda con VHB?
 R = Tiende a ser LIMITADO Y ELIMINAR EL HBsAg POR LA RESPUESTA INMUNE, al mismo
tiempo que desaparece el VHD.
• Cual es el curso del VHD en súperinfección con VHB?
 R = Exposición del VHD EN INDIVIDUO PREVIAMENTE INFECTADO POR VHB. LA
REPLICACIÓN DEL VHD ES MAYOR Y MÁS RÁPIDA al encontrar previamente expresado al
HBs Ag. En 70% de pacientes de los casos la infección se vuelve crónica y se encuentran
mayores tasas de mortalidad
• Cual es el estudio de laboratorio que orienta al diagnostico del VHD?
 R = IgG anti VHD o por detección de RNA DE VHD EN SUERO.
• Cual es el manejo del VHD?
 R = No hay tratamiento especifico, se ha encontrado mejoría con INTERFERON ALFA.
VHE- CALICIVIRUS
FAMILIAS DE HEPATITIS VIRICA
• VHA
PICORNAVIRUS
• VHB
HEPADNAVIRUS
• VHC
FLAVIVIRUS
• VHE
CALICIVIRUS
HEPATITIS AUTOINMUNE
• Que predisposición genética tienen las personas que padecen hepatitis autoinmune?
 R = HLA-B8, HLA-DR3 Y DR-52.
• Que clasificaciones tiene la hepatitis autoinmune?
 TIPO I: Autoinmune clásica o Lupoide, se presenta en mujeres jóvenes donde 30% presenta trastornos como
TIROIDITIS AUTOINMUNE O CUCI. A diferencia de otros grupos se presenta HIPERGAMAGLOBULINEMIA EN
SUERO 5-6 GR/DL.
 TIPO II: Mas común en Europa y los anticuerpos que distinguen a este grupo son los ANTIMICROMOSOMA DE
HÍGADO Y RINON TIPO 1 (ANTI LKM 1), asociada a enfermedades inmunológicas como el VITÍLIGO, DM 1,
TIROIDITIS AUTOINMUNE.
 TIPO III: Se encuentran anticuerpos anti antígeno hepático soluble/hígado- páncreas (anti-SLA/LP) encontrando
mas frecuentemente la TIROIDITIS AUTOINMUNE COMO ASOCIADA.
• Cual es el cuadro clínico de las hepatitis autoinmunes?
 R = MÚLTIPLES TELANGIECTASIAS, astenia, ARTRALGIAS, ESTRIAS CUTÁNEAS, acné, hirsutismo, AMENORREA Y
HEPATOMEGALIA.
• Cuales son los CRITERIOS diagnósticos para hepatitis autoinmune?
1) MAYORES: Elevación persistente de transaminasas, Hipergamaglobulinemia 2.5 o mayor, Ac circulantes y/o
células LE positivas, marcadores serológicos para virus de hepatitis negativos.
2) MENORES: Manifestaciones sistémicas como fiebre-artralgias-erupciones cutáneas, Complicaciones con
otras enfermedades autoinmunes , incluyendo las del colágeno, eritrosedimentacion elevada.
3) HISTOLOGIA: Hepatitis crónica o cirrosis con marcada infiltración de células y necrosis de hepatocitos,
ausencia de hallazgos específicos para el diagnóstico de otras entidades.
• Cual es el manejo de la hepatitis autoinmune?
 R = De elección son los GLUCOCORTICOIDES SOLOS O COMBINADOS CON AZATRIOPINA. La remisión se
considera cuando los px demuestran mejoría clínica, niveles de transaminasas y gammaglobulinas normales en
caso de recidivas se usa tratamiento inmunosupresor de por vida.
HEPATITIS ALCOHOLICA
• Cual es el cuadro clínico de un paciente con hepatitis alcohólica?
 R = Perdida de peso, anorexia, ICTERICIA, FIEBRE Y
HEPATOMEGALIA DOLOROSA.
• Cuales son los hallazgos de laboratorio en la hepatitis alcohólica?
1) AUMENTO DE LAS ENZIMAS HEPÁTICAS
2) ANEMIA
3) TROMBOCITOPENIA
4) LEUCOCITOSIS CON PREDOMINIO DE NEUTROFILOS Y
5) SE CONFIRMA EL DIAGNOSTICO POR BIOPSIA.
• Cual es el manejo de la hepatitis alcohólica?
 R = Abstinencia, CORTICOIDES y plan nutricio.
CIRROSIS HEPATICA
• Que parámetros definen la presión portal?
 R = Flujo venoso portal y la resistencia interior del hígado
• Cuales son las causas mas frecuentes de cirrosis hepática?
 R = Alcohol, HEPATITIS B Y C, METOTREXATO, alfametildopa e
hidralazina.
• Cual es la clasificación morfológica de la cirrosis?
1) CIRROSIS MICRONODULAR: Nódulos menores de 3 mm.
2) CIRROSIS MACRONODULAR: Nódulos de mas de 3 mm.
3) CIRROSIS MIXTA: Mircro y macronodulares que ocurre comúnmente por
alcohol.
• Cual es el marcador mas útil para el diagnostico de cirrosis biliar primaria?
 R = ANTICUERPOS ANTIMITOCONDRIALES.
• Cual es el cuadro clínico de la cirrosis?
 R = Asintomatico o los siguientes:
A. SINTOMAS CONSTITUCIONALES:
B. Astenia, anorexia, perdida de peso muscular.
C. ASCITIS:
D. Por retención de sodio y agua.
E. SIGNOS CUTANEOS:
F. Ictericia, palidez por la anemia. Telangiectacias cutáneas y eritema palmar (que predomina sobre las eminencias tenar e
hipotenar y en las llemas de los dedos) son datos bastantes caracteristicos.
G. CIRCULACION COLATERAL:
H. Los sitios mas comunes de aparición con significado patológico son el esófago y el fondo gástrico.
I. ALIENTO:
J. Olor dulzón debido a la exhalación de mercaptanos, productos derivados de la metionina por defecto de su metilación.
K. ALTERACIONES ENDOCRINAS:
L. Varones con signos de feminización como ginecomastia y cambios en la distribución del vello coroporal. El hipogonadismo en
varones produce atrofia testicular, perdida de la libido, impotencia. En ambos sexos el vello axilar es escaso. En las mujeres es
frecuente la oligomenorrea, amenorrea y esterilidad.
M. ALTERACIONES HEMATOLOGICAS:
N. Alteraciones en la coagulación dado que todos los factores se sintetizan en hígado menos el factor de von Willebrand.
O. ALTERACIONES PULMONARES:
P. Ascitis torácica o derrame pleural
Q. ALTERACIONES DIGESTIVAS:
R. Hemorragia digestiva, colelitiasis.
S. ALTERACIONES RENALES:
T. Retención de sodio con ascitis
U. SX HEPATORRENAL:
V. Se trata de una insuficiencia renal funcional como respuesta a mecanismos de descompensación tales como el uso exagerado
de diuréticos, AINES, paracentesis. La histología es normal. Este síndrome se caracteriza por una insuficiencia renal funcional
con función tubular normal en un px con insuficiencia hepática.
W. INFECCIONES:
X. IVU mas comunes.
Y. ALTERACIONES NEUROLOGICAS:
Z. Encefalopatia hepática donde la primer manifestación en aparecer son los cambios en el comportamiento. El GABA es el
principal neurotransmisor inhibitorio del cerebro, se ha postulado que el GABA producido a nivel intestinal escapa del
metabolismo hepático, atraviesa la barrera hematoencefalica y penetra al cerebro desencadenando la encefalopatía hepática.
• 184. Cual es el manejo de la cirrosis hepática?
 R = Dieta con contenido de proteínas 1 a 1.2 g/gr. Prohibicion del consumo de
alcohol. Hay tratamientos utiles específicos de cirrosis, como las flebotomías en la
hemocromatosis, la d-penicilamina en la enfermedad de Wilson, los corticoides en
las cirrosis autoinmunes, los antivíricos para cirrosis por VHB y VHC y el acido
urodesoxicolico para la cirrosis biliar primaria. La cirugía se asocia a una mortalidad
operatoria del 30% pero las indicaciones para la cirugía son rotura de varices o
sangrados. A las varices tipo 2 y 3 se administra betabloqueador en caso que no
exista contraindicación. Seguimiento ecográfico cada 6m para detectar
hepatocarcinoma.
• 185. Cual es el manejo de emergencia en la cirrosis hepática?
 R = En caso de sangrado alto una endoscopia debe realizarse para dar
escleroterapia. Uso de vasopresina por via IV teniendo cuidado de la necrosis de la
piel. Anastomosis portosistemica intrahepática por via transyugular TIPS para
controlar la hemorragia digestica por varices esofágicas irreductible por otros
métodos.
HEPATOCARCINOMA
• Cuales son los factores de riesgo para desarrollar un hepatocarcinoma?
 R = Aparece en personas que ya tienen alguna enfermedad hepática como
HEPATITIS POR VIRUS B O C, CIRROSIS.
• Cual es el cuadro clínico de un px con hepatocarcinoma?
 R = Puede cursar asintomático. Cuando existen manifestaciones lo mas habitual es
que se presenten síntomas inespecíficos como baja de peso, dolor abdominal,
saciedad precoz o masa palpable.
Cuales son los estudios de gabinete utilizados para diagnosticar hepatocarcinoma?
 R = TAC DINÁMICA HELICOIDAL MULTICORTE DE ELECCIÓN. Us de seguimiento. La
ALFAFETOPROTEINA es el marcador tumoral mas utilizado para el dx y seguimiento
(puede elevarse en embarazo y tumores testiculares). Confirmación por biopsia.
• Cual es el manejo del hepatocarcinoma?
 R = RESECCIÓN del tumor. INYECCIÓN DE ETANOL en tumor lo reseca y es fácil de
realizar. QUIMIOEMBOLIZACION que frecuentemente se complica con el llamado
síndrome postembolizacion caracterizado por fiebre, elevación enzimática, dolor
abdominal y nauseas. TRANSPLANTE HEPÁTICO.
ABSCESO HEPATICO AMEBIANO
ENARM  COMPENDIO
ENARM  COMPENDIO
ENARM  COMPENDIO
ENARM  COMPENDIO
ENARM  COMPENDIO
ENARM  COMPENDIO
ENARM  COMPENDIO
ENARM  COMPENDIO
ENARM  COMPENDIO
ENARM  COMPENDIO
ENARM  COMPENDIO
ENARM  COMPENDIO
ENARM  COMPENDIO
ENARM  COMPENDIO
ENARM  COMPENDIO
ENARM  COMPENDIO
ENARM  COMPENDIO
ENARM  COMPENDIO
ENARM  COMPENDIO
ENARM  COMPENDIO
ENARM  COMPENDIO
ENARM  COMPENDIO
ENARM  COMPENDIO
ENARM  COMPENDIO
ENARM  COMPENDIO
ENARM  COMPENDIO
ENARM  COMPENDIO
ENARM  COMPENDIO
ENARM  COMPENDIO
ENARM  COMPENDIO
ENARM  COMPENDIO
ENARM  COMPENDIO
ENARM  COMPENDIO
ENARM  COMPENDIO
ENARM  COMPENDIO
ENARM  COMPENDIO
ENARM  COMPENDIO
ENARM  COMPENDIO
ENARM  COMPENDIO
ENARM  COMPENDIO
ENARM  COMPENDIO
ENARM  COMPENDIO
ENARM  COMPENDIO
ENARM  COMPENDIO
ENARM  COMPENDIO
ENARM  COMPENDIO
ENARM  COMPENDIO
ENARM  COMPENDIO
ENARM  COMPENDIO
ENARM  COMPENDIO
ENARM  COMPENDIO
ENARM  COMPENDIO
ENARM  COMPENDIO
ENARM  COMPENDIO
ENARM  COMPENDIO
ENARM  COMPENDIO
ENARM  COMPENDIO
ENARM  COMPENDIO
ENARM  COMPENDIO
ENARM  COMPENDIO
ENARM  COMPENDIO
ENARM  COMPENDIO
ENARM  COMPENDIO
ENARM  COMPENDIO
ENARM  COMPENDIO
ENARM  COMPENDIO
ENARM  COMPENDIO
ENARM  COMPENDIO
ENARM  COMPENDIO
ENARM  COMPENDIO
ENARM  COMPENDIO
ENARM  COMPENDIO
ENARM  COMPENDIO
ENARM  COMPENDIO
ENARM  COMPENDIO
ENARM  COMPENDIO
ENARM  COMPENDIO
ENARM  COMPENDIO
ENARM  COMPENDIO
ENARM  COMPENDIO
ENARM  COMPENDIO
ENARM  COMPENDIO
ENARM  COMPENDIO
ENARM  COMPENDIO
ENARM  COMPENDIO
ENARM  COMPENDIO
ENARM  COMPENDIO
ENARM  COMPENDIO
ENARM  COMPENDIO
ENARM  COMPENDIO
ENARM  COMPENDIO
ENARM  COMPENDIO
ENARM  COMPENDIO
ENARM  COMPENDIO
ENARM  COMPENDIO
ENARM  COMPENDIO
ENARM  COMPENDIO
ENARM  COMPENDIO
ENARM  COMPENDIO
ENARM  COMPENDIO
ENARM  COMPENDIO
ENARM  COMPENDIO
ENARM  COMPENDIO
ENARM  COMPENDIO
ENARM  COMPENDIO
ENARM  COMPENDIO
ENARM  COMPENDIO
ENARM  COMPENDIO
ENARM  COMPENDIO
ENARM  COMPENDIO
ENARM  COMPENDIO
ENARM  COMPENDIO
ENARM  COMPENDIO
ENARM  COMPENDIO
ENARM  COMPENDIO
ENARM  COMPENDIO
ENARM  COMPENDIO
ENARM  COMPENDIO
ENARM  COMPENDIO
ENARM  COMPENDIO
ENARM  COMPENDIO
ENARM  COMPENDIO
ENARM  COMPENDIO
ENARM  COMPENDIO
ENARM  COMPENDIO
ENARM  COMPENDIO
ENARM  COMPENDIO
ENARM  COMPENDIO
ENARM  COMPENDIO
ENARM  COMPENDIO
ENARM  COMPENDIO
ENARM  COMPENDIO
ENARM  COMPENDIO
ENARM  COMPENDIO
ENARM  COMPENDIO
ENARM  COMPENDIO
ENARM  COMPENDIO
ENARM  COMPENDIO
ENARM  COMPENDIO
ENARM  COMPENDIO
ENARM  COMPENDIO
ENARM  COMPENDIO
ENARM  COMPENDIO
ENARM  COMPENDIO
ENARM  COMPENDIO
ENARM  COMPENDIO
ENARM  COMPENDIO
ENARM  COMPENDIO
ENARM  COMPENDIO
ENARM  COMPENDIO
ENARM  COMPENDIO
ENARM  COMPENDIO
ENARM  COMPENDIO
ENARM  COMPENDIO
ENARM  COMPENDIO
ENARM  COMPENDIO
ENARM  COMPENDIO
ENARM  COMPENDIO
ENARM  COMPENDIO
ENARM  COMPENDIO
ENARM  COMPENDIO
ENARM  COMPENDIO
ENARM  COMPENDIO
ENARM  COMPENDIO
ENARM  COMPENDIO
ENARM  COMPENDIO
ENARM  COMPENDIO
ENARM  COMPENDIO
ENARM  COMPENDIO
ENARM  COMPENDIO
ENARM  COMPENDIO
ENARM  COMPENDIO
ENARM  COMPENDIO
ENARM  COMPENDIO
ENARM  COMPENDIO
ENARM  COMPENDIO
ENARM  COMPENDIO
ENARM  COMPENDIO
ENARM  COMPENDIO
ENARM  COMPENDIO
ENARM  COMPENDIO
ENARM  COMPENDIO
ENARM  COMPENDIO
ENARM  COMPENDIO
ENARM  COMPENDIO
ENARM  COMPENDIO
ENARM  COMPENDIO
ENARM  COMPENDIO
ENARM  COMPENDIO
ENARM  COMPENDIO
ENARM  COMPENDIO
ENARM  COMPENDIO
ENARM  COMPENDIO
ENARM  COMPENDIO
ENARM  COMPENDIO
ENARM  COMPENDIO
ENARM  COMPENDIO
ENARM  COMPENDIO
ENARM  COMPENDIO
ENARM  COMPENDIO
ENARM  COMPENDIO
ENARM  COMPENDIO
ENARM  COMPENDIO
ENARM  COMPENDIO
ENARM  COMPENDIO
ENARM  COMPENDIO
ENARM  COMPENDIO
ENARM  COMPENDIO
ENARM  COMPENDIO
ENARM  COMPENDIO
ENARM  COMPENDIO
ENARM  COMPENDIO
ENARM  COMPENDIO
ENARM  COMPENDIO
ENARM  COMPENDIO
ENARM  COMPENDIO
ENARM  COMPENDIO
ENARM  COMPENDIO
ENARM  COMPENDIO
ENARM  COMPENDIO
ENARM  COMPENDIO
ENARM  COMPENDIO
ENARM  COMPENDIO
ENARM  COMPENDIO
ENARM  COMPENDIO
ENARM  COMPENDIO
ENARM  COMPENDIO
ENARM  COMPENDIO
ENARM  COMPENDIO
ENARM  COMPENDIO
ENARM  COMPENDIO
ENARM  COMPENDIO
ENARM  COMPENDIO
ENARM  COMPENDIO
ENARM  COMPENDIO
ENARM  COMPENDIO
ENARM  COMPENDIO
ENARM  COMPENDIO
ENARM  COMPENDIO
ENARM  COMPENDIO
ENARM  COMPENDIO
ENARM  COMPENDIO
ENARM  COMPENDIO
ENARM  COMPENDIO
ENARM  COMPENDIO
ENARM  COMPENDIO
ENARM  COMPENDIO
ENARM  COMPENDIO
ENARM  COMPENDIO
ENARM  COMPENDIO
ENARM  COMPENDIO
ENARM  COMPENDIO
ENARM  COMPENDIO
ENARM  COMPENDIO
ENARM  COMPENDIO
ENARM  COMPENDIO
ENARM  COMPENDIO
ENARM  COMPENDIO
ENARM  COMPENDIO
ENARM  COMPENDIO
ENARM  COMPENDIO
ENARM  COMPENDIO
ENARM  COMPENDIO
ENARM  COMPENDIO
ENARM  COMPENDIO
ENARM  COMPENDIO
ENARM  COMPENDIO
ENARM  COMPENDIO
ENARM  COMPENDIO
ENARM  COMPENDIO
ENARM  COMPENDIO
ENARM  COMPENDIO
ENARM  COMPENDIO
ENARM  COMPENDIO
ENARM  COMPENDIO
ENARM  COMPENDIO
ENARM  COMPENDIO
ENARM  COMPENDIO
ENARM  COMPENDIO
ENARM  COMPENDIO
ENARM  COMPENDIO
ENARM  COMPENDIO
ENARM  COMPENDIO
ENARM  COMPENDIO
ENARM  COMPENDIO
ENARM  COMPENDIO
ENARM  COMPENDIO
ENARM  COMPENDIO
ENARM  COMPENDIO
ENARM  COMPENDIO
ENARM  COMPENDIO
ENARM  COMPENDIO
ENARM  COMPENDIO
ENARM  COMPENDIO
ENARM  COMPENDIO
ENARM  COMPENDIO
ENARM  COMPENDIO
ENARM  COMPENDIO
ENARM  COMPENDIO
ENARM  COMPENDIO
ENARM  COMPENDIO
ENARM  COMPENDIO
ENARM  COMPENDIO
ENARM  COMPENDIO
ENARM  COMPENDIO
ENARM  COMPENDIO
ENARM  COMPENDIO
ENARM  COMPENDIO
ENARM  COMPENDIO
ENARM  COMPENDIO
ENARM  COMPENDIO
ENARM  COMPENDIO

More Related Content

What's hot

Caso Clínico Terapéutico. Tormenta Tiroidea
Caso Clínico Terapéutico. Tormenta TiroideaCaso Clínico Terapéutico. Tormenta Tiroidea
Caso Clínico Terapéutico. Tormenta Tiroideaevidenciaterapeutica.com
 
Hemorragia Digestiva Alta
Hemorragia Digestiva AltaHemorragia Digestiva Alta
Hemorragia Digestiva Altaunidaddocente
 
Sindrome Coronario Agudo Sin Elevacion del Segmento ST (SICA SESST)
Sindrome Coronario Agudo Sin Elevacion del Segmento ST (SICA SESST)Sindrome Coronario Agudo Sin Elevacion del Segmento ST (SICA SESST)
Sindrome Coronario Agudo Sin Elevacion del Segmento ST (SICA SESST)Ricardo Mora MD
 
Hipertensión portal y ascitis
Hipertensión portal y ascitisHipertensión portal y ascitis
Hipertensión portal y ascitismarialebarriosm
 
Fe na (FRACCION DE EXCRESION DE SODIO)
Fe na (FRACCION DE EXCRESION DE SODIO)Fe na (FRACCION DE EXCRESION DE SODIO)
Fe na (FRACCION DE EXCRESION DE SODIO)Ana Luisa Paredes
 
Hemorragia de Vías Digestivas Altas
Hemorragia de Vías Digestivas Altas Hemorragia de Vías Digestivas Altas
Hemorragia de Vías Digestivas Altas DanielaRuizM1
 
Fisiopatologia de la Cirrosis Hepática
Fisiopatologia de la Cirrosis HepáticaFisiopatologia de la Cirrosis Hepática
Fisiopatologia de la Cirrosis Hepáticachecoesm
 
hematoma subdural, epidural, intraparenquimatoso
hematoma subdural, epidural, intraparenquimatosohematoma subdural, epidural, intraparenquimatoso
hematoma subdural, epidural, intraparenquimatosoPaola Dominguez
 

What's hot (20)

Reperfusion en iamcest
Reperfusion en iamcestReperfusion en iamcest
Reperfusion en iamcest
 
Caso Clínico Terapéutico. Tormenta Tiroidea
Caso Clínico Terapéutico. Tormenta TiroideaCaso Clínico Terapéutico. Tormenta Tiroidea
Caso Clínico Terapéutico. Tormenta Tiroidea
 
Hemorragia Digestiva Alta
Hemorragia Digestiva AltaHemorragia Digestiva Alta
Hemorragia Digestiva Alta
 
Sindrome Coronario Agudo Sin Elevacion del Segmento ST (SICA SESST)
Sindrome Coronario Agudo Sin Elevacion del Segmento ST (SICA SESST)Sindrome Coronario Agudo Sin Elevacion del Segmento ST (SICA SESST)
Sindrome Coronario Agudo Sin Elevacion del Segmento ST (SICA SESST)
 
Criterios de SIDA
Criterios de SIDA Criterios de SIDA
Criterios de SIDA
 
Hipertensión portal y ascitis
Hipertensión portal y ascitisHipertensión portal y ascitis
Hipertensión portal y ascitis
 
INSUFICIENCIA CARDIACA PRESENTACION
INSUFICIENCIA CARDIACA PRESENTACIONINSUFICIENCIA CARDIACA PRESENTACION
INSUFICIENCIA CARDIACA PRESENTACION
 
Pancreatitis
Pancreatitis Pancreatitis
Pancreatitis
 
(2021-03-18) INSUFICIENCIA CARDIACA (PPT)
(2021-03-18) INSUFICIENCIA CARDIACA (PPT)(2021-03-18) INSUFICIENCIA CARDIACA (PPT)
(2021-03-18) INSUFICIENCIA CARDIACA (PPT)
 
Apendicitis aguda final
Apendicitis aguda finalApendicitis aguda final
Apendicitis aguda final
 
Sindrome coronario agudo
Sindrome coronario agudo Sindrome coronario agudo
Sindrome coronario agudo
 
Shock (atls)
Shock (atls)Shock (atls)
Shock (atls)
 
Examen de rm 2012 parte b
Examen de rm 2012 parte bExamen de rm 2012 parte b
Examen de rm 2012 parte b
 
Fe na (FRACCION DE EXCRESION DE SODIO)
Fe na (FRACCION DE EXCRESION DE SODIO)Fe na (FRACCION DE EXCRESION DE SODIO)
Fe na (FRACCION DE EXCRESION DE SODIO)
 
Hemorragia subaracnoidea
Hemorragia subaracnoideaHemorragia subaracnoidea
Hemorragia subaracnoidea
 
Hemorragia de Vías Digestivas Altas
Hemorragia de Vías Digestivas Altas Hemorragia de Vías Digestivas Altas
Hemorragia de Vías Digestivas Altas
 
Fisiopatologia de la Cirrosis Hepática
Fisiopatologia de la Cirrosis HepáticaFisiopatologia de la Cirrosis Hepática
Fisiopatologia de la Cirrosis Hepática
 
hematoma subdural, epidural, intraparenquimatoso
hematoma subdural, epidural, intraparenquimatosohematoma subdural, epidural, intraparenquimatoso
hematoma subdural, epidural, intraparenquimatoso
 
Preguntas ca de pancreas
Preguntas ca de pancreasPreguntas ca de pancreas
Preguntas ca de pancreas
 
Dolor torácico
Dolor torácicoDolor torácico
Dolor torácico
 

Viewers also liked (14)

PEDIATRIA ENARM
PEDIATRIA ENARMPEDIATRIA ENARM
PEDIATRIA ENARM
 
ENARM Obstetricia ginecologia
ENARM Obstetricia ginecologiaENARM Obstetricia ginecologia
ENARM Obstetricia ginecologia
 
Huitron
HuitronHuitron
Huitron
 
Temario enarm 2013
Temario enarm 2013Temario enarm 2013
Temario enarm 2013
 
4.GASTROENTEROLOGIA Y CIRUGIA
4.GASTROENTEROLOGIA Y CIRUGIA4.GASTROENTEROLOGIA Y CIRUGIA
4.GASTROENTEROLOGIA Y CIRUGIA
 
Para medicina escensi 2
Para medicina escensi 2Para medicina escensi 2
Para medicina escensi 2
 
1.HEMATOLOGIA
1.HEMATOLOGIA1.HEMATOLOGIA
1.HEMATOLOGIA
 
10.NEUMOLOGIA
10.NEUMOLOGIA10.NEUMOLOGIA
10.NEUMOLOGIA
 
7.DERMATOLOGIA
7.DERMATOLOGIA7.DERMATOLOGIA
7.DERMATOLOGIA
 
Repaso ENARM Nefrología
Repaso ENARM NefrologíaRepaso ENARM Nefrología
Repaso ENARM Nefrología
 
Enarm 2014
Enarm 2014Enarm 2014
Enarm 2014
 
2.CARDIOLOGIA
2.CARDIOLOGIA2.CARDIOLOGIA
2.CARDIOLOGIA
 
Hipertension en el embarazo ACOG 2013
Hipertension en el embarazo ACOG 2013Hipertension en el embarazo ACOG 2013
Hipertension en el embarazo ACOG 2013
 
Enarm 2145 preguntas
Enarm 2145 preguntasEnarm 2145 preguntas
Enarm 2145 preguntas
 

Similar to ENARM COMPENDIO

Lesion renal aguda, enfermedad renal cronica, sx. nefritico, sx. nefrotico
Lesion renal aguda, enfermedad renal cronica, sx. nefritico, sx. nefroticoLesion renal aguda, enfermedad renal cronica, sx. nefritico, sx. nefrotico
Lesion renal aguda, enfermedad renal cronica, sx. nefritico, sx. nefroticoErick Ojeda Mendoza
 
microangiopatia trombocitopenica trombotica
microangiopatia trombocitopenica tromboticamicroangiopatia trombocitopenica trombotica
microangiopatia trombocitopenica tromboticaquintocuarto4
 
Copia de SDX MIELOPROLIFERATIVOO.pdf
Copia de SDX MIELOPROLIFERATIVOO.pdfCopia de SDX MIELOPROLIFERATIVOO.pdf
Copia de SDX MIELOPROLIFERATIVOO.pdfGUERYCICELYCASTILLOM
 
Alteraciones plaquetarias
Alteraciones plaquetariasAlteraciones plaquetarias
Alteraciones plaquetariasRicardo Perez
 
Laboratorio hematologico anemias leucemias
Laboratorio hematologico anemias leucemiasLaboratorio hematologico anemias leucemias
Laboratorio hematologico anemias leucemiasYesi VZ
 
PATOLOGIA RENAL Y DE VIAS URINARIAS 2022.pptx
PATOLOGIA RENAL Y DE VIAS URINARIAS 2022.pptxPATOLOGIA RENAL Y DE VIAS URINARIAS 2022.pptx
PATOLOGIA RENAL Y DE VIAS URINARIAS 2022.pptxShermelyLeydiBozaGer
 
Enfermedades autoinmunes 1
Enfermedades autoinmunes 1Enfermedades autoinmunes 1
Enfermedades autoinmunes 1Karla González
 
HEMATO TROMBOCITOPENIAS.pptx
HEMATO TROMBOCITOPENIAS.pptxHEMATO TROMBOCITOPENIAS.pptx
HEMATO TROMBOCITOPENIAS.pptxMaria136071
 
Go Clase 35 Anemia En El Embarazo Dr Fuster
Go Clase 35 Anemia En El Embarazo Dr FusterGo Clase 35 Anemia En El Embarazo Dr Fuster
Go Clase 35 Anemia En El Embarazo Dr FusterDanteVallesH
 

Similar to ENARM COMPENDIO (20)

Lesion renal aguda, enfermedad renal cronica, sx. nefritico, sx. nefrotico
Lesion renal aguda, enfermedad renal cronica, sx. nefritico, sx. nefroticoLesion renal aguda, enfermedad renal cronica, sx. nefritico, sx. nefrotico
Lesion renal aguda, enfermedad renal cronica, sx. nefritico, sx. nefrotico
 
microangiopatia trombocitopenica trombotica
microangiopatia trombocitopenica tromboticamicroangiopatia trombocitopenica trombotica
microangiopatia trombocitopenica trombotica
 
Copia de SDX MIELOPROLIFERATIVOO.pdf
Copia de SDX MIELOPROLIFERATIVOO.pdfCopia de SDX MIELOPROLIFERATIVOO.pdf
Copia de SDX MIELOPROLIFERATIVOO.pdf
 
5.NEFROLOGIA
5.NEFROLOGIA5.NEFROLOGIA
5.NEFROLOGIA
 
Alteraciones plaquetarias
Alteraciones plaquetariasAlteraciones plaquetarias
Alteraciones plaquetarias
 
Laboratorio hematologico anemias leucemias
Laboratorio hematologico anemias leucemiasLaboratorio hematologico anemias leucemias
Laboratorio hematologico anemias leucemias
 
Preeclampsia eclampsia
Preeclampsia eclampsiaPreeclampsia eclampsia
Preeclampsia eclampsia
 
Sindrome mieloproliferativo
Sindrome mieloproliferativoSindrome mieloproliferativo
Sindrome mieloproliferativo
 
FIEBRE REUMÁTICA.docx
FIEBRE REUMÁTICA.docxFIEBRE REUMÁTICA.docx
FIEBRE REUMÁTICA.docx
 
Interpretacion del-hemograma
Interpretacion del-hemogramaInterpretacion del-hemograma
Interpretacion del-hemograma
 
SRIS
SRISSRIS
SRIS
 
PATOLOGIA RENAL Y DE VIAS URINARIAS 2022.pptx
PATOLOGIA RENAL Y DE VIAS URINARIAS 2022.pptxPATOLOGIA RENAL Y DE VIAS URINARIAS 2022.pptx
PATOLOGIA RENAL Y DE VIAS URINARIAS 2022.pptx
 
Purpuras
PurpurasPurpuras
Purpuras
 
Purpuras 111113040632-phpapp02
Purpuras 111113040632-phpapp02Purpuras 111113040632-phpapp02
Purpuras 111113040632-phpapp02
 
Enfermedades autoinmunes 1
Enfermedades autoinmunes 1Enfermedades autoinmunes 1
Enfermedades autoinmunes 1
 
HEMATO TROMBOCITOPENIAS.pptx
HEMATO TROMBOCITOPENIAS.pptxHEMATO TROMBOCITOPENIAS.pptx
HEMATO TROMBOCITOPENIAS.pptx
 
Sindrome nefrotico
Sindrome nefroticoSindrome nefrotico
Sindrome nefrotico
 
Miocardiopatias UAN
Miocardiopatias UANMiocardiopatias UAN
Miocardiopatias UAN
 
Surviving sepsis
Surviving sepsisSurviving sepsis
Surviving sepsis
 
Go Clase 35 Anemia En El Embarazo Dr Fuster
Go Clase 35 Anemia En El Embarazo Dr FusterGo Clase 35 Anemia En El Embarazo Dr Fuster
Go Clase 35 Anemia En El Embarazo Dr Fuster
 

Recently uploaded

Cartilla Nacional Mexicana de 10 a 19 años
Cartilla Nacional Mexicana de 10 a 19 añosCartilla Nacional Mexicana de 10 a 19 años
Cartilla Nacional Mexicana de 10 a 19 añosLauraGarduza2
 
Psicología: Revista sobre las bases de la conducta humana.pdf
Psicología: Revista sobre las bases de la conducta humana.pdfPsicología: Revista sobre las bases de la conducta humana.pdf
Psicología: Revista sobre las bases de la conducta humana.pdfdelvallepadrob
 
Radiologia_de_Urgencias_y_Emergencias_3deg_Ed.pdf
Radiologia_de_Urgencias_y_Emergencias_3deg_Ed.pdfRadiologia_de_Urgencias_y_Emergencias_3deg_Ed.pdf
Radiologia_de_Urgencias_y_Emergencias_3deg_Ed.pdfAntonioRicardoOrrego
 
6.METODOLOGIA ATENEA MICHAEL. ZAPATA.pdf
6.METODOLOGIA ATENEA MICHAEL. ZAPATA.pdf6.METODOLOGIA ATENEA MICHAEL. ZAPATA.pdf
6.METODOLOGIA ATENEA MICHAEL. ZAPATA.pdfbibianavillazoo
 
PRIMEROS AUXILIOS BOMBEROS 2024 actualizado
PRIMEROS AUXILIOS BOMBEROS 2024 actualizadoPRIMEROS AUXILIOS BOMBEROS 2024 actualizado
PRIMEROS AUXILIOS BOMBEROS 2024 actualizadoNestorCardona13
 
OFICIAL TABIQUE DESVIADO presentacion de desviacion del tabique por sinusitis
OFICIAL TABIQUE DESVIADO presentacion de desviacion del tabique por sinusitisOFICIAL TABIQUE DESVIADO presentacion de desviacion del tabique por sinusitis
OFICIAL TABIQUE DESVIADO presentacion de desviacion del tabique por sinusitisYeseniaChura1
 
seminario patología de los pares craneales 2024.pptx
seminario patología de los pares craneales 2024.pptxseminario patología de los pares craneales 2024.pptx
seminario patología de los pares craneales 2024.pptxScarletMedina4
 
TEXTO PRN 8VA ESPAÑOL.pdf reanimacion neonatal
TEXTO PRN 8VA ESPAÑOL.pdf reanimacion neonatalTEXTO PRN 8VA ESPAÑOL.pdf reanimacion neonatal
TEXTO PRN 8VA ESPAÑOL.pdf reanimacion neonatalJanKarlaCanaviriDelg1
 
Cuadro-comparativo-Aparato-Reproductor-Masculino-y-Femenino.pptx
Cuadro-comparativo-Aparato-Reproductor-Masculino-y-Femenino.pptxCuadro-comparativo-Aparato-Reproductor-Masculino-y-Femenino.pptx
Cuadro-comparativo-Aparato-Reproductor-Masculino-y-Femenino.pptxguadalupedejesusrios
 
Relacion final de ingresantes 23.11.2020 (2).pdf
Relacion final de ingresantes 23.11.2020 (2).pdfRelacion final de ingresantes 23.11.2020 (2).pdf
Relacion final de ingresantes 23.11.2020 (2).pdfAlvaroLeiva18
 
Hemorragia de tubo digestivo alto y bajo (1).pdf
Hemorragia de tubo digestivo alto y bajo (1).pdfHemorragia de tubo digestivo alto y bajo (1).pdf
Hemorragia de tubo digestivo alto y bajo (1).pdfELIZABETHTOVARZAPATA
 
SEGUNDA Y TERCERA SEMANA DEL DESARROLLO EMBRIONARIO.pptx
SEGUNDA  Y  TERCERA  SEMANA  DEL  DESARROLLO  EMBRIONARIO.pptxSEGUNDA  Y  TERCERA  SEMANA  DEL  DESARROLLO  EMBRIONARIO.pptx
SEGUNDA Y TERCERA SEMANA DEL DESARROLLO EMBRIONARIO.pptxArian753404
 
PRESENTACIÓN SÍNDROME GUILLAIN BARRE.pptx
PRESENTACIÓN SÍNDROME GUILLAIN BARRE.pptxPRESENTACIÓN SÍNDROME GUILLAIN BARRE.pptx
PRESENTACIÓN SÍNDROME GUILLAIN BARRE.pptxCristianOswaldoMunoz
 
Patologías de los eritrocitos-Histologia
Patologías de los eritrocitos-HistologiaPatologías de los eritrocitos-Histologia
Patologías de los eritrocitos-Histologia Estefa RM9
 
TANATOLOGIA de medicina legal y deontología
TANATOLOGIA  de medicina legal y deontologíaTANATOLOGIA  de medicina legal y deontología
TANATOLOGIA de medicina legal y deontologíaISAIDJOSUECOLQUELLUS1
 
ACRONIMO TIMERS TRATAMIENTO DE HERIDAS AVANZADAS
ACRONIMO TIMERS TRATAMIENTO DE HERIDAS AVANZADASACRONIMO TIMERS TRATAMIENTO DE HERIDAS AVANZADAS
ACRONIMO TIMERS TRATAMIENTO DE HERIDAS AVANZADASjuanjosenajerasanche
 
infografía seminario.pdf.................
infografía seminario.pdf.................infografía seminario.pdf.................
infografía seminario.pdf.................ScarletMedina4
 
patologia de robbins capitulo 4 Lesion celular.pdf
patologia de robbins capitulo 4 Lesion celular.pdfpatologia de robbins capitulo 4 Lesion celular.pdf
patologia de robbins capitulo 4 Lesion celular.pdfVilcheGuevaraKimberl
 

Recently uploaded (20)

Material de apoyo, modulo psicologia de la personalidad
Material de apoyo, modulo psicologia de la personalidadMaterial de apoyo, modulo psicologia de la personalidad
Material de apoyo, modulo psicologia de la personalidad
 
Cartilla Nacional Mexicana de 10 a 19 años
Cartilla Nacional Mexicana de 10 a 19 añosCartilla Nacional Mexicana de 10 a 19 años
Cartilla Nacional Mexicana de 10 a 19 años
 
PAM Y VACAM en el adulto mayor iestdv.pptx
PAM Y VACAM en el adulto mayor iestdv.pptxPAM Y VACAM en el adulto mayor iestdv.pptx
PAM Y VACAM en el adulto mayor iestdv.pptx
 
Psicología: Revista sobre las bases de la conducta humana.pdf
Psicología: Revista sobre las bases de la conducta humana.pdfPsicología: Revista sobre las bases de la conducta humana.pdf
Psicología: Revista sobre las bases de la conducta humana.pdf
 
Radiologia_de_Urgencias_y_Emergencias_3deg_Ed.pdf
Radiologia_de_Urgencias_y_Emergencias_3deg_Ed.pdfRadiologia_de_Urgencias_y_Emergencias_3deg_Ed.pdf
Radiologia_de_Urgencias_y_Emergencias_3deg_Ed.pdf
 
6.METODOLOGIA ATENEA MICHAEL. ZAPATA.pdf
6.METODOLOGIA ATENEA MICHAEL. ZAPATA.pdf6.METODOLOGIA ATENEA MICHAEL. ZAPATA.pdf
6.METODOLOGIA ATENEA MICHAEL. ZAPATA.pdf
 
PRIMEROS AUXILIOS BOMBEROS 2024 actualizado
PRIMEROS AUXILIOS BOMBEROS 2024 actualizadoPRIMEROS AUXILIOS BOMBEROS 2024 actualizado
PRIMEROS AUXILIOS BOMBEROS 2024 actualizado
 
OFICIAL TABIQUE DESVIADO presentacion de desviacion del tabique por sinusitis
OFICIAL TABIQUE DESVIADO presentacion de desviacion del tabique por sinusitisOFICIAL TABIQUE DESVIADO presentacion de desviacion del tabique por sinusitis
OFICIAL TABIQUE DESVIADO presentacion de desviacion del tabique por sinusitis
 
seminario patología de los pares craneales 2024.pptx
seminario patología de los pares craneales 2024.pptxseminario patología de los pares craneales 2024.pptx
seminario patología de los pares craneales 2024.pptx
 
TEXTO PRN 8VA ESPAÑOL.pdf reanimacion neonatal
TEXTO PRN 8VA ESPAÑOL.pdf reanimacion neonatalTEXTO PRN 8VA ESPAÑOL.pdf reanimacion neonatal
TEXTO PRN 8VA ESPAÑOL.pdf reanimacion neonatal
 
Cuadro-comparativo-Aparato-Reproductor-Masculino-y-Femenino.pptx
Cuadro-comparativo-Aparato-Reproductor-Masculino-y-Femenino.pptxCuadro-comparativo-Aparato-Reproductor-Masculino-y-Femenino.pptx
Cuadro-comparativo-Aparato-Reproductor-Masculino-y-Femenino.pptx
 
Relacion final de ingresantes 23.11.2020 (2).pdf
Relacion final de ingresantes 23.11.2020 (2).pdfRelacion final de ingresantes 23.11.2020 (2).pdf
Relacion final de ingresantes 23.11.2020 (2).pdf
 
Hemorragia de tubo digestivo alto y bajo (1).pdf
Hemorragia de tubo digestivo alto y bajo (1).pdfHemorragia de tubo digestivo alto y bajo (1).pdf
Hemorragia de tubo digestivo alto y bajo (1).pdf
 
SEGUNDA Y TERCERA SEMANA DEL DESARROLLO EMBRIONARIO.pptx
SEGUNDA  Y  TERCERA  SEMANA  DEL  DESARROLLO  EMBRIONARIO.pptxSEGUNDA  Y  TERCERA  SEMANA  DEL  DESARROLLO  EMBRIONARIO.pptx
SEGUNDA Y TERCERA SEMANA DEL DESARROLLO EMBRIONARIO.pptx
 
PRESENTACIÓN SÍNDROME GUILLAIN BARRE.pptx
PRESENTACIÓN SÍNDROME GUILLAIN BARRE.pptxPRESENTACIÓN SÍNDROME GUILLAIN BARRE.pptx
PRESENTACIÓN SÍNDROME GUILLAIN BARRE.pptx
 
Patologías de los eritrocitos-Histologia
Patologías de los eritrocitos-HistologiaPatologías de los eritrocitos-Histologia
Patologías de los eritrocitos-Histologia
 
TANATOLOGIA de medicina legal y deontología
TANATOLOGIA  de medicina legal y deontologíaTANATOLOGIA  de medicina legal y deontología
TANATOLOGIA de medicina legal y deontología
 
ACRONIMO TIMERS TRATAMIENTO DE HERIDAS AVANZADAS
ACRONIMO TIMERS TRATAMIENTO DE HERIDAS AVANZADASACRONIMO TIMERS TRATAMIENTO DE HERIDAS AVANZADAS
ACRONIMO TIMERS TRATAMIENTO DE HERIDAS AVANZADAS
 
infografía seminario.pdf.................
infografía seminario.pdf.................infografía seminario.pdf.................
infografía seminario.pdf.................
 
patologia de robbins capitulo 4 Lesion celular.pdf
patologia de robbins capitulo 4 Lesion celular.pdfpatologia de robbins capitulo 4 Lesion celular.pdf
patologia de robbins capitulo 4 Lesion celular.pdf
 

ENARM COMPENDIO

  • 1. COMPENDIO Dr. Alejandro Sandoval G. Travesi CMN La Raza ENARM
  • 2. Consejos 1. Leer diariamente y en bloques 2. Adiós Partys un tiempo 3. Has ejercicio y come bien durante el estudio 4. Toma algún curso bueno si tienes la posibilidad 5. Ten Fe.
  • 4. Hemoglobina Hematocrito Recién nacido 19.5 ± 5.0 54 ± 10 Mujeres 14.0 ± 2.0 42 ± 5 Hombres 16.0 ± 2.0 47 ± 7 Cifras normales de hemoglobina y hematocrito al nivel del mar (Wintrobe)
  • 6.
  • 7. QUIMICA Y ELECTROLITOS • GLUCOSA • 70- 105 • UREA • 10- 50 • ACIDO URICO • 2.5- 7.7 • NITROGENO UREICO • 6- 20 • CREATININA SERICA • 0.5- 1.2 • COLESTEROL • 120- 200 • TRIGLICERIDOS • 10- 160 • HDL • 40- 90 • LDL • 0 – 120 • VLDL • 0- 50 • ALBUMINA • 3.5- 5.5 • SODIO • 135- 145 • POTASIO • 4- 5.3 • CLORO • 98- 106
  • 8. PFH Y ENZIMAS CARDIACAS • BILIRRUBINA TOTAL • 0 – 1.5 • BILIRRUBINA INDIRECTA • 0 – 1.50 • BILIRRUBINA DIRECTA • 0 – 0.5 • AST/TGO • 6 – 38 • ALT/TGP • 0 – 35 • DHL • 100 – 190 • GGT • 7 – 50 • CPK • 0 – 226
  • 9. GASES ARTERIALES • PH • 7.35--7.45 • PCO2 • 33-45mmHg • PO2 • 75-105mmHg
  • 10. OSMOLARIDAD SERICA Y LCR • OSMOLALITY SERUM • 275-295mOsmol/Kg • LCR • CELLCOUNT • 0.5cells/mm3 • CHLORIDE • 118--132mEq/L • GAMMA GLOBULIN • 3--12% total Proteins • GLUCOSE • 40--70mg/dL • PRESSURE • 70--180mm h2O • PROTEINS,TOTAL • < 40mg/dL
  • 11. ANEMIA POR DEFICIT DE HIERRO
  • 12. ANEMIA POR DEFICIT DE HIERRO
  • 13.
  • 14. ANEMIA MEGALOBLÁSTICA # de Eritrocitos Otros estudios Aspirado de médula ósea: HIPERCELULAR bilirrubina indirecta = lig. DHL = Hemoglobina Hematocrito V.G.M. H.G. M. NC.M.H.G. Reticulocitos N o lig. Leucocitos Diferencial L= Frotis N= Plaquetas
  • 16. A que se refiere el termino POLICITEMIA ESPURIA? R = AUMENTO DEL HTO por aumento de la concentración del VOLUMEN PLASMÁTICO NO POR EL > DE LA MASA ERITROCITARIA Cual es el cc de la TROMBOSIS ESENCIAL? R = El primer signo puede ser trombosis mesentérica, hepática o portal. Eritromelalgia. Cuales son los datos de laboratorio de la trombosis esencial? 1) PLAQUETAS >2, 000,000 2) FSP CON > DE PLAQUETAS GRANDES 3) MO CON > DE MEGACARIOCITOS 4) GEN FILADELFIA AUSENTE que descarta leucemia mieloide crónica Que otras enfermedades pueden causar > plaquetario sin ser TE? R = AR, CUCI e infección crónica Cual es el manejo de la TE? R = HIDROXIUREA y ASA hasta reducir a < 500, 000 Que es la MIELOFIBROSIS? 1) Trastorno mieloproliferativo caracterizado por FIBROSIS DE LA MO, esplenomegalia y cuadro leuco eritroblastico en FSP con POIQUILOCITOS EN LAGRIMA. 2) HEMATOPOYESIS EXTRAMEDULAR
  • 17. Cual es la etiología de la mielofibrosis? R = Se desarrolla en respuesta a un > EN LA SECRECIÓN DE FACTOR DE CRECIMIENTO DERIVADO DE LAS PLAQUETAS Cual es el cc de la mielofibrosis? 1) Fatiga, distensión abdominal por esplenomegalia, DOLOR OSEO PRINCIPALMENTE RETROESTERNAL, hepatomegalia. 2) Hemorragias por secuestro de plaquetas 3) HIPERTENSIÓN PORTAL Y VARICES ESOFÁGICAS POR HEMATOPOYESIS HEPÁTICA Cuales son los datos de laboratorio y punción obtenidos en la mielofibrosis? 1) Anemia 2) FSP con POIQUILOCITOS EN LAGRIMA, formas inmaduras mieloides y PLAQUETAS GIGANTES desgranuladas 3) PUNCIÓN SECA por aumento de fibras reticulares Cual es la triada clásica de la mielofibrosis? 1) POIQUILOCITOS EN LAGRIMA 2) SANGRE LEUCO-ERITROBLASTICA 3) PLAQUETAS GIGANTES 4) Supervivencia de 3-5ª Cual es el manejo de la mielofibrosis? R = Transfusiones continuas, TETOSTERONA. TMO + talidomida Cuando se considera LEUCEMIA según la OMS? R = Cuando hay al menos 20% DE BLASTOS EN SANGRE O MO.
  • 20. Cuales son las bases para el dx de LLC? R = Es una malignidad ORIGINADA EN LOS LINFOCITOS B, en la cual cursan asintomáticos, CON INFILTRACIÓN DE LB EN LOS ÓRGANOS Cual es el cc de LLC? R = Fatiga, LINFADENOPATIA Y HEPATO-ESPLENOMEGALIA Cual es el hallazgo de laboratorio de la LLC? 1) LINFOCITOSIS AISLADA > 5000 2) MARCADOR CD 19 EN LB Y CD 5 EN LT Cual es el manejo de la LLC? R = Clorambucil Cual es la estirpe histológica de la que proviene la LEUCEMIA DE CÉLULAS VELLOSAS? R = LINFOCITOS B Cual es el cc de LCV? R = Fatiga, ESPLENOMEGALIA o asintomático
  • 21. PUNCION SECA I. MIELOFIBROSIS • Cual es la etiología de la mielofibrosis? R = Se desarrolla en respuesta a un > EN LA SECRECIÓN DE FACTOR DE CRECIMIENTO DERIVADO DE LAS PLAQUETAS • Cual es el cc de la mielofibrosis? 1) Fatiga, distensión abdominal por esplenomegalia, DOLOR OSEO PRINCIPALMENTE RETROESTERNAL, hepatomegalia. 2) Hemorragias por secuestro de plaquetas 3) HIPERTENSIÓN PORTAL Y VARICES ESOFÁGICAS POR HEMATOPOYESIS HEPÁTICA II. LEUCEMIA DE CELULAS VELLOSAS • Cual es la estirpe histológica de la que proviene la leucemia de células vellosas? R = LINFOCITOS B • Cual es el cc de LCV? R = Fatiga, ESPLENOMEGALIA o asintomático
  • 22. Que es el MIELOMA MÚLTIPLE? 1) Es una neoplasia de células plasmáticas que sustituye la MO. 2) Hay DESTRUCCIÓN ÓSEA con formación de PARAPROTEÍNAS plasmáticas IgG o IgA 3) Hay HIPERVISCOCIDAD debido a paraproteínas 4) Puede provocar IRA por las paraproteínas Cual es el cc del mieloma múltiple? 1) DOLOR ÓSEO 70% que aumenta con los movimientos, principalmente en la espalda CON EVIDENCIAS DE FRACTURAS VERTEBRALES 2) Edad > 65ª , ANEMIA, dolor óseo, infección 3) Amiloidosis manifiesta: macroglosia, neutropatia, ICC o hepatomegalia Cuales son los datos de laboratorio del MM? 1) HIPERCALCEMIA, Sx anémico con ANEMIA NORMO-NORMO por efecto supresor de eritropoyetina por IL6 2) NEFRITIS intersticial con pérdida DE CADENAS LIGERAS 3) Biopsia de MO con infiltración normal o difusa Cual es el manejo de mieloma múltiple? 1) Vincristina, doxorrubicina y dexametazona 2) TMO autologo 3) TALIDOMIDA + DEXAMETAZONA PARA RECAÍDAS 4) Dx dif. Con plasmocitoma solitario que responde bien a la radioterapia que no presenta la clínica típica del mieloma Como estadificas la carga tumoral en el MM? 1) BAJA: con espiga IgG <5 gr/dl sin hipercalcemia ni IR 2) ALTA: con espiga IgG > 7 gr, Hto <25% y calcio > 12 Que es la MACROGLOBULINEMIA DE WALDESTROM? R = Enfermedad MALIGNA DE LB, con hibrido de LB célula plasmática con PARAPROTEINA IgM
  • 23. Cuales son las causas secundarias más comunes de PTI? 1) Fármacos como la heparina 2) LES y LLC Cual es el mecanismo por el cual la heparina causa trombocitopenia? R = Auto anticuerpos IgG vs factor 4 plaquetario Cual es el manejo de la PTI? 1) PREDNISONA 1-2 mg/kg/dia con respuesta en una semana 2) INMUNOGLOBULINA iv para aumento de plaquetas 3) ESPLENECTOMÍA Que es la PURPURA TROMBOCITOPENICA TROMBOTICA? R = Es una enfermedad caracterizada por ANEMIA HEMOLÍTICA MICROANGIOHEPATICA, TROMBOCITPENIA y AUMENTO NOTABLE DE DHL SÉRICA (a diferencia de la PTI donde no hay elevación de la DHL) Cual es la etiología de la PTT? R = Deficiencia de “PROTEASA” que divide al factor de VON WILLEBRAND (VIII), por lo tanto se acumula dicho factor originando aglutinación de plaquetas en endotelio vascular. TAMBIÉN LO PUEDE DESENCADENAR EL EMBARAZO, LOS ESTRÓGENOS ELEVADOS Y LAS INFECCIONES. Cual es el cc de la PTT? 1) PENTADA CLASICA: Anemia hemolítica microangiopatica, Trombocitopenia, Afección neurológica, Falla Renal y Fiebre. ATARF 2) Fiebre, palidez, PETEQUIAS 3) Anemia, hemorragia. 4) Síntomas neurales como HEMIPARESIA QUE DURA MINUTOS
  • 24. DEFICIT DE ADHESION PLAQUETARIO I. Que es la tromboastenia de GLANZMAN? A. Es un trastorno AR que produce hemorragias ( trastorno plaquetario). LAS PLAQUETAS NO PUEDEN AGREGARSE por que no hay RECEPTORES (GLUCOPROTEINAS IIB Y IIIA) para el fibrinógeno. B. Se maneja con desmopresina I. Que es el síndrome de BERNARD-SOULTIER? A. Es un trastorno AR que OCASIONA HEMORRAGIAS. Las PLAQUETAS NO SE ADHIEREN por FALTA DEL RECEPTOR IIB para el factor de vWF B. Se maneja con transfusion de plaquetas
  • 25. A que se debe la HEMOFILIA TIPO C y cual es su principal cc? R = Déficit del FACTOR XI y se manifiesta con HEMORRAGIAS LEVES. Que es la HEMOFILIA TIPO A? R = Es un trastorno hereditario AR LIGADO AL X con déficit del FACTOR VIII y SOLO AFECTA A VARONES Cual es el cc de la hemofilia tipo A? R = HEMORRAGIA EN ARTICULACIONES de rodilla, tobillo, codo, músculos y aparato GI Cuales son los datos de lab en la hemofilia tipo A? 1) TPT SE PROLONGA únicamente 2) FACTOR VIII DISMINUIDO Y VWF NORMAL Cual es el manejo de la hemofilia tipo A? R = Concentrados de FACTOR VIII 4000 UI para 70 kg Como puede transmitirse la enfermedad de anticuerpos vs el factor VIII? R = Puede desarrollarse POST PARTO o sin una causa, hemofilia A que han recibido concentrados plasmáticos.
  • 26. Cual es el cc de la enfermedad de anticuerpos vs el factor VIII? R = Hemorragia grave Cual es el manejo de elección en la enfermedad de anticuerpos vs el factor VIII? R = CICLOFOSFAMIDA Que es la HEMOFILIA TIPO B o enfermedad de Christmas? R = Trastorno AR LIGADO AL X con déficit de FACTOR IX en el cual se eleva el TPT Cual es el manejo de la hemofilia tipo B o enfermedad de Christmas? R = Concentrados de FACTOR IX 6000 para 70 kg Que factores de coagulación se ven afectados en COAGULOPATIA POR ENFERMEDAD HEPÁTICA? 1) Se fabrican todos los factores de coagulación en el hígado EXCEPTO EL FACTOR VIII 2) Primero se ven afectados los vitamina K dependientes II, VII, IX y X y el factor V 3) Aumenta la fibrinólisis por la disminución de plasmina Cual es el manejo de la coagulopatia por enfermedad hepática? R = PLASMA FRESCO CONGELADO
  • 27. Cual es el cc de la coagulación intravascular diseminada? 1) HEMORRAGIAS Y TROMBOSIS 2) Hemorragia +++ por punción, heridas o espontanea 3) Trombosis + como isquemia digital o gangrena Que es el SÍNDROME DE TROSEAU en la coagulación intravascular diseminada? R = Coagulación intravascular diseminada manifiesta como TROMBOSIS VENOSA SUPERFICIAL Y PROFUNDA POR CÁNCER principalmente de manera recurrente. Cuales son los datos de lab para la coagulación intravascular diseminada? 1) HIPOFIBRINOLEMIA 2) Aumento de productos de degradación de la fibrina, TROMBOCITOPENIA y AUMENTO DEL TP Y TPT 3) DÍMERO D como producto de degradación de la fibrina que aumenta si hay fallo hepático. Cual es el manejo para la coagulación intravascular diseminada? 1) TRATAR EL TRASTORNO SUBYACENTE 2) Heparina, S- AMINOCAPROICO (AUMENTA EL FIBRINÓGENO) 3) El ÉXITO del tx consiste en que AUMENTE EL FIBRINÓGENO 4) CRIOPRECIPITADOS PARA FIBRINÓGENO Como se diagnostica la disfibrinolemia? R = Aumento del TIEMPO DE REPTILASA Que medicamento se utiliza de elección en los estados de hipercoagubilidad? R = WARFARINA
  • 28. PATOLOGIA CELULA PATOGNOMONICA ANEMIA DE CELULAS FALCIFORMES Cuerpos de Howell-Jolly:Estructuras únicas o dobles, pequeñas y redondas. Se observan como gránulos densos y de color azul rojizo o violeta. Localizados excéntricamente DEFICIT DE 6 GPDH Cuerpos de Heinz: Citoplasma de los glóbulos rojos , aparecen como pequeños puntos oscuros bajo el microscopio LINFOMA DE HODKING Reed Stemberg cells: Celula B modificada por lo general es muy grande dando la pariencia de ojos de Buho con nucléolos prominentes en forma de inclusiones, la variedad mononuclear tiene un solo nucleo y un nucléolo prominente. LEUCEMIA AGUDA Bastones de Auer: Son estructuras filiformes dentro de los blastos ANEMIA SIDEROBLASTICA Cuerpos de Papenheimer: Son acúmulos de hemosiderina unida a proteínas. Consisten en gránulos basófilos, con las tinciones habituales, que además, se tiñen también de azul con el colorante de Perls (azul de Prusia).
  • 29. PATOLOGIA CELULA PATOGNOMONICA INTOXICACION POR PLOMO, LEUCEMIA Y TALASEMIA. Punteado basofilo: Pueden ser agregados ribosómicos originados por una degeneración vacuolar del citoplasma o precipitados de cadenas globínicas libres. Consiste en puntitos basófilos, con las tinciones habituales, de tamaño variable y dispersos por toda la superficie del hematíe. ANEMIA MEGALOBLASTICA Anillos de cabot: Están formados por restos de la membrana nuclear o de microtúbulos. Consisten en una especie de hilos basófilos, con las tinciones habituales, que adoptan una forma de anillo o de ocho y que pueden ocupar toda la periferia celular. CÉLULA WARTHIN-FINKELDEY Sarampión: Célula gigante multinucleótica con citoplasma eosinofílico e inclusiones nucleares
  • 30. PATOLOGIA • De que patología son característicos los CUERPOS DE HEINZ? R = Déficit de glucosa 6PDH • De que patología son patognomónicos los cuerpos de HOWELL-JOLLY? R = Anemia de células falciformes • De que patología son patognomónicos los BASTONES DE AUER? R = Leucemia mieloide aguda en la que además la distingue la presencia de “MIELOPEROXIDASA” • Que es la LEUCEMIA MIELOGENA CRÓNICA? R = Se caracteriza por SOBREPRODUCCIÓN DE CÉLULAS MIELOIDES (principalmente LEUCOCITOS), presenta EL COMOSOMA FILADELFIA (translocacion de los brazos 9 y 22) BCR/ABL que produce “TIROCINA CINASA”. • Que diferencia la LEUCEMIA MIELOIDE AGUDA de la leucemia linfoblastica aguda? R = Mieloperoxidasa que no se produce en la LLA • Cual es el hallazgo patognomónico de linfoma de Hodking? R = RED STEMBERG CELLS (CÉLULA B MODIFICADA por lo general es muy grande dando la apariencia de ojos de Búho con nucléolos prominentes en forma de inclusiones, la variedad mononuclear tiene un solo núcleo y un nucléolo prominente). • Que patología te da trombos hialinos en la biopsia? R = PTT y SUH • Citogenetica reconocible con T (18:14) de que enfermedad es patognomónico? R = Linfoma no Hodking
  • 31. ENF. DE HODKING • Característica de la Enfermedad de Hodgkin? • Linfadenopatia indolora, Síntomas constitucionales, no se conoce su origen, diseminación ordenada • Cuadro clínico? • MASA INDOLORA (CUELLO), mas frecuente en varones, diseminación ordenada, fiebre de larga evolución, DISMINUCIÓN DE PESO, PRURITO INTENSO, sudación nocturna, DOLOR GANGLIONAR (ALCOHOL) • Diagnostico definitivo? • Biopsia del ganglio linfático afectado, CÉLULAS DE REED-STENBERG, no hay anormalidades cariotipicas • El virus Epstein Barr con que enfermedad tumoral se ha relacionado? • Enfermedad de Hodgking • En la Enfermedad de Hodgking cual es la variedad histológica mas frecuente? • ESCLEROSIS NODULAR
  • 32. • En la Enfermedad de Hodgking cual es la variedad de mejor pronostico? • VARIEDAD LINFOCITICA • En la Enfermedad de Hodgking cual es la variedad menos frecuente? • Depleción linfocitaria • En la Enfermedad de Hodgking que se relaciona con VIH? • DEPLECIÓN LINFOCITARIA • Tratamiento de la de la Enfermedad de Hodgking? 1) MOPP (mecloretamina, Vincristina, procarbacina, prednisona) 2) ABVD (adriamicina, bleomicina, Vincristina, Dacarbacina) • Clasificación de ann-arbor? • Etapa 1 una sola cadena ganglionar • Etapa 2 2 regiones ganglionares (mismo lado del diafragma) • Etapa 3 regiones ganglionares en ambos lados del diafragma • Etapa 4 metástasis • (A asintomático, B síntomas constitucionales)
  • 33. ENF. NO HODKING • Característica de la Enfermedad no Hodgking? • No tiene cél de Reed Stemberg, si hay citogenética reconocible T (8:14), correlación con proto- oncogen C-myc • Cuadro clínico de la E no H? • Linfadenopatia desordenada, a menudo diseminada al momento del diagnostico, síntomas constitucionales • En que consiste el termino de ganglio de Richter? 1) Es un ganglio generalmente SUPRACLAVICULAR . 2) GANGLIO que comparte una LLC q posteriormente se convertirá (EVOLUCIONARA) en LINFOMA NO HODGKIN • Tratamiento de los linfomas no Hodgkin? 1) 1 solo ganglio afectado: radioterapia local … si esta asintomático valorar conducta expectante 2) Tx: clorambucilo o CVP+fludarabina, CHOP • Tratamiento para un MALTOMA? • Terapia para HELICOBACTER PYLORI
  • 34. LEUCEMIA AGUDA • Cáncer mas frecuente en niños?  Leucemia siendo la aguda la mas frecuente (80%) • El cáncer de los ganglios linfáticos se llama?  Linfoma • El cáncer de medula ósea?  Leucemia • Característica de la leucemia Linfoblastica Aguda?  Tienen blastos (20% en MO y 90% en sangre periférica) y su caract. Es la capacidad de infiltrar órganos (encías, pericardio, testículos, MO, SNC etc), 2da. Capacidad de producir Citopenias o Pancitopenias
  • 35. • Cuadro Clínico de LLA?  Por procesos de infiltración hay: Hemorragias (petequias, equimosis) Adenopatías, Hepato-esplenomegalias, puede llegar a una CID • Diferencia entre LLA y la hipoplasia medular y purpura trombocitopenica idiopática  En que puede haber trombocitopenia pero TANTO EN LA HIPOPLASIA COMO EN LA PURPURA NUNCA habrá ganglios linfáticos palpables o hepatoesplenomegalias • Características de la CID?  Tiempo de protrombina alargado, dímero D presente, trombocitopenia. • Que tipo de leucemia mielocitica aguda tiene la característica de presentarse como una CID  La promielocitica y la monocitica aguda (habitualmente son de buen pronostico mientras que no se presente con CID)
  • 36. • Citogeneticas desfavorables?  Monosomia 5 y 7, Cromosoma Ph t(4;11) (Encontrar Cromosoma ph en LMA(2%) es de muy mal pronostico habitualmente se observa en LMC (98%) • Con respecto a marcadores de superficie:  Determinación de una Enzima desoxinucleotidil transferasa terminal….. es una LLA (95%) • Si es MIELOPEROXIDASA POSITIVA…. LMA  Marcador CD19, CD10…. Es una LLA de células B (mas frecuente)  Marcador CD2, CD5, CD7…. Es una LLA de células T (menos frecuente) • Tratamiento de Leucemias Mielocitica Agudas?  1ra etapa: Inducción de la Remisión Daunorrubicina o Idarrubicina mas citarabina  2da etapa: Quimioterapia intensa de repetición mas transplante de MO autologa • Tratamiento de la Leucemia Linfoblastica aguda?  1ra etapa: Inducción de la Remisión Vincristina, Prednisona, Daunorrubicina, L Asparaginasa (4 medicamentos) Metrotexate (profilaxis del SNC)  2da etapa: Quimioterapia a dosis altas mas transplante de MO
  • 37. • Leucocitosis con Desviación a la DERECHA:  Cuando los LINFOCITOS son los que están elevados (arriba del 20%) • Leucocitosis con Desviación a la IZQUIERDA:  Cuando los NEUTRÓFILOS están elevados • Característica Histológicas de LMA?  Mieloperoxidasas positivas, cuerpos de AUER en citoplasma • Características Histológicas de LLA?  Antígeno CALLA (Antígeno común de LLA) y PAS Acido periódico de Shift positivo • Citogenética Favorables?  En px con LMA t(8:21), t(15:17),inv (cr16), Inv (p13; q22) buen pronostico
  • 38. LEUCEMIA CRONICA • Característica de una LLC? 1) Linfocitos mayores de 5000, linfocitos de apariencia normal (madura), con expresión de CD19, y CD5 2) Origen: malignidad de linfocitos B (95%) inmunoincopetentes • Cuadro clínico de una LLC?  Se presenta en px MAYORES DE 50 AÑOS (90%), linfocitosis impresionante, linfadenopatia y Hepatoesplenomegalia • Clasificación de la LLC (CLASIFICACION RAI)  ETAPA O Linfocitosis mas de 5000  ETAPA 1 Linfocitosis MAS LINFADENOPATIA (revisar al px)  ETAPA 2 Linfocitosis MAS ORGANOMEGALIA HEPATOESPLENOMEGALIA  ETAPA 3 Linfocitosis mas anemia  Etapa 4 Linfocitosis , lo anterior mas trombocitopenia (mal pronostico) • Laboratorios de una LLC?  LINFOCITOSIS AISLADA (mas de 5000), Leucos mayor de 20000, inmunogenotipo: CD19 (LLC de Celulas B), CD5 (LLC de Celulas T)
  • 39. • Tratamiento de LLC con sintomatología?  CLORAMBUCILO alternativa fludarabina en px jóvenes • Característica de LMC? 1) Leucocitosis, Cromosoma Ph t(9:22) presentes en un 95%, 2) Leucocitosis con desviación a la izquierda (aumento delos neutrófilos) • Tx de LMC?  EL TX no es urgente Mesilato de Imatinib, alternativa Hidroxiurea o interferon alfa • Cuadro clínico de LMC?  Crecimiento impresionante de bazo, mas leucocitosis con desviación a la izq con serie mieloide, sensibilidad esternal, cromosoma Ph (95%)
  • 40. SINDROME PARANEOPLASICO/ AC • Frecuentemente presentan pleocitosis linfocitaria en LCR con aumento de proteínas y de IgG. Algunos pacientes presentan anticuerpos característicos en suero y LCR que proveen una fuerte evidencia de que el cuadro neurológico es paraneoplásico y que además sugieren el tipo de cáncer asociado, por ejemplo el anticuerpo anti- Yo se asocia con degeneración cerebelosa y cáncer ginecológico, el anticuerpo anti-Hu con encéfalomielitis paraneoplásica y cáncer pulmonar de células pequeñas y el anticuerpo anti-Ki con opsocionus y cáncer de mama
  • 41. MICELANEAS • En un paciente con anemia ferropénica que esperaría encontrar en su estudio de laboratorio? • Ferritina, ferremia y saturación de transferrina bajas y transferrina alta. • Por cuanto tiempo se realiza el tratamiento de la anemia ferropenica? • R = 6 meses
  • 43. EKG
  • 44. EKG  I N T E R V A L O S  PR: Se denomina así, al espacio que va del inicio de la Onda P al comienzo de la Onda R. Valor normal: 120 a 200 milisegundos ( 0.12 – 0.20 segundos).  COMPLEJO QRS: Corresponde a la activación del miocardio ventricular. Valor Normal: 60 a 100 milisegundos ( 0.06 – 0.10 segundos).  QT: Se mide desde el inicio del Complejo QRS hasta el final de la Onda T y corresponde a la duración total de la Sístole Ventricular. Valor normal: 240 a 480 milisegundos ( 0.24 – 0.48 segundos ). ( Varia de acuerdo a la frecuencia cardiaca )  S E G M E N T O  ST: Es el intervalo normalmente iso-electrico entre el final del Complejo QRS y el inicio de la Onda T. Valor normal: 60 a 160 milisegundos ( 0.06 a 0.16 segundos )
  • 45. EKG • I N T E R V A L O Q T  P r o l o n g a d o:  C a r d i o p a t í a I s q u é m i c a.  I. C. C.  M i o c a r d i t i s.  Drogas: Quinidina, Amiodarona, Antidepresivos triciclicos...  H i p o m a g n e s e m i a.  H i p o c a l c e m i a.  H i p o k a l e m i a ?.  A c o r t a d o:  R e p o l a r i z a c i ó n p r e c o z.  D r o g a s: Digital...  H i p e r c a l c e m i a.  H i p e r k a l e m i a.
  • 46. HIPOKALEMIA • < 3 mEq/L: onda T plana, depresión ST, ondas U • < 2,5 mEq/L: onda U prominente, inversión onda T, PR y QT prolongado, QRS ensanchado
  • 47. HIPERKALEMIA : POTASIO SÉRICO > 5 mEq/L LEVE 5.0 - 5.5 mEq/L MODERADA 5.5 - 6.0 mEq/L SEVERA > 6.0 mEq/L
  • 48. K+ 5.0 - 6.5 mEq/L Cambios mínimos K+ 6.5 - 8.0 mEq/L Onda T picuda Aplanamiento y desaparición de la onda P Depresión del segmento ST K+ 8.0 a más Ensanchamiento del QRS BAV Arritmias cardíacas Ritmo idioventricular Paro cardíaco en asistolia. HIPERPOTASEMIA Y EKG
  • 50. PULSO VENOSO YUGULAR AXVAYC • Se distinguen fundamentalmente dos ondas, la "a" y la "v". La primera, la onda "a", ocurre justo antes del sístole, y se debe a la CONTRACCIÓN DE LA AURÍCULA DERECHA (al final del diástole, cuando se termina de vaciar al ventrículo derecho). El colapso de la vena después de la onda "a", es el descenso "x" y se debe a la RELAJACIÓN DE LA AURÍCULA. AX • La onda "v" se debe al LLENE PASIVO DE LA AURÍCULA DERECHA debido al retorno venoso normal, mientras la VÁLVULA TRICÚSPIDE PERMANECE CERRADA durante el sístole. Por lo tanto, es una onda que ocurre al mismo tiempo del sístole y que se vería sobre el vena yugular. El colapso que se observa después de la onda "v", se denomina el descenso "y", que corresponde al PASO DE LA SANGRE DE LA AURÍCULA AL VENTRÍCULO DURANTE EL DIÁSTOLE, después que se abre la válvula tricúspide. VAY • Con registros muy finos, se describe una pequeña muesca ubicada en el descenso de la onda "a", que se ha llamado la onda "c", ATRIBUIDA AL CIERRE DE LA VÁLVULA TRICÚSPIDE, después que se ha terminado de contraer la aurícula derecha y está comenzando el sístole, pero no es posible de ver a simple vista.
  • 51.
  • 52.
  • 54.
  • 56. ANGINA ESTABLE • Es la más frecuente, aparece con el esfuerzo y remite espontáneamente con el reposo y/o la medicación. Posee una duración de pocos minutos y presenta un patrón regular, por lo que el paciente puede ser capaz de identificarla e incluso predecir su aparición. Su origen se halla primordialmente en una arteriopatía aterosclerótica que causa la progresiva reducción de la luz vascular, de uno o varios vasos coronarios, en porcentajes del orden del 70% o superior.
  • 57.
  • 58. ANGINA INESTABLE • La angina inestable no se relaciona con un mayor trabajo cardíaco, es decir no deriva de un mayor consumo miocárdico de oxígeno. Su causa debe buscarse en una disminución aguda del flujo cardíaco coronario, que puede deberse a la complicación de una placa aterosclerótica coronaria por erosión, fisura o rotura y trombosis sobreañadida que cause una interrupción súbita del flujo coronario o por causas extrínsecas al árbol coronario que produzcan inestabilización. Su sintomatología clínica es muy similar a la que registra el infarto agudo de miocardio, sin embargo, en la angina inestable no se produce necrosis miocárdica. • La angina inestable incluye diversos tipos de anginas caracterizadas por su evolución imprevisible, aunque no siempre fatal y que se apartan claramente del patrón típico de angina estable: angina de reciente comienzo, angina progresiva, angina de reposo, angina prolongada, angina vasoespástica o de Prinzmetal y angina postinfarto, todas ellas consideradas urgencias médicas.
  • 59. IAM
  • 60. IAM
  • 61. IAM
  • 62. SICA
  • 63. CORONARIAS INVOLUCRADAS EN IAM DERIVACION CON ELEVACION DEL ST TIPO DE INFARTO ARTERIA CORONARIA RESPONSIBLE COMPLICACIONES V1-V2 Septal Descendente anterior (ramos septales) Bloqueos de rama V3-V4 Pared anterior Descendente anterior (ramos diagonales) Disfuncion VI, IC, Bloqueo de rama V5-V6 Lateral alto Circunfleja Hipotension (evitar nitroglicerina o morfina) DII, DIII y aVF Posteroinferior con extension al ventriculo derecho Derecha (ramos proximales) Hipotension V1-V4 Posterior Circunfleja o descendente posterior Disfuncion VI
  • 64.
  • 65. PREGUNTAS Cual es la triada clásica del infarto del ventrículo derecho? Hipotensión, campos pulmonares limpios y elevación de la PVY Cual es el tratamiento de la taquicardia auricular multifocal? Suele asociarse a enfermedad pulmonar grave, mejora con la ventilación mecánica y la oxigenación
  • 68.
  • 69. RESUMEN SOPLOS • ESTENOSIS MITRAL; SINTOMAS Y SOPLO  DISNEA de esfuerzo, ORTOPNEA, EAP, disfonía por compresión del NLR. Imagen de 4 arcos en Rx por congestion venocapilar.  Soplo diastólico • INSUFICIENCIA MITRAL; SINTOMAS Y SOPLO  DISNEA de esfuerzo y fatiga, ORTOPNEA, DISNEA PAROXÍSTICA NOCTURNA. Por prolapso valvular dolor torácico (signo de Barlow) dx dif con SICA.  Soplo sistólico • ESTENOSIS AORTICA; SINTOMAS Y SOPLO  Triada clásica, ANGINA DE PECHO - IC – SINCOPE. Palpitaciones, visión borrosa. HV.  Soplo sistólico • INSUFICIENCIA AORTICA; SINTOMAS Y SOPLO  Disnea que va de esfuerzo a DISNEA PAROXÍSTICA NOCTURNA, estertores.  Soplo diastólico.
  • 70. BRI
  • 71. BRD
  • 72. B AV I • Prolongacion del intervalo PR en forma continua • >20” adultos • >16” en infantes
  • 73. B AV II/ M1
  • 78. FA
  • 79.
  • 86.
  • 88.
  • 89.
  • 90. Endocarditis infecciosa A. Amoxicilina 2gr 1 hora antes B. Ampicilina 2gr IM 30 minutos antes (en intolerancia oral) Alergia a penicilina 1) Claritromicina 500 mg 1 hora antes 2) Clindamicina 600 mg 1 hora antes 3) Cefalexina 2gr 1 hora antes Profilaxis antibiótica en procedimientos dentales, cavidad oral, respiratorio y esófago
  • 91. Endocarditis infecciosa • Ampicilina 2 gr IM o IV + Gentamicina 1,5mg/Kg/ 30 minutos antes, 6 h después Ampicilina 1gr IM/IV o Amoxicilina oral 1gr Alérgicos a penicilina • Vancomicina 1gr IV en 1-2 h + Gentamicina 1,5 mg/Kg IV/IM terminando la perfusión 30 minutos después del procedimiento Profilaxis antibiótica en procedimientos genitourinarios y gastrointestinales
  • 96. JOVEN + DOLOR PRECORDIAL, FIEBRE Y ST (LESIÓN) EXTENSA = PERICARDITIS AGUDA. LO EXACERBA LA RESPIRACIÓN. ES FRECUENTE ANTECEDENTE DE IRAS VIRALES DE NO CONOCERSE LA ETIOLOGÍA SE DAN AINES (AAS)
  • 97. PERICARDITIS • El supradesnivel del ST en la pericarditis se diferencia del infarto agudo del miocardio porque en éste el supradesnivel es convexo y más localizado, pueden coexistir ondas T negativas al mismo tiempo del supradesnivel, presencia de ondas Q cuando es un infarto Q y el EKG no normaliza antes de hacerse negativa la onda T. • De la angina de Prinzmetal se distingue porque el supradesnivel es transitorio y sólo durante el dolor; además, compromete derivaciones más localizadas. • La imagen de la repolarización precoz es parecida pero nunca hay depresión del PR, no evoluciona y no hay síntomas.
  • 99. Adulto + anasarca + complejo bajos = pericarditis constrictiva
  • 100. Rx de mujer , imagen de hiperflujo pulmonar, ECG con BRD y eje a la derecha= Comunicación Interauricular (OS)
  • 102. CIV
  • 103. CIA
  • 110. HAS
  • 111. HAS
  • 112. HAS
  • 113. HAS
  • 114.
  • 115. • Cual es el signo electrocardiografico mas común en tromboembolismo pulmonar? • R = Taquicardia sinusal. • -A 42-year-old man develops shortness of breath (SOB) and chest pain 7 days after an open cholecystectomy. His blood pressure is 145/86 mm Hg, pulse is 120/min, respirations 24/min, and oxygen saturation of 97%. Pulmonary embolism is clinically suspected. Which of the following is the most common ECG finding of pulmonary embolism? • (A) a deep S wave in lead I • (B) depressed ST segments in leads I and II • (C) prominent Q wave in lead I, and inversion of T wave in lead III • (D) sinus tachycardia • (E) clockwise rotation in the precordial leads • Correcta D • Cuales son los síntomas de estenosis mitral? • R = DISNEA, ortopnea y disnea paroxística nocturna • Como es el soplo en insuficiencia mitral? • R = PanSISTOLICO en vértice e irradiado a axila. • -A 25-year-old woman is found to have a midsystolic murmur on routine evaluation. The murmur does not radiate but it does increase with standing. She otherwise feels well and the rest of the examination is normal. • R = Mitral valve prolapsed. • -A 65-year-old man with a previous history of an anterior MI comes for follow-up. On examination, he has a systolic murmur heard best at the apex and radiating to the axilla. Transient external compression of both arms with blood pressure cuffs 20 mm Hg over peak systolic pressure increases the murmur. • R = Regurgitación mitral
  • 116. • Que caracteriza al signo de MUSSET y en que patología se presenta? • R = Hay movimientos de la cabeza en cada pulsación debido a INSUFICIENCIA DE LA VÁLVULA AORTICA. • -A 75-year-old man is bought to the hospital because of a syncopal episode. There was no incontinence or post-event confusion. On examination, his blood pressure is 140/80 mm Hg, pulse 72/min with no postural changes. His second heart sound is diminished and there is a systolic ejection murmur that radiates to the carotids. With the Valsalva maneuver, the murmur decreases in length and intensity. • R = Aortic stenosis. • En que patología se escucha el soplo de AUSTIN FLINT? • Insuficiencia aortica • Que sucede cuando hay regurgitación aortica en forma aguda? • Insuficiencia ventricular izquierda manifestada como EDEMA AGUDO PULMONAR. • -A 32-year-old asymptomatic woman has a rapidly rising, forceful pulse that collapses quickly. Which of the following is the most likely diagnosis? • (A) mitral stenosis • (B) mitral regurgitation • (C) aortic stenosis • (D) aortic regurgitation • Respuesta correcta D • Que medicamento es de elección en el síndrome de Marfan con regurgitación aortica? • R = BETABLOQUEADORES • Que medicamentos pueden disminuir la dosis de homocisteina? • R = Acido fólico, B6 y B12. • Cuál es el fenómeno de hibernación y aturdimiento miocardico? • R = Son las áreas del miocardio que se encuentran subperfundidas que se adaptan para ser viables con disfunción contráctil sostenida. • Que es el síndrome de TIETZE? • R = Inflamación de uniones condrocostales tumefactas, rojas que causa dolor retroesternal • Que antianginosos han demostrado prolongar la vida? • R = Los BETABLOQUEADORES a excepción de el pindolol que exacerba la angina
  • 117. RESUMEN SOPLOS • ESTENOSIS MITRAL; SINTOMAS Y SOPLO  Disnea de esfuerzo, ortopnea, EAP, disfonía por compresión del NLR. Imagen de 4 arcos en Rx por congestión venocapilar.  Soplo diastólico • INSUFICIENCIA MITRAL; SINTOMAS Y SOPLO  Disnea de esfuerzo y fatiga, ortopnea, disnea paroxística nocturna. Por prolapso valvular dolor torácico (signo de Barlow) dx dif con SICA.  Soplo sistólico • ESTENOSIS AORTICA; SINTOMAS Y SOPLO  Triada clásica, angina de pecho - IC – Sincope. Palpitaciones, visión borrosa. HV. Signo de Musset.  Soplo sistólico • INSUFICIENCIA AORTICA; SINTOMAS Y SOPLO • Disnea que va de esfuerzo a disnea paroxística nocturna, estertores. PULSO DE CORRIGAN , QUINQUE Y DUROZEIZ . Soplo de Austin Flint.  Soplo diastólico.
  • 118. • En cuanto tiempo tiene que estar el TTP cuando se da tratamiento anticoagulante? • R = 50-70 segundos. • En que pacientes se utiliza la profilaxis antiarrítmica? • R = LATIDOS ECTÓPICOS FRECUENTES y taquicardia ventricular. • Como se trata una fibrilación ventricular que no responde a una cardioversión? • R = Amiodarona y RCP • Como se dx infarto del ventrículo derecho? • R = Con la ELEVACIÓN DEL ST DEL HEMITORAX DERECHO • -A 69-year-old woman complains of some atypical chest pain 2 days prior to presentation. On examination, the JVP is at 8 cm, positive Kussmaul’s sign, and normal heart sounds. The lungs are clear. The ECG is abnormal, and the CXR shows a normal cardiac silhouette. • Que tan frecuente se presenta un AUNERISMA VENTRICULAR IZQUIERDO POST IAM? • R = 20% y se da el dx por medio de la ELEVACIÓN PERSISTENTE DEL ST DURANTE 4-8 SEMANAS. • -Three months after an anterior MI, a 73-yearold man has a follow-up ECG. He is clinically feeling well with no further angina symptoms. His ECG shows Q waves in the anterior leads with persistant ST-segment elevation. The current ECG is most compatible with which of the following diagnosis? • (A) ventricular aneurysm • (B) hibernating myocardium • (C) acute infarction • (D) silent infarction • (E) early repolarization
  • 119. • Que caracteriza al SÍNDROME DE DRESSLER? • R = SE PRESENTA POST IAM, debido a un FENÓMENO AUTO INMUNITARIO CON PERICARDITIS, FIEBRE, LEUCOCITOSIS y ocasionalmente derrame pericardico o pleural. • -A 67-year-old man presents with an anterior myocardial infarction (MI) and receives thrombolytic therapy. Three days later, he develops chest pain that is exacerbated by lying down, and his physical findings are normal except for a friction rub. His ECG shows evolving changes from the anterior infarction but new PR-segment depression and 1-mm ST-segment elevation in all the limb leads. Which of the following is the most likely diagnosis? • (A) reinfarction • (B) pulmonary embolus • (C) viral infection • (D) post-MI pericariditis • (E) dissecting aneurysm • Respuesta correcta D • Cual es el manejo del infarto agudo del VD? • R = CARGA LIQUIDA para mejorar el llenado ventricular derecho e INOTRÓPICOS. • Cual es el mecanismo de acción de los antiarrítmicos clase Ia y cuales son? • R = BLOQUEADORES DE LOS CANALES DE SODIO. Procainamida, quinidina y disopiramida. • Cual es el mecanismo de acción de los antiarrítmicos clase Ib y cuales son? • R = BLOQUEADORES DE LOS CANALES DE SODIO. Lidocaina y difenilhidantoina. • Cual es el mecanismo de acción de los antiarrítmicos clase II y cuales son? • R = BETABLOQUEADORES, retardando la conducción AV. Propanolol, metoprolol y esmolol.
  • 120. • Cual es el mecanismo de acción de los antiarrítmicos clase III y cuales son? • R = PROLONGAN EL POTENCIAL DE ACCIÓN. Amiodarona, sotalol, dofelitida. • Cual es el mecanismo de acción de los antiarrítmicos clase IV y cuales son? • R = BLOQUEADORES DE LOS CANALES LENTOS DE CALCIO. Verapamil, diltiazem, digoxina y adenosina. • Con que fenómenos se relaciona la taquicardia paroxística supraventricular, la cual suele cursar asintomática. • R = Intoxicación por digitalicos, bloqueos AV y fenómeno de reentrada. • Que tratamiento se utiliza en taquicardia paroxística supraventricular en caso de estar contraindicados los antiarrítmicos clase IV y que ha demostrado 100% de éxito? • R = Cardioversion. • Cual es el fármaco de elección en caso de prevención de ataques de taquicardia paroxística supraventricular? • R = DIGOXINA • Que caracteriza al síndrome de LOWN-GANONG-LEVINE? • R = INTERVALO PR CORTO Y MORFOLOGÍA NORMAL DE QRS • En que casos no necesita tratamiento las taquicardias supraventriculares causadas por vías accesorias o síndrome de preexitacion? • R = En casos de no presentar palpitaciones, mareos o sincope. • Cual es el procedimiento de elección en pacientes con síndrome de preexitacion? • R = ABLACIÓN POR RADIOFRECUENCIA • Que agentes farmacológicos deben evitarse en el síndrome de preexitacion? • R = Digoxina, betabloqueador y bloqueador de los canales de calcio. • Cual es el tratamiento a largo plazo de el síndrome de preexitacion? • R = Procainamida, verapamil y digoxina.
  • 121. • Cuales son los fármacos de elección en extrasístoles ventriculares sintomáticas? • R = BETABLOQUEADORES. • Que medida se utiliza cuando la taquicardia ventricular es recurrente? • R = Marcapasos • - A22-year-old woman complains of palpitations and has a regular heartbeat at a rate of 170/min, with a blood pressure of 110/70 mm Hg. The rate abruptly changes to 75/min after applying carotid sinus pressure. Which of the following is the most likely diagnosis? • (A) sinus tachycardia • (B) paroxysmal atrial fibrillation • (C) paroxysmal atrial flutter • (D) paroxysmal supraventricular tachycardia (PSVT) • Respuesta correcta D • Cual es el mecanismo de elección para la prevención de muerte súbita en un paciente con factores de riesgo? • R = Desfibrilador implantado • Que caracteriza al síndrome QT PROLONGADO? • R = SINCOPE RECURRENTE, arritmias ventriculares y muerte súbita. Es una ANOMALÍA CONGÉNITA hereditaria que afecta los CANALES DE Na y K. • Cual es la terapéutica mas eficaz en el síndrome de QT prolongado? • R = BETABLOQUEADORES Y DESFIBRILADOR IMPLANTADO. • Que hallazgo EKG caracteriza al bloqueo AV tipo I? • R = PROLONGACIÓN DEL PR > .20” EN ADULTOS Y > .16”. LA ALTERACIÓN ES NODAL. • Que hallazgo EKG caracteriza al bloqueo AV tipo II? 1) - 1er grado o fenómeno de WENCKEBACH, MOBITZ I: prolongación progresiva del PR hasta que 1 onda P no conduce. 2) - 2do grado o MOBITZ II: intervalo PR, sin embrago hay onda P que no conduce con un patrón 2:1 o 3:1 • En que casos se diagnostica el síndrome de SENO ENFERMO? • R = PARO SINUSAL, bloqueo sinoauricular, BRADICARDIA SINUSAL PERSISTENTE o bradiarritmias.
  • 122. • Cual es el manejo del síndrome de seno enfermo? • R = TEOFILINA ORAL Y MARCAPASOS. • Cual es la indicación terapéutica del Mobitz III? • R = MARCAPASOS • Cuales son los síntomas inadvertidos de ICC? • R = Tos crónica, nicturia • Que medicamento se utiliza en caso de intoxicación por digoxina? • R = Anticuerpos FAP • Cual es el cuadro clínico característico de alguien con ICC con edema agudo pulmonar? • R = DISNEA, tos, ESPUTO ROSADO, diaforesis, cianosis. • En que consiste el fenómeno de reentrada? • R = Involucra un circuito que conduce de forma anterograda al ventrículo y de forma retrograda a la aurícula utilizando vías accesorias. • Que tratamiento se utiliza en ICC después de que los diuréticos y nitratos no mejoran la sintomatología? • R = NISERITIDA, forma recombinante de péptido atrial natriuretico del cerebro humano. • Que agente antihipertensivo se debe evitar en ICC? • R = CALCIOANTAGONISTAS • Como es típicamente la Rx de torax en edema agudo pulmonar con ICC? • R = PATRÓN EN MARIPOSA en la distribución del edema alveolar.
  • 123. • En el EAP la presión capilar en cuna se encuentra siempre elevada, a que cantidad aproximadamente? • R = > 25 mm Hg • La morfina es eficaz en el manejo de derrame pleural cardiogenico, debido a que mecanismo? • R = AUMENTA LA CAPACITANCIA VENOSA, disminuye la presión auricular izquierda y disminuye la ansiedad. • Cuál es la diferencia entre cardioversión y desfibrilación? • R = La primera se encuentra sincronizada con complejo QRS y la 2da no hay sincronización. • Cuáles son los agentes más comunes para miocarditis aguda? • R = VIRAL (COXACKIE) bacteriano, riketsias, espiroquetas, micoticos y parasitarios. • -A 23-year-old man develops sharp left-sided chest pain, fever, and a friction rub heard at the lower left sternal border, unaffected by respiration. The pain is also aggevated by lying down and relieved by sitting up. He is otherwise well with no other symptoms and the remaining physical examination is normal. Which of the following is the most likely cause for his symptoms? • (A) rheumatic fever • (B) tuberculosis (TB) • (C) herpes simplex virus • (D) MI • (E) coxsackievirus • Respuesta correcta E • La miocarditis Riketsial , por que enfermedad es originada? • R = TIFO, fiebre de las montanas rocallosas y fiebre Q.
  • 124. • Cual es el agente causal y cual es el periodo de incubación de la ENFERMEDAD DE CHAGAS? • R = Es causado por el TROFOSZOITO TRIPANOSOMA CRUZI y su principal manifestación clínica aparece en 10 años. • Que afección sistémica es la regla para un individuo que padece Chagas? • R = MEGAESOFAGO O MEGACOLON. • Entre las afecciones parasitarias cual es la mas frecuente de afección cardiaca? • R = La triquinosis. • Cual es la fisiología de la maniobra de valsalva? • R = Se produce un aumento del tono parasimpático por liberación de acetilcolina, lo que genera un retardo en la conducción AV causando un bloqueo AV transitorio. • Que dato puede ser único o inicial en el EKG en MIOCARDITIS INFECCIOSA? • R = Ectopia ventricular • Que medicamento esta indicado en el espasmo coronario inducido por la COCAÍNA? • R = BETABLOQUEADORES Y CALCIOANTAGONISTAS • Cuales son los síntomas de MIOCARDIOPATIA DILATADA? • R = Disnea, dolor torácico . • Cual es el manejo del paciente con CARDIOPATÍA HIPERTRÓFICA? • R = BETABLOQUEADORES, CALCIOANTAGONISTAS Y MARCAPASOS.
  • 125. • -A 45-year-old woman has developed increasing SOB on exertion and fatigue. She has a loud systolic ejection murmur heard best at the left sternal border, and the murmur increases with standing. A double apical impulse is also felt. • R = Cardiomiopatia hipertrofica. • 66. Cuales son las causas principales de MIOCARDIOPATIA RESTRICTIVA? • R = AMILOIDOSIS, RADIACIÓN, fibrosis por cirugía, sarcoidiosis, hemocromatosis y síndrome carcinoide. • -57 A 63-year-old man develops edema, and dyspnea on exertion. He has no prior cardiac or renal conditions, and his examination is significant for macroglossia, elevated jugular venous pressure (JVP), hepatomegaly, and 3+ pedal edema. His investigations reveal 3.5 g/d of protein in the urine, anemia, normal fasting glucose, and serum immunoelectrophoresis is positive for a monoclonal immunoglobulin. Which of the following is the most characteristic neurologic finding associated with this condition? • -Peripheral motor and sensory neuropathy: In addition to peripheral motor and sensory neuropathy, cardiac involvement, tongue enlargement, gastrointestinal (GI) manifestations, and carpal tunnel syndrome are also seen in amyloidosis. The specific diagnosis requires tissue biopsy with presence of amyloid with specific stains. In primary amyloidosis and myeloma, the amyloid protein is of the ALtype. In reactive amyloidosis, the protein is of the amyloid Aprotein (AA) type. • Cuales son los criterios mayores de Jones de CARDIOPATÍA REUMÁTICA? 1) CARDITIS: pericarditis, ICC, cardiomegalia y soplos. 2) ERITEMA MARGINADO 3) NÓDULOS SUBCUTÁNEOS: los nódulos subcutáneos SON INDOLOROS y menores de 2 cm 4) COREA DE SHYDENHAM: Movimientos coreoatetoides 5) ARTRITIS: dura de 1-5 semanas y responde a AINES • Cuales son los criterios menores de cardiopatía reumática? • R = Fiebre, VSG elevada, prolongación del PR. • Cual es el manejo de cardiopatía reumática? • R = PENICILINA BENZATINICA 1.2 millones DU o PEN-PRO 600, 000 x 10 dias. • Como se previene la fiebre reumática? • R = PEN-BEN 1.2 cada 4 semanas • En los pacientes con Wolf-Parkinson-Withe. Que medicamentos están contraindicados? • R = ADENOSINA, DIGOXINA, CALCIOANTAGONISTAS Y BETABLOQUEADORES YA QUE BLOQUEAN EL NODO AV.
  • 126. • En los pacientes con Wolf-Parkinson-Withe. Cual es el manejo indicado? • R = Cardioversión eléctrica y ablación por radiofrecuencia de vías accesorias. • Cuales son los síndromes de preexitacion? • R = Síndrome de WPW y Lown-Ganong-Levine • Cual es el microorganismo causal de la ENFERMEDAD DE LYME? • R = Borrelia Burgdorjeri • Que características tiene la pericarditis secundaria a síndrome urémico? • R = Pericardio afelpado, derrame hemorrágico y exudativo. • Que caracteriza al síndrome de Dressler? • R = Pericarditis post IAM o cirugía del corazón abierto y de etiología autoinmune. • Como se define la hipertensión pulmonar primaria? • R = Como aumento de la resistencia vascular pulmonar sin enfermedad subyacente. • -Auscultation of the heart of a 17-year-old boy reveals an increased intensity of the pulmonary component of the second heart sound. He complains of dyspnea on exertion but no other cardiac or pulmonary symptoms. Which of the explanations is the most likely cause of his dyspnea? • (A) pulmonary stenosis • (B) aortic stenosis • (C) MI • (D) pulmonary hypertension • (E) systemic hypertension • Respuesta correcta D • Cual es el manejo de hipertensión pulmonar primaria? • R = CALCIOANTAGONISTAS, anticoagulantes, prostaciclina (potente vasodilatador pulmonar)
  • 127. • Cual es el tratamiento de elección de la hipertensión primaria? • R = Endotelina • A que hace alusión el termino Cor-pulmonale? • R = Hipertrofia ventricular derecha e insuficiencia por enfermedad pulmonar, mas frecuente en EPOC • Cual es el dato de laboratorio mas frecuente en el Cor-pulmonale? • R = Policitemia • Cuales son los tumores que afectan al corazón? • R = Carcinoma broncogenico, carcinoma mamario, melanoma maligno, linfomas, carcinoma de células renales y sarcoma de kaposi. • Cuales son los tumores primarios del corazón mas frecuentes? • R = MIXOMA AURICULAR el cual es un tumor benigno que puede embolizar sistémicamente y se ubica comúnmente en la aurícula derecha. • -A 47-year-old woman has new-onset transient right arm weakness and word finding difficulty symptoms lasting 3 hours. She is also experiencing exertional dyspnea, and had a syncopal event 1 month ago. Her echocardiogram reveals a cardiac tumor in the left atrium, it is pendunculated and attached to the endocardium. Which of the following is the most likely cause of this lesion? • (A) myxoma • (B) sarcoma • (C) rhabdomyoma • (D) fibroma • (E) lipoma • Respuesta correcta A • En que consiste la ENFERMEDAD DE BURGUER? • R = Es un PROCESO INFLAMATORIO Y TROMBOTICO de las arterias y venas periféricas PRODUCIENDO CLAUDICACIÓN, DOLOR Y NECROSIS. • Cuales son los datos clínicos mas frecuentes de la enfermedad de Takayasu? • R = Soplos vasculares, PULSOS PERIFÉRICOS DISMINUIDOS y ASIMETRÍA DE LA PA DE LAS EXTREMIDADES. • Cual es el tratamiento de la enfermedad de Takayasu? • R = ESTEROIDES para la inflamación y después COLOCAR STENT O PUENTES.
  • 128. • Cual es el tratamiento para pacientes con displasia fibromuscular de la intima? • R = ASA • Que fenómenos deben descartarse en una persona que haya cursado con enfermedad de Raynaud? • R = 80% ESCLERODERMIA, MIOSITIS, LES, AR. • Cual es el síndrome doloroso regional o complejo tipo 1 distrofia simpática refleja? • R = DOLOR ARDOROSO O QUEMANTE de mas duración de lo esperado POR TRAUMATISMO EN EXTREMIDAD secundario a aplastamiento o quemadura. • Cuales son los datos clínicos de la distrofia simpática refleja? • R = DOLOR LOCAL, hiperestesia, calor, RIGIDEZ MUSCULAR, RIGIDEZ ARTICULAR, con consiguiente DESUSO Y OSTEOPENIA RADIOLÓGICA • Que tratamientos se utilizan en distrofia simpática refleja? • R = OPIOIDES Y GABAPENTINA • Cual es el tipo mas común de síndrome de salida torácica? • R = NEUROGENICO 90%, venoso y arterial. • En el síndrome de salida torácica , que es el síndrome de PAGET-SCHROETTER O TROMBOSIS DE ESFUERZO? • R = Se presenta como EDEMA UNILATERAL AGUDO DEL BRAZO, pesantez de axila, CIANOSIS DE LA MANO E INGURGITACIÓN DE LAS VENAS DEL HOMBRO Y TÓRAX. • Qué medidas se toman en un sujeto con choque que no responde a estímulos? • R = DEXTROSA AL 50%, naloxona 2 ml iv. Manteniendo la diuresis horaria mayor de .5 • Que te indica la PVC? • R = MENOR DE 5 mm hg indica HIPOVOLEMIA, MAYOR DE 18 SOBRECARGA VOLUMEN.
  • 129. • Cuáles son los datos clínicos de taponamiento cardiaco? • R = Taquicardia, HIPOTENSIÓN, pulso paradójico, AUMENTO DE LA PVY. Excepto en pacientes urémicos o con hipotiroidismo. • -A 25-year-old man complains of left precordial chest pain that radiates to the left shoulder but not down the left arm. The pain is accentuated by inspiration and relieved by sitting up. The pain is accompanied by fever and chills. His blood pressure is 105/75 mm Hg, pulse 110/min and regular, and temperature 37.5°C. Aside from the tachycardia, there are no abnormal physical findings in the heart or lungs. The ECG shows STsegment elevation in all leads except aVR and VI. On the third hospital day, the patient’s blood pressure falls, JVP rises, and he goes into CHF. Which of the following is the most likely diagnosis? • (A) a second pulmonary embolus • (B) extension of a myocardial infarct • (C) cardiac tamponade • (D) secondary bacterial infection • (E) rupture of a chordae tendineae • Respuesta correcta C • -A 56-year-old man presents with SOB, fatigue, and edema. He has also noticed weight gain, abdominal discomfort, and distension. He has a prior history of lung cancer treated with radiotherapy to the chest. There is no history of liver or cardiac disease in the past. On examination, he has an elvated JVP, prominent y descent of neck veins, and positive Kussmaul’s sign. The heart sounds are normal. The CXR shows a normal cardiac silhouette and the ECG has low voltages. • -A 55-year-old woman with metastatic lung cancer presents with dyspnea and pedal edema. On examination, the JVP is at 10 cm, with a negative Kussmaul’s sign. The heart sounds are diminished and the lungs have bibasilar crackles. The ECG shows QRS complexes of variable height • Como afecta el cliostazol en la enfermedad arterial periférica? • R = Es un INHIBIDOR DE LA FOSFODIESTERASA 3 que disminuye la agregación plaquetaria, activa la lipoproteína lipasa y causa vasodilatación.
  • 130. • Cuál es el objetivo del tratamiento en un paciente con choque? • R = PVC 8-12, PA ½ 65-90, índice cardiaco 2-4 lts/min, y O2 venoso > 70%. • Cuáles son los efectos de la dopamina? • R = 2-3 ug/kg/min estimula RECEPTORES DOPA y B AGONISTAS con AUMENTO DEL FG, FC Y IONOTROPISMO. >5 ug/kg/min estimula receptores ALFA adrenergicos produciendo VASOCONSTRICCIÓN PERIFÉRICA. • Cual es el medicamento de elección para choque cardiogenico? • R = DOBUTAMINA con aumento del ionotropismo y disminución de la poscarga • Que efecto tiene la desmopresina utilizada en choque distributivo? 1) Vasoconstricción periférica, disminución de la FC, vasodilatación coronaria, pulmonar y cerebral. 2) La vasoconstricción es mediada por oxido nítrico. • Cual es la causa de la miocardiopatía hipertrófica? • R = DEFECTO CONGÉNITO en los sarcomeros. • Cual es la causa de miocardiopatía dilatada? • R = Isquemia, cardiotoxicos. EMBARAZO. • Cual es la causa de la miocardiopatía restrictiva? • R = Primaria: fiebre endomiocardica, endocarditis de coffler o ideopatica, Secundaria. RADIACION. • -A 55-year-old woman is recently diagnosed with amyloidosis. She is now noticing increasing SOB, fatigue, and edema. On examination, the JVP is at 10 cm with a negative Kussmaul’s sign but prominent x and y descent. The blood pressure is 90/70 mm Hg, no pulsus paradoxus, pulse 100/min with low volume, and normal heart sounds. • -A 60-year-old man presents with SOB, increasing abdominal distention, and lower leg edema. He has no prior history of cardiac, renal, or liver disease. On examination, the JVP is at 8 cm with a negative Kussmaul’s sign but prominent x and y descent. The blood pressure is 95/75 mm Hg, no pulsus paradoxus, pulse 100/min with low volume, and normal heart sounds. There is shifting dullness of the abdomen and pedal edema. His blood glucose and hemoglobin A1C are elevated.
  • 131. • Que medicamentos están contraindicados en miocardiopatía restrictiva? • Digitalicos y agonistas B adrenérgicos. • Cuales son los criterios para ENDOCARDITIS BACTERIANA DE DUKE. • CRITERIOS MAYORES: • 1. Hemocultivos positivos para EI 1.1. Microorganismos típicos de EI en dos hemocultivos separados 1.1.1 ESTREPTOCOCO VIRIDANS S. BOVIS HACEK 1.1.2. S. Aureus o Enterococus adquiridos en la comunidad en ausencia de foco primario 1.2. Hemocultivos persistentes positivos 1.2.1. Hemocultivos extraidos con más de 12 horas de separación 1.2.2. La totalidad de tres, o la mayoría de cuatro o más hemocultivos separados siempre que entre el primero y el último haya al menos una hora • 2. Evidencia de afectación miocárdica 2.1. Ecocardiograma positivo 2.1.1. VEGETACIÓN EN VÁLVULA O ESTRUCTURAS ADYACENTES o en el choque del jet, o sobre dispositivos protésicos en ausencia de otra explicación anatómica 2.1.2. Absceso 2.1.3. Nueva dehiscencia parcial de una válvula protésica 2.2. Nueva regurgitación valvular (incremento o cambio en un soplo preexistente no es suficiente)
  • 132. • CRITERIOS MENORES 1. Predisposición. Una cardiopatía predisponente o ser ADVP. 2. FIEBRE > 38ºC 3. Fenómenos vasculares: émbolos en arterias mayores, infartos pulmonares, sépticos, aneurismas micóticos, hemorragia intracraneal, hemorragia conjuntival y LESIONES DE JANEWAY 4. Fenómenos inmunológicos (glomerulonefritis, NÓDULOS DE OSLER, MANCHAS DE ROTH Y factor reumatoide). 5. Ecocardiograma: sugestivos de eI sin alcanzar los criterios mayores antes mencionados. 6. Evidencia microbiológica (hemocultivos positivos que no cumplen los criterios mayores) o evidencia serológica de infección activa con un microorganismo que produce EI. • -A 28-year-old man develops viridans group streptococci septicemia. Which of the following cardiac lesions has the highest risk of developing endocarditis? • (A) ventricular septal defect • (B) atrial septal defect, secundum type • (C) mitral valve prolapse with regurgitation • (D) pure mitral stenosis • (E) asymmetric septal hypertrophy • Respuesta correcta A • En que consiste la PERICARDITIS CONTSTRICTIVA? • R = Constrictive pericarditis is characterized by a prominent y descent of the neck veins and LOW VOLTAGE ON ECG. THE PRESENCE OF A POSITIVE KUSSMAUL’S SIGN HELPS DIFFERENTIATE THE SYNDROME FROM COR PULMONALE AND RESTRICTIVE CARDIOMYOPATHIES. Apericardial knock is characteristic of constrictive pericarditis. It is in fact an early S3, occurring 0.06–0.12 seconds after aortic closure. S1 and S2 are frequently distant. • -A 56-year-old man presents with SOB, fatigue, and edema. He has also noticed weight gain, abdominal discomfort, and distension. He has a prior history of lung cancer treated with radiotherapy to the chest. There is no history of liver or cardiac disease in the past. On examination, he has an elvated JVP, prominent y descent of neck veins, and positive Kussmaul’s sign. The heart sounds are normal. The CXR shows a normal cardiac silhouette and the ECG has low voltages.
  • 133. • -A 64-year-old presents with dyspnea and edema. He had previous coronary bypass surgery 5 years ago, which was uncomplicated. Since then he has had no further chest pain. On examination, his JVP is at 8 cm, with prominent Kussmaul’s sign. The heart sounds are easily heard but there is an early diastolic filling sound (pericardial knock). • Cuál es el diagnostico diferencial entre miocardiopatia constrictiva y pericarditis restrictiva? 1) PERICARDITIS RESTRICTIVA: Puede ser causada por tuberculosis, Rx del tórax y cirugía cardiaca. 2) PERICARDITIS CONSTRICTIVA: Causada por un tamponade. 3) MIOCARDIOPATIA CONSTRICTIVA: Lo causa la amiloidosis, hemocromatosis, sarcoidiosis, esclerodermia, carcinoide.
  • 135.
  • 137.
  • 138.
  • 139.
  • 140.
  • 141. ESPONDILITIS ANQUILOSANTE • Que espondiloartropatias se asocian al HLA-B27?  R = ESPONDILITIS ALQUILOSANTE, ARTRITIS REACTIVA, ESPONDILOARTROSIS PSORIASICA, uveítis anterior aguda. • Cual es el síntoma inicial de la espondilitis alquilosante (EA) y que hallazgos son frecuentes en la ex fis?  R = DOLOR en regiones SACROILIACAS Y LUMBARES. En la ex fis HAY DISMINUCIÓN DE LA MOVILIDAD DE LA COLUMNA LUMBAR, dolor al presionar las regiones sacroiliacas y disminución de la expansión del tórax. • Cual es la manifestación extrarticular mas frecuente de la EA?  R = UVEITIS ANTERIOR, 2/3 de px tiene CAMBIOS EN LA MUCOSA Y SUBMUCOSA DEL COLON E ÍLEON TERMINAL • Cuales son los hallazgos más comunes en imagen obtenidas en la espondilitis anquilosante?  R = Erosiones y esclerosis sacroiliacas, IMAGEN EN COLUMNA DE BAMBU • Cual es el manejo de la EA?  R = AINES, COX-2, Infliximab, Etanercept mas terapia física. Indometacina, INFLIXIMAB que disminuye el FNTa.
  • 142.
  • 143. ENFERMEDAD DE REITER • Cuales son los datos clinicos más comunes de ARTRITIS REACTIVA o enfermedad de Reiter y con que HLA se relaciona?  R = Ligado al HLA B27, oligoartritis, CONJUNTIVITIS, URETRITIS, CERVICITIS, ULCERAS ORALES x un CUADRO INFECCIOSO PREVIO. • Que situación precede a una manifestación de ARTRITIS REACTIVA o síndrome de Reiter y cual es el patógeno > común?  R = Alguna INFECCIÓN por SALMONELLA, SHIGELLA, YERSINIA, CLAMYDIA TRACHOMATIS +++o CAMPYLOBACTER que PRECEDE a la MANIFESTACIÓN ARTICULAR 3-4 semanas después. • Cual es el estudio de laboratorio o gabinete de elección del síndrome de Reiter?  R = CULTIVO CON ANTIBIOGRAMA, se puede encontrar también Ac anti bacteriano en el suero o liquido sinovial o DNA bacteriano.
  • 144. ENFERMEDAD DE STILL • Que caracteriza clínicamente a la enfermedad de Still y que la precede?  R = VARIANTE DE LA ARTRITIS REUMATOIDE en la cual hay FIEBRE DE MAS DE 40 y con precipitación posterior varios grados debajo, ODINOFAGIA Y POLIADENOPATIAS. LO PRECEDE FARINGITIS. • Cual es una manifestación clínica característica de la enfermedad de Still?  R = EXANTEMA NO PRURIGINOSO, maculopapular de COLOR SALMÓN en TÓRAX Y ABDOMEN. (EL CUAL NO HAY EN REITER) • Cuales son los laboratorios distintivos de la enfermedad de Still, que inmunoglobulina se eleva y como afecta al FR?  R = LEUCOCITOSIS > 40, VSG AUMENTADA, hipergamaglobulinemia IgG y anemia. NO se detectan AUTOANTICUERPOS NI FR. • Cual es el manejo de la enfermedad de Still?  R = AINES
  • 145. ARTRITIS SORIASICA • Que es la artritis soriasica?  R = Ligado al HLA B27. Se define como una ARTRITIS usualmente SERONEGATIVA que se ASOCIA A SORIASIS. • Cual es el cuadro clínico de la artritis soriasica?  R = PRECEDE PSORIASIS A ARTRITIS, ARTRITIS ASIMÉTRICA con DEDOS EN APARIENCIA DE SALCHICHA (DACTILITIS) de los dedos de manos y pies. • Como diagnosticas artritis soriasica?  R = HC, y RX con evidencia de SACROILEITIS ASIMÉTRICA, alteraciones de la conformación de hueso nuevo con IMAGEN DE PUNTA DE LÁPIZ EN UNA COPA. • Cual es el manejo de la artritis soriasica?  R = AINES E INFLIXIMAB. Tratamiento de soriasis.
  • 147. LUPUS ERITEMATOSO SISTEMICO • Que HLA tienen relación con el LES? 1) HLA B8, HLA DR2 y DR3. 2) Los alelos de HLA están en el brazo corto del CROMOSOMA 6. • Que fármacos inducen la producción de anticuerpos antinucleares y LES?  R = HIDRALAZINA, procainamida, clorpromazina, ISONIAZIDA y diversos anticonvulsivantes. • Cual es el cc del LES en cada aparato – sistema y con que padecimientos se relaciona?  1. Se relaciona con: PTI, ANEMIA HEMOLITICA Y TIROIDITIS.  2. Constitucionales: la FATIGA es lo mas común  3. Mucocutaneas: 80%, LESIONES DISCOIDES observándose en cara, pabellones auriculares y el AREA DE V DEL ESCOTE. Eritema malar o en ALAS DE MARIPOSA QUE ES FOTOSENSIBLE. EL neonatal QUE SE PRESENTA EN LOS HIJOS DE PX CON LES POR EL PASO DE IgG CARACTERIZÁNDOSE POR LESIONES CUTÁNEAS Y BLOQUEO CARDIACO CONGÉNITO EN LOS PRIMEROS 6 MESES DE EDAD. ALOPECIA.  4. Musculoesqueletico: ARTRALGIAS con artritis. Osteonecrosis en la cabeza femoral, humeral, cóndilos femorales, carpo, tarso manifestándose clínicamente con dolor que mejora con el reposo.  5. Renal: nefritis lupica siendo la mas común la TIPO IV ¨GLOMERULONEFRITIS PROLIFERATIVA DIFUSA¨.  6. SNC: Meningitis aséptica, EVC, síndromes desmielinizantes y cefaleas.  7. Cardiopulmonar: En corazón con VEGETACIONES ENDOCARDICAS de LIBMAN-SACKS, en pleura lo mas común es pleuritis con o sin derrame pleural con niveles altos de proteínas DHL, complejos inmunes, ANA y células LE. LA VALVULA MAS AFECTADA ES LA MITRAL.  8. Gastrointestinal: Se puede presentar con SEROSITIS, disfagia y dispepsia muy a menudo.  9. Hematológico: PANCITOPENIA Y >TTP. Anemia por deficiencia de hierro MICROCITICA- HIPOCROMICA.
  • 148.
  • 150.
  • 151.
  • 153. POLIARTERITIS NODOSA • Cuales son los signos y síntomas MAS COMUNES de poliarteritis nodosa?  R = DOLOR EN EXTREMIDAD POR ARTRALGIA, MIALGIA QUE AFECTA PANTORRILLAS, puede afectar riñón causando HAS, IRA O HEMORRAGIA POR MICROANEURISMAS. MONONEURITIS MÚLTIPLE (PIE CAÍDO), corazón con IAM. Livides reticular, nódulos subcutáneos, y ulceras en piel. Son necesarios 3 de 10 criterios para su diagnostico. • Cuales son las pruebas de laboratorio en la poliarteritis nodosa y con que otro virus se relaciona habitualmente?  R = DESCARTAR VHBQUE SE ASOCIA EN 20-30%. ANCA (-) Y ANA (-) .Anemia, leucocitosis y trombocitosis. Se confirma diagnostico con toma de BIOPSIA DE LA LESIÓN. • Que datos se obtienen al tomar la biopsia en la poliarteritis nodosa?  R = BIOPSIA DE ARTERIA de mediano o pequeño calibre con PRESENCIA DE INFILTRADO GRANULOCITICO, PMN Y CÉLULAS MONONUCLEARES EN LA PARED DEL VASO. • Cual es el manejo de la poliarteritis nodosa?  R = ESTEROIDES, pero SI SE ASOCIA CON VHB SE AGREGAN RETROVIRALES.
  • 154. ARTERITIS DE LA TEMPORAL
  • 155.
  • 156.
  • 158. ARTRITIS SEPTICA PIOGENA • Cuales son los factores de riesgo para la artritis séptica piógena (gonocócica o no)?  R = Abuso de drogas IV, jóvenes sexualmente activos, enfermedades concomitante como DM, AR o lupus, antecedentes de cirugía o prótesis. • Cual es el cc de la artritis séptica piógena gonocócica o no?  R = Es una entidad de inicio agudo y progresivo caracterizado por dolor e inflamación local (salvo en el primer año de vida donde es similar a la sepsis). El dolor es moderado a intenso con limitación funcional, aumento de la temperatura local y rubor es monoarticular, cuando es poliarticular es en paciente inmunocomprometido siendo la rodilla la mas afectada. • Cuales son los hallazgos radiológicos en la artritis séptica piógena?  R = Edema de tejidos blandos y derrame articular. EN AFECCIÓN POR ANAEROBIOS SE VE GAS EN TEJIDOS BLANDOS. El gamagrama óseo con tecnecio (valora flujo) o con galio (valora actividad inflamatoria) pueden utilizarse de manera temprana. • Cual es el manejo de la artritis séptica piógena? 1) Consiste en la administración de antibióticos y drenaje de la articulación afectada. En caso de ARTRITIS PIÓGENA debe administrarse de 3 A 4 SEMANAS y en caso de ARTRITIS GONOCÓCICA DE 7 A 14 DÍAS. 2) En menores de 2M: DICLOXACILINA mas CEFOTAXIMA 3) En LACTANTES: DICLOXACILINA mas CEFOTAXIMA 4) En niños MAYORES DE 3ª: DICLOXACILINA 5) Artritis séptica GONOCÓCICA: 1g por via IM o IV cada 24 hrs. 6) Artritis séptica en DROGADICTOS por via IV: DICLOXACILINA mas GENTAMICINA 7) Artritis séptica en INMUNOCOMPROMETIDOS: CEFTAZIDIMA mas AMIKACINA 8) Artritis séptica POSTOPERATORIA: VANCOMICINA mas AMIKACINA 9) Artitis séptica en FRACTURA ABIERTA: AMOXICILINA con ACIDO CLAVULANICA
  • 160. RESUMEN DE LAS ENFERMEDADES METABOLICAS OSEAS
  • 161. CPK • Las siguientes patologías cursan con aumento de la CPK 1) HIPOTIROIDISMO 2) RAZA NEGRA 3) POLIMIOSITIS Y 4) DERMATOMIOSITIS
  • 163.
  • 164. ACALASIA • Que es la acalasia?  R = Es un trastorno por motilidad inadecuada que se caracteriza por una RELAJACIÓN INCOMPLETA y AUMENTO DE LA PRESIÓN BASAL DEL EEI, además de presentar APERISTALSIS DEL CUERPO. • Que causa la acalasia?  R = Es IDIOPÁTICO, aunque una de las hipótesis indica que es de origen central con DESTRUCCIÓN PARCIAL DEL NÚCLEO DORSAL DEL VAGO o sus fibras nerviosas a nivel del esófago con la siguiente alteración de la función. • Cuales son los síntomas más comunes de la acalasia?  R = DISFAGIA EN EL 90% DE LOS CASOS, INICIALMENTE A SÓLIDOS Y DESPUÉS A LÍQUIDOS. El segundo síntoma mas frecuente es la REGURGITACIÓN EN 75% mas acentuada en decúbito, EL DOLOR 20- 30% ES MENOS COMÚN, DE LOCALIZACIÓN EPIGÁSTRICA, RETROESTERNAL, E IRRADIADO HACIA CUELLO, DORSO Y EXTREMIDADES SUPERIORES. • Cuales son los estudios de gabinete necesarios para el diagnostico de acalasia?  R = La MANOMETRIA ES DE ELECCIÓN la cual nos muestra una PRESIÓN DEL EEI AUMENTADA (>69 MM/HG), SIN PRESENCIA DE ONDAS PERISTÁLTICAS A LA DEGLUCIÓN. La ESOFAGOGRAFIA CON BARIO MUESTRA ALTERACIONES RELACIONADAS CON LAS ONDAS PERISTÁLTICAS Y LA AUSENCIA DE RELAJACIÓN DEL EEI DANDO UNA IMAGEN EN PICO DE PÁJARO O EN PUNTA DE LÁPIZ característica. • Cual es el manejo de la acalasia? 1) MEDICO: A base de DILATACIONES HIDROSTÁTICAS O NEUMÁTICA, además de LA INYECCIÓN CON TOXINA BOTULINICA. 2) Quirúrgico: MIOTOMIA DE HELLER con una tasa de curación del 95%.
  • 165. ESPASMO ESOFAGICO DIFUSO • Que es el espasmo esofágico difuso?  R = Se trata de un trastorno motor esofágico que afecta fundamentalmente a la musculatura lisa y que se caracteriza por LA APARICIÓN DE ACTIVIDAD CONTRÁCTIL NO PROPULSIVA que alterna con episodios de peristalsis normal. • Cual es el cuadro clínico del espasmo esofágico difuso?  R = DOLOR TORAXICO CON LOCALIZACIÓN fundamentalmente RETROESTERNAL, aunque a veces SE IRRADIA A CUELLO Y A LOS HOMBROS SIMULANDO UNA ENFERMEDAD CARDIACA aumentando cuando están bajo estrés sicológico o emocional. La DISFAGIA es tan frecuente o mas que el dolor torácico. También es intermitente SE DESENCADENA CON FRECUENCIA CON LA TOMA DE SÓLIDOS O BEBIDAS FRÍAS. • Cuales son las alteraciones histológicas que esperas encontrar en el espasmo esofágico difuso?  R = HIPERPLASIA DE LAS CÉLULAS musculares y ocasionalmente mínimas ALTERACIONES a nivel de las fibras nerviosas terminales del PLEXO DE AUERBACH, no existiendo alteraciones significativas en las células ganglionares y nervios o en todo caso estas son mínimas. • Cuales son los hallazgos al gabinete en el espasmo esofágico difuso?  R = ESOFAGOGRAMA: Que nos muestra hallazgos característicos con APARICIÓN DE FRECUENTES ONDAS NO PROPULSIVAS en el musculo liso esofágico QUE INDENTAN LA COLUMNA DE BARIO y retrasan su evacuación, lo cual se describe como ESÓFAGO EN SACACORCHOS. MANOMETRIA: Alteraciones segmentarias, apareciendo generalmente en los DOS TERCIOS INFERIORES DEL CUERPO ESOFÁGICO. PHmetria obligatoria para descartar ERGE como causa del espasmo. • Cual es el manejo MEDICO Y QUIRURGICO del espasmo esofágico difuso? • MEDICO: NIFEDIPINO, ANTIDEPRESIVOS Y ANSIOLÍTICOS. • QUIRURGICO: La ESOFAGOMIOTOMIA AMPLIADA hasta el cayado aórtico puede ser una alternativa muy acertada EN CASOS GRAVES REFRACTARIOS a cualquier otro tratamiento.
  • 166.
  • 167. PERISTALSIS ESOFÁGICA SINTOMÁTICA O ESÓFAGO EN CASCANUECES • Que es la peristalsis esofágica sintomática o esófago en cascanueces y con que se relaciona?  R = Trastorno de la motilidad esofágica que se caracteriza por la PRESENCIA DE CONTRACCIONES DE GRAN AMPLITUD, SIN ALTERACIONES EN EL PERISTALTISMO del cuerpo esofágico y relajación del esfínter esofágico inferior. Se RELACIONA CON ERGE y a la histología no presenta alteraciones. • Cual es el cuadro clínico de la peristalsis esofágica sintomática o esófago en cascanueces?  R = DOLOR TORÁCICO RECURRENTE y con menor frecuencia disfagia. • Cuales son los hallazgos de gabinete en la peristalsis esofágica sintomática o esófago en cascanueces en la manometria o en el esofagograma?  R = El diagnostico se establece por la MANOMETRIA y se caracteriza por la aparición de ONDAS DE GRAN AMPLITUD (180 MM/HG), QUE OCASIONALMENTE TAMBIÉN SON DE LARGA DURACIÓN (6SEG), siendo condición indispensable que EL PERISTALTISMO ESOFÁGICO SEA NORMAL. Siempre se debe descartar ERGE realizando una PHmetria. NO HALLAZGOS CON ESOFAGOGRAMA. • Cual es el manejo de la peristalsis esofágica sintomática o esófago en cascanueces? 1) MEDICO: Si existe ERGE se recomiendo uso de OMEPRAZOL y evitar MIORRELAJANTES (nifedipino, diltiazem o nitratos) POR QUE EMPEORARÍAN EL ERGE. BENZODIAZEPINAS Y SOBRE TODO LOS ANTIDEPRESIVOS TRICICLICOS parecen útiles. 2) QUIRURGICO: ESOFAGOMIOTOMIA AMPLIADA HASTA EL CAYADO AÓRTICO puede ser una alternativa en pacientes intratables. La inyección de toxina botulinica tiene resultados muy pobres.
  • 168. EEI HIPERTENSO • Como se define el EEI hipertenso?  R = Como la existencia DE UN TONO O PRESIÓN media máxima basal DEL EEI IGUAL O SUPERIOR A DOS DEVIACIONES ESTÁNDAR, mas de 40-45 mm/Hg, siendo además condicionantes indispensables que existan RELAJACIONES COMPLETAS DEL EEI y que EL PERISTALTISMO ESOFÁGICO SEA NORMAL. • Cual es el cuadro clínico del EEI hipertenso?  R = DOLOR TORÁCICO O DISFAGIA. Mejorando generalmente el dolor tras el tratamiento, siendo la disfagia la mas difícil de tratar. • Cual es el manejo del EEI hipertenso?  R = RELAJANTES DE LA MUSCULATURA LISA
  • 169.
  • 170.
  • 172. CÁNCER GÁSTRICO • Cual es la estirpe etiológica mas común del cáncer gástrico?  R = ADENOCARCINOMA. • Cual es el cuadro clínico de un paciente con cáncer gástrico?  R = PERDIDA DE PESO, dolor abdominal, SACIEDAD TEMPRANA. Generalmente se presentan con palidez. Entre las manifestaciones dermatológicas mas frecuentes se encuentran la ACANTOSIS NIGRICANS y SIGNO DE LESER-TRELAT (queratosis seborreica de aparición súbita) • Como se manifiesta la metastasis comúnmente el cáncer gástrico?  R = GANGLIOS EN SUPRACLAVICULAR (Virchow), GANGLIOS PERIUMBILICALES (Hermana Maria Jose), estructuras rectales (signo del anaquel de blúmer), OVARIOS (tumor de Krukenberg). • Cual es el estándar de oro para el diagnostico del cáncer gástrico?  R = PANENDOSCOPIA con toma de 7 BIOPSIAS de la lesión y después TAC PARA ESTADIFICACION en caso de metástasis. • Cual es el manejo del cáncer gástrico?  R = Quirúrgico. La gastrectomía total se recomienda en caso de lesiones en el estomago proximal. La QUIMIOTERAPIA no mejora la sobrevida sobre la cirugía, sin embargo COMBINADA CON LA RADIOTERAPIA si la mejora.
  • 173. LINFOMA GASTRICO • 60. Cual es la estirpe etiológica mas común en el linfoma gástrico?  R = No HODKING 95% • 61. Cual es el cuadro clínico del linfoma gástrico?  R = Sintomatología no especifica como DOLOR ABDOMINAL, nausea, vomito, PERDIDA DE PESO, SUDORES NOCTURNOS y diarrea. • 62. Cual es la apariencia del linfoma gástrico a la endoscopia?  R = Aparecen como LESIONES FUNGOIDES, PÓLIPOS, ENGROSAMIENTO DE PLIEGUES. • 63. Cual es el manejo del linfoma gástrico?  R = Quirúrgico
  • 174.
  • 175.
  • 176. MALABSORCION • Cuales son los datos clínicos de malabsorción?  R = Diarrea, DISMINUCIÓN DE PESO y desnutrición, acompañados de DISTENCIÓN ABDOMINAL, síntomas y signos secundaros a los déficit específicos, ESTEATORREA. • Que datos de laboratorio se encuentran en un paciente con malabsorción?  R = Hipocolesterolemia, HIPOALBUMINEMIA, alteraciones electrolíticas y PH. • Cual es la posología de la prueba con D-Xilosa en malabsorción?  R = Se da EN AYUNAS midiendo su nivel en suero 1hr después y la excreción en orina en 6hr, este azúcar SE ABSORBE EN FORMA PASIVA SI LA MUCOSA INTESTINAL ESTA INTEGRA. • Cuales son las principales causas de malabsorción?  R = DEFICIENCIA DE LACTASA, ESPRUE celiaco, ESPRUE TROPICAL, ENFERMEDAD DE WIPPLE, sobre crecimiento bacteriano y síndrome de intestino corto.
  • 177. ESPRUE CELIACO • Que es el esprue celiaco?  R = Causa malabsorción debido a lesión ocasionada por el GLUTEN a nivel intestinal, el cual es contenido en el trigo, CEBADA, CENTENO Y AVENA. • Cual es el cuadro clínico del esprue celiaco?  R = Hay malabsorción de CHBTS, PROTEÍNAS Y GRASAS. Perdida progresiva de peso, distensión abdominal, flatulencia y diarrea, evacuaciones voluminosas y de mal olor. • Cuales estudios de diagnostico hay para el esprue celiaco y cuales son los anticuerpos especificos? 1) La biopsia en LA UNIÓN DUODENOYUYENAL que suele REPORTAR ATROFIA O ACORTAMIENTO DE LAS VELLOSIDADES, HIPERPLASIA DE LAS CRIPTAS y aumento en la mitosis CON INCREMENTO DE LINFOCITOS, NEUTROFILOS, MASTOCITOS Y EOSINOFILOS EN LAMINA PROPIA. 2) Cuantificación de grasa en heces de 24 hrs mayor a 7g/dia , la ABSORCIÓN DE D-XILOSA SUELE SER ANORMAL, los ANTICUERPOS SÉRICOS ANTIGLIADINA SON POSITIVOS EN 90%, el anticuerpo IgA es el mas especifico. 3) Los anticuerpos IgA ANTIENDOMISIO Y ANTIRRETICULINA son los MAS ESPECÍFICOS. • Cual es el manejo del esprue celiaco?  R = Dieta libre de gluten
  • 178. ESPRUE TROPICAL • Cual es la etiología del esprue tropical?  R = Es producido por la COLONIZACIÓN PERSISTENTE del intestino delgado por KLEBSIELLA PNEUMONIAE, ENTEROBACTER CLOACAE o E. COLI. • Cual es el cuadro clínico del esprue tropical? 1) CRISIS DE DIARREA ACUOSA, acompañada de DOLOR ABDOMINAL y FLATULENCIA, los cuales disminuyen de intensidad a la semana de evolución y posteriormente se hacen crónicos. 2) ANEMIA MEGALOBLASTICA. • Cuales son los estudios diagnósticos del esprue tropical? 1) La biopsia de intestino delgado reporta ALARGAMIENTO DE LAS CRIPTAS, ensanchamiento y acortamiento de las vellosidades e INFILTRACIÓN DE LA LAMINA PROPIA POR CÉLULAS INFLAMATORIAS. 2) Se encuentra esteatorrea, ABSORCIÓN DE D-XILOSA ANORMAL. • Cual es el manejo del esprue tropical?  R = TETRACICLINA y ACIDO FÓLICO para mejorar ANEMIA MEGALOBLASTICA.
  • 179. ENFERMEDAD DE WHIPPLE • Cual es el agente causal de la enfermedad de Whipple y con que HLA se relaciona?  R = Es secundaria a la infección por TROPHERYMA WHIPELLII y se relaciona con HLA B27 + en 25%. • Cual es el cuadro clínico de la enfermedad de Whipple?  R = Perdida de peso, diarrea o esteatorrea, artralgias, FIEBRE, dolor abdominal y HEMORRAGIA INTESTINAL, LAS ARTRALGIASSON MIGRATORIAS, no destructivas y afectan las grandes articulaciones, se presenta SACROILEITIS EN 25%. Un signo característico es la MIORRITMIA OCULOMASTICATORIA, la cual consiste en la convergencia rítmica de los ojos asociada a contracciones sincrónicas de los músculos de la masticación. A nivel ocular se manifiesta como UVEÍTIS, vitreitis, retinitis y papiledema. A la ex fis se encuentra LINFADENOPATIA periférica. • Cuales son los hallazgos de laboratorio en la enfermedad de Whipple?  R = La biopsia de intestino delgado muestra MACRÓFAGOS POSITIVOS CON LA TINCIÓN PAS. Sin embargo se debe demostrar el AGENTE CAUSAL EN EL LCR, tejido cerebral o GANGLIOS LINFÁTICOS. Anemia secundaria a enfermedad crónica, deficiencia de hierro, folato o VB12, asociada a neutrofilia. Prueba de D-Xilosa anormal, LINFADENOPATIA ABDOMINAL detectada por ultrasonido. Radiografía de TORAX CON INFILTRADOS, FIBROSIS O DERRAME PLEURAL. • Cual es el manejo de la enfermedad de Whipple y por cuanto tiempo se realiza?  R = TMP/SMZ durante 1ª, si hay ALERGIA A SULFAMIDAS esta indicada la PENICILINA por vía oral durante el mismo periodo y se deben tratar los trastornos nutricios.
  • 180.
  • 181.
  • 182. SX COLON IRRITABLE • Cuales son los criterios de ROMA II para integrar el síndrome de colon irritable?  R = Que los síntomas hallan durado POR LO MENOS 12 SEMANAS (no necesariamente continuas) durante los 12m previos de malestar o dolor abdominal que tiene 2 de las siguientes características. Se dice que el síndrome de intestino irritable se diagnostica cuando cumple al menos con 1 o mas de los criterios 2, 4 y 6 o bien 1, 3 y 5 DEPENDIENDO DE SI ES DIARREA LA QUE PREDOMINA O CONSTIPACIÓN. Se exacerba con el estrés y es MAS FRECUENTE EN MUJERES. 1. Menos de 3 evacuaciones por semana 2. Mas de 3 evacuaciones al día 3. Evacuaciones de consistencia dura 4. Evacuaciones sueltas o duras 5. Pujo al evacuar 6. Urgencia al evacuar 7. Sensación de evacuación completa 8. Moco durante la evacuación 9. Distención abdominal • Cual es el manejo del síndrome de colon irritable?  R = Debe ser INDIVIDUALIZADO a los síntomas. PInaverio con dimeticona.
  • 183. POLIPOS DEL COLON • Cual es la clasificación de los pólipos neoplásicos de colon?  R = A) TUBULARES 75% que corresponde a ADENOMAS. B) VELLOSOS 10% de los adenomas, mas relacionado con cáncer colonico y C) TUBULOVELLOSOS 15% de los adenomas. Requiriéndose un periodo mínimo de 10ª para que un adenoma se convierta a cáncer. • Cuales son las alteraciones genéticas implicadas en los adenomas y el cáncer de colon?  R = Mutación del GEN RAS y lesiones de los cromosomas 5, 17 y 18. • Cual es el cuadro clínico de un paciente con SÍNDROME DE PEUTZ-JEGHERS?  R = Es un TRASTORNO AD caracterizado POR MANCHAS HIPERPIGMENTADAS en los labios, mucosa bucal, cara y dedos, PRESENTANDO HAMARTROMAS EN TODO EL TUBO DIGESTIVO e INTUSUCEPCION CON STDB. • Cual es el manejo del Síndrome de Peutz-Jeghers?  R = Resección de pólipo sintomático por endoscopia • Cual es el cuadro clínico de la poliposis juvenil difusa?  R = Es un TRASTORNO AD caracterizado por una población homogénea de pólipos, tanto hamartromas como adenomas, los pólipos pueden ocasionar hemorragia e intususcepción en este síndrome. 10% RIESGO DE DESARROLLO DE CA DE COLON. • Cual es el manejo de la poliposis juvenil difusa?  R = RESECCIÓN del colon con y anastomosis ileorectal. SEGUIMIENTO CON PROCTOSCOPIA CADA 6M y extirpación de cualquier pólipo nuevo.
  • 184.
  • 185. • Cual es el cuadro clínico del SÍNDROME DE COWDEN? 1) Enfermedad AD caracterizada por HAMARTROMAS en todo el tubo digestivo, PAPULAS FACIALES Y BUCALES, CRECIMIENTOS QUERATOSICOS EN MANOS Y PIES. 2) Estos pacientes pueden presentar cáncer de mama, tiroides o de útero. • Cual es el cuadro clínico del SÍNDROME DE CRONKHITE-CANADA?  R = NO ES HEREDITARIO y se caracteriza por HAMARTROMAS INTESTINALES generalizados, ALOPECIA, PIGMENTACIÓN CUTÁNEA y ATROFIA DE NAILS. Los SÍNTOMAS INCLUYEN VOMITO, DIARREA, MALABSORCIÓN y enteropatía con perdida de proteínas, LA MAYORÍA DE LOS PACIENTES MUERE POCO DESPUÉS DEL DIAGNOSTICO. • Cual es el cuadro clínico de la poliposis adenomatosa familiar?  R = TRASTORNO AD y su defecto se localiza en el CROMOSOMA 5. Se caracteriza por la presencia de mas de 100 POLIPOS en todo el colon y recto, 100% DESARROLLA CÁNCER. • Cual es el manejo de la poliposis adenomatosa familiar?  R = Resección con RECTOCOLECTOMIA total con ileostomía. • Cual es el cuadro clínico de la ENFERMEDAD DE GARDNER?  R = POLIPOSIS ADENOMATOSA familiar que se asocia a OSTEOMIELITIS, QUISTES EPIDERMOIDES Y FIBROMAS DE LA PIEL. • Que es el SÍNDROME DE TURCOT?  R = POLIPOSIS ADENOMATOSA FAMILIAR que se asocia con NEOPLASIAS DEL SNC. • Como se manifiesta un Adenocarcinoma de colon ubicado en el lado derecho?  R = Perdida de peso, HEMATOQUZIA, anemia por déficit de hierro y UNA MASA ABDOMINAL EN EL CID. • Como se manifiesta un Adenocarcinoma de colon ubicado en el lado izquierdo?  R = Perdida de peso, cambios en hábitos intestinales, RECTORRAGIA y dolor abdominal tipo cólico. Se debe realizar una colonoscopia con biopsia.
  • 186.
  • 187.
  • 188.
  • 189.
  • 190.
  • 191.
  • 192. TROMBOSIS MESENTERICA • Que pacientes tienen el riesgo de desarrollo de trombosis venosa mesentérica?  R = Pacientes traumatizados, hipertensión portal, peritonitis o con hipercoagubilidad • Cual es el cuadro clínico de la trombosis venosa mesentérica?  R = DOLOR ABDOMINAL DIFUSO, DISTENCIÓN ABDOMINAL, nauseas, vómitos y signos de deshidratación • Cual es el estándar de oro para el dx de la trombosis venosa mesentérica?  R = TAC EN ESPIRAL con medio de contraste oral e IV • Cual es el manejo de la TVM?  R = ANTI COAGULACIÓN E HIDRATACIÓN.
  • 193. STDB
  • 194. TIPS STDA Y STDB o Los pacientes que requieran mas de 10u de paquetes globulares deberán recibir también PLASMA FRESCO CONGELADO, PLAQUETAS O AMBOS. o La colocación de BANDAS ELÁSTICAS es el tratamiento de elección en la rotura de VARICES ESOFÁGICAS. o El LAVADO NASO GÁSTRICO se utiliza para DIFERENCIAR entre el STDA Y STDB. o La POSITIVIDAD EN EL ASPIRADO del tubo naso gástrico, RUIDOS INTESTINALES HIPERACTIVOS y elevación de los niveles del BUN apoyan el diagnostico de STDA. o En la hematemesis secundaria a la ingesta de AINES se debe realizar una esofagogastroscopia y administrar un inhibidor de la bomba. o El hematoma retroperitoneal produce una IMAGEN DE VIDRIO DESPULIDO en la radiografía de abdomen.
  • 195. ABSCESO ANAL • Cuales son las causas de absceso anorrectal especificas y no especificas?  R = ESPECIFICAS: Chron, CUCI, Tb, actinomicosis, ca, leucemias. NO ESPECIFICAS: Infección bacteriana de glándulas anales • Cual es la complicación principal que sucede al drenar un absceso anal?  R = Posterior formación de FISTULAS • Cual es el cuadro clínico en un px que presenta un absceso anal?  R = DOLOR ANAL INTENSO o una TUMORACIÓN ANAL MUY DOLOROSA que apareció en forma reciente. • Cual es el manejo de un absceso perianal?  R = DRENAJE Y METRONIDAZOL O CIPROFLOXACINO.
  • 197. FISTULA ANAL • Cual es la etiología de una fistula anal?  R = Abscesos crónicos, CHRON Y CUCI. • Cual es el cuadro clínico de las fistulas anales?  R = Es característico la historia de un ABSCESO PREVIAMENTE DRENADO en la misma localización, DOLOR PERIANAL, la principal molestia es la PERMANENTE SALIDA DE MATERIAL PURULENTO a través de un orificio que MANCHA LA ROPA INTERIOR. • Cual es el estudio de imagen utilizado para el diagnostico de las fistulas anales?  R = ULTRASONIDO RECTAL • Cual es el manejo de las fistulas anales?  R = La forma para predecir por donde drena la fistula es la regla de GOODSALL- SALMON y una vez identificada se realiza FISTULOTOMIA.
  • 198. FISURA ANAL • Cual es la etiología de las fisuras anales?  R = ISQUEMIA EN MUCOSA POR ESFÍNTER HIPERTÓNICO. • Cual es el cuadro clínico de las fisuras anales?  R = DOLOR intenso TIPO ARDOROSO que se relaciona al inicio de la evacuación con una SENSACIÓN DE DESGARRO al pasar el bolo fecal. SANGRADO FRECUENTE, suele ser en POCA CANTIDAD. • Cual es el manejo de las fisuras anales?  R = Baños de asiento, laxante, RELAJANTE DE MUSCULATURA PÉLVICA COMO CREMA DE ISOSORBIDE, DILTIAZEM O NIFEDIPINO.
  • 201. HERNIAS ABDOMINALES • De donde emerge la hernia inguinal directa?  R = Emerge DENTRO DE LOS VASOS EPIGÁSTRICOS originándose en el piso del conducto inguinal • De donde emerge la hernia inguinal indirecta?  R = Por FUERA DE LOS VASOS EPIGÁSTRICOS y son DE ORIGEN CONGÉNITO siendo mas FRECUENTES DEL LADO DERECHO. • De que manera se distingue a la ex fis una hernia inguinal directa de indirecta?  R = Una HERNIA INDIRECTA EJERCE CIERTA FUERZA CONTRA LA PUNTA DEL DEDO, en tanto que la DIRECTA PRESIONA LA PULPA DEL DEDO. Además cuando es colocado el dedo en el orificio inguinal y pedirle al paciente que tosa es posible parar la hernia indirecta y no así la directa. • Cual es la localización de la hernia crural?  R = DEBAJO DEL LIGAMENTO INGUINAL, medial a los vasos femorales manifestándose CON DOLOR EN LA REGIÓN INGUINAL, progresivo e incapacitante.
  • 202.
  • 203.
  • 204. CAUSAS DE ELEVACION DEL AST O ALT
  • 205.
  • 206.
  • 208. VHA-PICORNAVIRUS • Cual es el dato de laboratorio que orienta al VHA?  R = IgM VS VHA AGUDO E IgG CUANDO YA HABÍA SIDO INFECTADO EL PACIENTE • Cual es el manejo del VHA?  R = Sintomático, Inmunización pasiva como profilaxis postexposicion. • Cuales son las indicaciones de la VACUNA PASIVA del VHA?  R = PERSONAS QUE PLANEAN UN VIAJE, contactos íntimos, PERSONAL DE GUARDERÍAS, ASILOS E INSTITUCIONES. NO esta INDICADA en px que tienen DATOS CLÍNICOS DEL VHA • Cuales son las indicaciones de la VACUNA ACTIVA del VHA?  R = ENFERMEDAD HEPÁTICA CRÓNICA, VARONES HOMOSEXUALES, DROGADICTOS.
  • 209. VHB- HEPADNAVIRIDAE • Cual es el dato de laboratorio que te orienta a VHB? 1) HBs Ag, la persistencia de >6m sugiere infección crónica. 2) Anti HBsconfiere inmunidad protectora ante el virus. 3) Hbe Ages el antígeno que refiere la REPLICACIÓN ACTIVA del virus y la CONTAGIOSIDAD, en px con infección crónica permanece constante. 4) IgM antiHBc infección aguda. 5) - 10% EVOLUCIONA A CRONICIDAD, BUEN PRONOSTICO. TX EN CRONICO • Cual es el manejo del VHB? 1) INTERFERON ALFA 2) LAMIVUDINA (análogo de nucleosidos que inhibe la TRANSCRIPTASA REVERSA), 3) ADEFOVIR (análogo de nucleótidos sintético que inhibe a la DNA polimerasa del VHB), 4) ENTECAVIR ( análogo nucleosido deoxiguanina que inhibe la actividad del DNA polimerasa del VHB).
  • 210.
  • 211. VHD • Que caracteriza al VHD?  R = Es EL VIRUS MAS PEQUEÑO con 36 nm, NECESITA DE LA ENVOLTURA DEL VHB para sobrevivir y su medio de CONTAGIO ES EL MISMO QUE EL DEL VHB. • Cual es el curso del VHD en coinfección aguda con VHB?  R = Tiende a ser LIMITADO Y ELIMINAR EL HBsAg POR LA RESPUESTA INMUNE, al mismo tiempo que desaparece el VHD. • Cual es el curso del VHD en súperinfección con VHB?  R = Exposición del VHD EN INDIVIDUO PREVIAMENTE INFECTADO POR VHB. LA REPLICACIÓN DEL VHD ES MAYOR Y MÁS RÁPIDA al encontrar previamente expresado al HBs Ag. En 70% de pacientes de los casos la infección se vuelve crónica y se encuentran mayores tasas de mortalidad • Cual es el estudio de laboratorio que orienta al diagnostico del VHD?  R = IgG anti VHD o por detección de RNA DE VHD EN SUERO. • Cual es el manejo del VHD?  R = No hay tratamiento especifico, se ha encontrado mejoría con INTERFERON ALFA.
  • 213. FAMILIAS DE HEPATITIS VIRICA • VHA PICORNAVIRUS • VHB HEPADNAVIRUS • VHC FLAVIVIRUS • VHE CALICIVIRUS
  • 214. HEPATITIS AUTOINMUNE • Que predisposición genética tienen las personas que padecen hepatitis autoinmune?  R = HLA-B8, HLA-DR3 Y DR-52. • Que clasificaciones tiene la hepatitis autoinmune?  TIPO I: Autoinmune clásica o Lupoide, se presenta en mujeres jóvenes donde 30% presenta trastornos como TIROIDITIS AUTOINMUNE O CUCI. A diferencia de otros grupos se presenta HIPERGAMAGLOBULINEMIA EN SUERO 5-6 GR/DL.  TIPO II: Mas común en Europa y los anticuerpos que distinguen a este grupo son los ANTIMICROMOSOMA DE HÍGADO Y RINON TIPO 1 (ANTI LKM 1), asociada a enfermedades inmunológicas como el VITÍLIGO, DM 1, TIROIDITIS AUTOINMUNE.  TIPO III: Se encuentran anticuerpos anti antígeno hepático soluble/hígado- páncreas (anti-SLA/LP) encontrando mas frecuentemente la TIROIDITIS AUTOINMUNE COMO ASOCIADA. • Cual es el cuadro clínico de las hepatitis autoinmunes?  R = MÚLTIPLES TELANGIECTASIAS, astenia, ARTRALGIAS, ESTRIAS CUTÁNEAS, acné, hirsutismo, AMENORREA Y HEPATOMEGALIA. • Cuales son los CRITERIOS diagnósticos para hepatitis autoinmune? 1) MAYORES: Elevación persistente de transaminasas, Hipergamaglobulinemia 2.5 o mayor, Ac circulantes y/o células LE positivas, marcadores serológicos para virus de hepatitis negativos. 2) MENORES: Manifestaciones sistémicas como fiebre-artralgias-erupciones cutáneas, Complicaciones con otras enfermedades autoinmunes , incluyendo las del colágeno, eritrosedimentacion elevada. 3) HISTOLOGIA: Hepatitis crónica o cirrosis con marcada infiltración de células y necrosis de hepatocitos, ausencia de hallazgos específicos para el diagnóstico de otras entidades. • Cual es el manejo de la hepatitis autoinmune?  R = De elección son los GLUCOCORTICOIDES SOLOS O COMBINADOS CON AZATRIOPINA. La remisión se considera cuando los px demuestran mejoría clínica, niveles de transaminasas y gammaglobulinas normales en caso de recidivas se usa tratamiento inmunosupresor de por vida.
  • 215.
  • 216. HEPATITIS ALCOHOLICA • Cual es el cuadro clínico de un paciente con hepatitis alcohólica?  R = Perdida de peso, anorexia, ICTERICIA, FIEBRE Y HEPATOMEGALIA DOLOROSA. • Cuales son los hallazgos de laboratorio en la hepatitis alcohólica? 1) AUMENTO DE LAS ENZIMAS HEPÁTICAS 2) ANEMIA 3) TROMBOCITOPENIA 4) LEUCOCITOSIS CON PREDOMINIO DE NEUTROFILOS Y 5) SE CONFIRMA EL DIAGNOSTICO POR BIOPSIA. • Cual es el manejo de la hepatitis alcohólica?  R = Abstinencia, CORTICOIDES y plan nutricio.
  • 217. CIRROSIS HEPATICA • Que parámetros definen la presión portal?  R = Flujo venoso portal y la resistencia interior del hígado • Cuales son las causas mas frecuentes de cirrosis hepática?  R = Alcohol, HEPATITIS B Y C, METOTREXATO, alfametildopa e hidralazina. • Cual es la clasificación morfológica de la cirrosis? 1) CIRROSIS MICRONODULAR: Nódulos menores de 3 mm. 2) CIRROSIS MACRONODULAR: Nódulos de mas de 3 mm. 3) CIRROSIS MIXTA: Mircro y macronodulares que ocurre comúnmente por alcohol. • Cual es el marcador mas útil para el diagnostico de cirrosis biliar primaria?  R = ANTICUERPOS ANTIMITOCONDRIALES.
  • 218. • Cual es el cuadro clínico de la cirrosis?  R = Asintomatico o los siguientes: A. SINTOMAS CONSTITUCIONALES: B. Astenia, anorexia, perdida de peso muscular. C. ASCITIS: D. Por retención de sodio y agua. E. SIGNOS CUTANEOS: F. Ictericia, palidez por la anemia. Telangiectacias cutáneas y eritema palmar (que predomina sobre las eminencias tenar e hipotenar y en las llemas de los dedos) son datos bastantes caracteristicos. G. CIRCULACION COLATERAL: H. Los sitios mas comunes de aparición con significado patológico son el esófago y el fondo gástrico. I. ALIENTO: J. Olor dulzón debido a la exhalación de mercaptanos, productos derivados de la metionina por defecto de su metilación. K. ALTERACIONES ENDOCRINAS: L. Varones con signos de feminización como ginecomastia y cambios en la distribución del vello coroporal. El hipogonadismo en varones produce atrofia testicular, perdida de la libido, impotencia. En ambos sexos el vello axilar es escaso. En las mujeres es frecuente la oligomenorrea, amenorrea y esterilidad. M. ALTERACIONES HEMATOLOGICAS: N. Alteraciones en la coagulación dado que todos los factores se sintetizan en hígado menos el factor de von Willebrand. O. ALTERACIONES PULMONARES: P. Ascitis torácica o derrame pleural Q. ALTERACIONES DIGESTIVAS: R. Hemorragia digestiva, colelitiasis. S. ALTERACIONES RENALES: T. Retención de sodio con ascitis U. SX HEPATORRENAL: V. Se trata de una insuficiencia renal funcional como respuesta a mecanismos de descompensación tales como el uso exagerado de diuréticos, AINES, paracentesis. La histología es normal. Este síndrome se caracteriza por una insuficiencia renal funcional con función tubular normal en un px con insuficiencia hepática. W. INFECCIONES: X. IVU mas comunes. Y. ALTERACIONES NEUROLOGICAS: Z. Encefalopatia hepática donde la primer manifestación en aparecer son los cambios en el comportamiento. El GABA es el principal neurotransmisor inhibitorio del cerebro, se ha postulado que el GABA producido a nivel intestinal escapa del metabolismo hepático, atraviesa la barrera hematoencefalica y penetra al cerebro desencadenando la encefalopatía hepática.
  • 219. • 184. Cual es el manejo de la cirrosis hepática?  R = Dieta con contenido de proteínas 1 a 1.2 g/gr. Prohibicion del consumo de alcohol. Hay tratamientos utiles específicos de cirrosis, como las flebotomías en la hemocromatosis, la d-penicilamina en la enfermedad de Wilson, los corticoides en las cirrosis autoinmunes, los antivíricos para cirrosis por VHB y VHC y el acido urodesoxicolico para la cirrosis biliar primaria. La cirugía se asocia a una mortalidad operatoria del 30% pero las indicaciones para la cirugía son rotura de varices o sangrados. A las varices tipo 2 y 3 se administra betabloqueador en caso que no exista contraindicación. Seguimiento ecográfico cada 6m para detectar hepatocarcinoma. • 185. Cual es el manejo de emergencia en la cirrosis hepática?  R = En caso de sangrado alto una endoscopia debe realizarse para dar escleroterapia. Uso de vasopresina por via IV teniendo cuidado de la necrosis de la piel. Anastomosis portosistemica intrahepática por via transyugular TIPS para controlar la hemorragia digestica por varices esofágicas irreductible por otros métodos.
  • 220. HEPATOCARCINOMA • Cuales son los factores de riesgo para desarrollar un hepatocarcinoma?  R = Aparece en personas que ya tienen alguna enfermedad hepática como HEPATITIS POR VIRUS B O C, CIRROSIS. • Cual es el cuadro clínico de un px con hepatocarcinoma?  R = Puede cursar asintomático. Cuando existen manifestaciones lo mas habitual es que se presenten síntomas inespecíficos como baja de peso, dolor abdominal, saciedad precoz o masa palpable. Cuales son los estudios de gabinete utilizados para diagnosticar hepatocarcinoma?  R = TAC DINÁMICA HELICOIDAL MULTICORTE DE ELECCIÓN. Us de seguimiento. La ALFAFETOPROTEINA es el marcador tumoral mas utilizado para el dx y seguimiento (puede elevarse en embarazo y tumores testiculares). Confirmación por biopsia. • Cual es el manejo del hepatocarcinoma?  R = RESECCIÓN del tumor. INYECCIÓN DE ETANOL en tumor lo reseca y es fácil de realizar. QUIMIOEMBOLIZACION que frecuentemente se complica con el llamado síndrome postembolizacion caracterizado por fiebre, elevación enzimática, dolor abdominal y nauseas. TRANSPLANTE HEPÁTICO.